You are on page 1of 906

(Anatomy)

Thoracic Organs System


Part Anatomy 2005
1.

mitral valve
A. midsternum
B. 2nd intercostals space left at parasternum

C. 4th intercostals space left at parasternum


D. 4th intercostals space right at parasternum
E. 5th intercostals space right at mid clavicular line(
left)
2.

lymph node

superior
cervical ganglion
A.
B.
C.
D.
E.
Thoracic Organs System
Part Anatomy 2006

1.

metastasis
A. apical
B. axillary
C. supraclavicle
D. intraclavicle
E. Thoracic
2.

45
20


obstruction
A. Trachea
B. Main bronchus
C. Medium bronchus
D. respiratory bronchiole
E. alveoli
3.

25


A. Diaphragm
B. Pericardium
C. Pleura
D. Intercostal muscle
E. Pulmonary parenchymal
4.

40 4

A. Nose
B. Nasopharynx
C. Trachea
D. bronchus
E. alveoli
5.

superior lobe

A. ICS 4
B. ICS 5
C. ICS 6

D. ICS 4
E. ICS 5
6.

55 1-2 20

Bronchus
A. Simple sqaumous
B. Simple cuboidal
C. Simple columnar
D. Stratified columnar
E. Stratified squamous
7.

18 10
intercostal space 5
midclavicular line
A. Superior vena cava
B. Aortic valve ascending aorta
C. Pulmonary valve pulmonary aorta
D. Tricuspid valve Right atrium
E. Mitral valve Left ventricle

8.

axillary artery

axillary

combine metabolic and respiratory acidosis artery

A. brachial artery
B. internal thoracic artery
C. subclavian artery
D. subscapular artery
E. vertebral artery
9.

aortic arch
pharyngeal arch

10.

11.

double aortic arch

Echocardiogram Double aortic arch


arch
A. 1
B. 2
C. 3
D. 4
E. 6
Thoracic Organs System

Part Anatomy 2007


1.

50 pleural effusion effusion

anterior Axillary line pleural reflexion


rib
A. 7
B. 8
C. 9
D. 10
E. 11
2.

30 2 fine

crepitation

A. Alveoli
B. Trachea

C. Main bronchus
D. Lobar bronchus
E. Segmented bronchus
3.

40
2 4

A. Nose
B. nasopharynx
C. Trachea
D. bronchus
E. alveoli

Thoracic Organs System



- pneumonia lobe stethoscope

root of spine of scapula


rib

left lung

root of spine of scapula


rib

left lung

1. upper lobe
2. lower lobe
th

(Snell RS. Clinical anatomy by regions. 8 .2008:6872.)


Abdominal Organs System
Part Anatomy 2005
1.

40

30
mildly pale,
no jaundice, palmar
erythema, spider nevi, impalpable liver , spleen 3 cm
under left costal margin,

ascites positive

A. portal vein
B. hepatic vein
C. hepatic artery
D. superior vena cava
E. inferior vena cava
2.

40 liver biopsy 10

nerve
3.

Esophageal stenosis syndactyly

A. migration
B. apoptosis
C. proliferation

D. differentiation
Abdominal Organs System
Part Anatomy 2006
1.

Organell drug detoxification


A. peroxisome
B. Nucleus
C. golgi complex
D. mitochondria
E. SER

2.
ultrasound small
sliding hiatal hernia


A. incarcerated hernia
B. strangulated hernia
C. esophageal reflux
3. 45





A. pancreas
B. large

intestine

C. Gall Bladder
D. Gastric parietal cell
E. Lower esophageal sphincter
4.

ascites

A. Portal vein
B. Splenic vein
C. Hepatic vein
D. IVC
5. chronic liver disease
4 cm.
ascites
A. Portal vein
B. Splenic vein
C. Hepatic vein
D. Mesenteric vein
E. Inferior venacava

6.

70



inferior mesenteric
artery 90%

Descending colon

A. Left gastroepiploic artery


B. Middle colic artery
C. Splenic artery
D. Sigmoid artery
E. Superior rectal artery
7.

laryngotracheal tube primitive

foregut
A. Laryngotracheal groove
B. Posterior esophageal fold
C. Bronchopulmonary segment
D. Tracheoesophageal septum
E. Laryngotracheal diverticulum
8.

24 wk Oligohydramnios

A. real agenesis
B. esophageal atresia
C. anenephaly
Abdominal Organs System
Part Anatomy 2007
1.

mediolateral episiotomy muscle


A. bulbospongiosus
B. ischiocavernosus
C. superficial transverse peroneal
D. deep transverse peroneal
E. external anal sphincter

2.

50 10 epigastric

A. stomach

B. liver
C. pancreas
D. small intestine
E. gallbladder
3.

20

transverse
abdominis
A. external abdominis muscle
B. Internal abdominis muscle
C. Rectus abdominis
D. Transversalis fascia
E. Peritoneum
4.

peristaltic sound trachea


A. later body wall
B. septum tranversum
C. costodiaphragmatic recess
D. pluroperitoneal membrane
E. dorsal mesentery of esophagus
5.

A. Urachal fistula
B. Omphalocele
C. gastrichisis
D. neuroblastoma
6.



A. 1/4 ASIS

B. 1/4 AIIS
C. 1/3 ASIS
D. 1/3 AIIS
E. 1/4 iliac crest
7. 30 3
BP

150/90 mmHg. , moon face,

central obesity, hyperpigmentation,


Fasting Blood Glucose 180 mg/dl Adrenal
gland
A. Adrenal capsule
B. Zona glomerulosa
C. Zona fasciculata
D. Zona reticularis
E. Adrenal medulla
Abdominal Organs System




- uterus structure
Brain and Nervous System
Part Anatomy 2005
1.

left homonymous
hemianopia
A. left optic nerve
B. optic chiasma
C. right optic tract
D. left optic radiation
E. occipital lobe
2.

axillary artery

A. brachial artery

B. internal thoracic artery


C. subclavian artery
D. suprascapular artery
E. vertebral artery
3.

sciatic

nerve
A. upper outer quadrant
B. upper inner quadrant
C. midgluteal
D. lower outer quadrant
E. lower inner quadrant
4.

femoral vein

lateral
medial
A. vein artery nerve ring

B. vein artery ring nerve


C. artery vein nerve ring
D. nerve artery vein ring
E. vein artery ring nerve
5.

35

(foot drop)
nerve
A. tibial nerve
B. femoral nerve
C. superficial peroneal nerve
D. deep peroneal nerve
E. centriole
6.

50


A. basilar artery
B. vertebral artery
C. anterior cerebral artery
D. middle cerebral artery
E. posterior cerebral artery
7.

medulla
8.

12 20 cervical spine

fracture 2

spinal cord
A. posterior column
B. Lissauer tract
C. corticospinal tract

D. spinothalamic tract
E. spinocerebellar tract
9.

25 cavernous sinus

sign
A. ptosis of right eyelid
B. loss of corneal reflex
C. engorgement of the retinol vein
D. constrictive pupils response
E. loss of all extraocular movement of right eye
10.

70 paralysis sensory deficit

perineum
thrombosis

A. right posterior cerebral artery


B. right middle cerebral artery

C. right anterior cerebral artery


D. recurrent artery of Henbner
E. basilar artery
11.

30 2

2

A. peripheral nerve
B. anterior horn cells
C. neuromuscular junction
D. sympathetic chain
E. muscle
12.

cavernous sinus

A. right ptosis

B. constricted pupil response to light


C. loss of corneal blink reflex
D. engorged retinal vein
E. loss of right external ocular movement
13.

50

corneal reflex negative


cranial nerve
A. CN II
B. CN III
C. CN IV
D. CN V
E. CN VI
14.

60 horizontal nystagmus

A. inferior olive

B. vestibular nucleus
C. medial lemniscus
D. nucleus ambiguous
E. superior cerebelar peduncle
15.

50 complete cord injury T10

sensation
A. nipple line
B. costal margin
C. xyphoid process
D. umbilicus
Brain and Nervous System
Part Anatomy 2006
1.

A. Broca area
B. Motor area
C. Angular gyrus
D. Hippocampus
E. Wernicke area
2.

visual field

A. optic chiasma
3.

60
3

A. Red nucleus
B. Basal ganglion
C. Dentate nucleus
D. Pontocerebellar fiber
4.

65

ptosis
deviate lateral pupil

5 mm. hyperreflexia
lesion
A. Cerebral artery
B. Midbrain
C. Pons
D. Medulla
E. Cerebellar hemisphere
5.

40 16 wk
AFP
A. cleft palate cleft lip
B. Neural Tube Defect
C. Tetralogy of fallot
D. duodenal atresia

6.

CT scan
lateral third ventricle

A. Hydrocephalus

B. Hydranencephaly
C. Meroanencephaly
D. meroencephaly
E. Holocephaly
F. haloposencephaly
g.

Meningohydroenephalocele

Brain and Nervous System


Part Anatomy 2007
1.

60

10
right
hemiparesis with motor aphasia

A. anterior cerebral artery right


B. anterior cerebral artery left

C. middle cerebral artery right


D. middle cerebral artery left
E. posterior cerebral artery right
2.

right hemiparesis + motor aphasia

A. right anterior Cerebral artery


B. right middle cerebral artery
C. right posterior Cerebral artery
D. left anterior Cerebral artery
E. left middle cerebral artery
3.

70

Alzheimer

A. cerebropontine
B. cerebellum

C. hippocampus
D. medulla oblongata
E. globus pulidus
4.

resting tremor

Metabolism

A. cerebellum
B. hippocampus
C. basal ganglia
D. anterior horn cell
E. mammillary body
5.

Alzheimer


A. crus cerebri
B. cerebellum

C. hippocampus
D. Globus pallidus
Brain and Nervous System



1.
medial CN
2.

Musculoskeletal System
Part Anatomy 2005
1.

35 full thickness of

skin lodal flap with skin

and subcutaneous tissue


A. basal layer
B. deep dermis
C. subcutaneous fat

D. stratum corneum
E. stratum lucidum
2.

16

flexor

Carpi radialis, flexor digitorum superficialis

3.

A.

extent

B.

flexion

C.

abduct adduct

D.

opponent

E.

sensory

femur



A. skin
B. muscle

C. vessel
D. nerve
E. bone
4.

60 2






A. hamstrings
B. quadriceps
C. gluteus maximus
D. biceps femoris
E. iliopsoas
5.

50 (eversion)

lesion
A. gastrocnemius, soleus
B. plantaris, quadratus plantae

C. tibialis anterior, tibialis posterior


D. peronius longus, peroneus brevis
E. extension digitorum longus,flexion digitorum longus
6.

A. axillary nerve
B. median nerve
C. musculocutaneous nerve
D. radial nerve
E. ulnar nerve
7.

40 total thyroidectomy

A. cervical nerve
B. recurrent laryngeal nerve

C. pharyngeal nerve
D. thoracic nerve
E. thyroglossal nerve
8.

A. superficial inguinal node


B. deep inguinal node
C. external iliac node
D. internal iliac node
E. lumbar node
9.

A. ulnar nerve
B. deep branch of ulnar nerve
C. superficial branch of ulnar nerve

D. recurrent branch of median nerve


E. radial nerve
10. 30 gluteal area

A. iliopsoas
B. hamstrings
C. gluteus minimus
D. gluteus maximus
E. obturator internus
11.
carpal bone radial

bone

ulnar bone humeral bone


radial

bone ulnar bone

A. annular ligament
B. flexor retinaculum

C. bicipital aponeurosis
D. intermuscular septum
E. interosseous membrane
12.

syndactyly

A. apoptosis
B. fusion
C. proliferative
D. migration
E. differentiation
13.

axillary nerve
A. medial side of forearm
B. lateral side of forearm
C. medial side of arm

D. lateral side of arm


E. all
Musculoskeletal System
Part Anatomy 2006
1.

Lumbar puncture 20


vertebra

A. L2
B. L3
C. L4
D. L5
E. L6
2.

50

Posterior

Translocation femur

A. Flexion, medial rotation, adduction


B. Flexion, lateral rotation, adduction
C. Flexion, lateral rotation, abduction

D. Extension, lateral rotation, abduction


E. Extension, medial rotation, abduction
3.

medial collateral

ligament
A.
B. posterior dislocation tibia
C. lock
D. passive abduction
E. lateral rotation full extension
F. abduction full extension
4.

40

3-4
nerve
A. Ulnar nerve
B. Radial nerve
C. Axillary Nerve
D. Median nerve

E. Musculocutaneous nerve
5.

hip joint
posterior

dislocation
A. Flexion + medial rotation + adduction
B. Flexion + lateral rotation + adduction
C. Flexion + medial rotation + abduction
D. Extension + lateral rotation + abduction
E. Extension + medial rotation + abduction
6.

A. tense vocal cord


B. relax vocal cord
C. adduct vocal cord
D. abduct vocal cord
E. close rami glostidis

7.

35

carpo-pedal

A. larynx
B. laryngeal nerve
C. thyroid gland

spasm

D. thymus
E. parathyroid gland
8.

bulb of penis nerve


A. Pudendal
B. Ilioinguinal
C. Genitofemoral

9.
10.

epidermis layer 5

Epidermis dermis cell junction
A. zona occludens
B. zona adherens
C. desmosome
D. gap junction
E. hemidesmosome

11.

(Blister) junction
A. Desmosome
B. Tight junction
C. Gap junction
D. Zona adheren
E. Zona occluta

12.



A. Sweat gland
B. Epidermal ridge
C. Stratum lucidum
D. Sebaceous gland
E. Stratum corneum

13.

Unilateral cleft lib


A. medial nasal and maxillar
B. median nasal and median nasal
C. maxillar and mandibular
D. median nasal and lateral nasal
E. lateral nasal and maxillar

Musculoskeletal System
Part Anatomy 2007
1.

20 posterior
dislocation

hip joint

A. flex, medial rotate, adduct


B. flex, medial rotate, adduct

C. flex, lateral rotate, abduct


D. extend, lateral rotate, abduct
E. extend, medial rotate, abduct
2. Bone Marrow biopsy

A. Tibial
B. Humerus
C. Radius
D. Femur
E. Iliac spine
3. 10 humorus

A. median nerve
B. radial nerve
C. Ulnar nerve

D. medial cord of branchial plexus


E. lateral cord of branchial plexus
4.

35

spasticity hyperreflexia

fasciculation

hyporeflexia

A. Peripheral nerve
B. Ventral root
C. Spinal cord
D. Brain stem
E. Alpha motor neuron
5.

A. ectoderm
B. mesoderm
C. endoderm

D. neural crest
E . neural tube
6. 15

A. epidermis
B. hair follicle
C. apocrine gland
D. sebaceous gland
E. arrector pilli muscle
7.

A. Large artery
B. Medium artery
C. Small artery
D. Capillary

E. Venule

(Anatomy)
Thoracic Organs System
Part Anatomy 2005
1. E. 5th intercostals space left at mid clavicular line

2. A.

Superior cervical ganglion sympathetic

ganglion Lesion Horner


syndrome

1)

2)

miosis : dilator pupilae muscle

ptosis :

3)

anhydrosis : vasodilation

Thoracic Organs System


Part Anatomy 2006
1. B. axillary



Axillary lymph node
2. D. respiratory bronchiole
respiratory bronchiole

Lower respiratory tract

Respiratory portion (
)


3. C. Pleura

Nerve

4. E. alveoli

particle 5
Mucociliary Escalator system

5. B. ICS 5
root of spine of scapula

rib

left lung

root of spine of scapula


rib

left lung

1.
upper lobe

2.

lower lobe

anterior view
Posterior view

6. E. Stratified squamous
Stratified squamous

Peudostratified

Columnar ciliated with goblet cell


7. E. Mitral valve Left ventricle
8. A. brachial artery
()
9.

Pharyngeal arch 4

10. distal part of the right dorsal aorta


11. D. 4
th

4 aortic arch
- arch of aorta
left common carotid left subclavian artery
- right subclavian artery,
right dorsal artery 7

th

intersegmental artery
- right dorsal artery dubble
aortic arch

Thoracic Organs System


Part Anatomy 2007
1C. 9
Rib 8


2. A. Alveoli
3. E. alveoli

Thoracic Organs System



- pneumonia lobe stethoscope

root of spine of scapula


rib

left lung

root of spine of scapula


rib

left lung

1. upper lobe

2. lower lobe
th

(Snell RS. Clinical anatomy by regions. 8 .2008:6872.)


Abdominal Organs System
Part Anatomy 2005
1. A. portal vein
Chronic liver disease : liver cirrhosis

fibrosis nodule

portal hypertension
palmar erythema ,spider nevi,
gynecomastia ,testicular atrophy, dilated suferficial
abdominal vein and ascites

portal hypertension
(Varices)
(rupture)

(malena)
(hematemesis)
2. phrenic nerve
(diaphragm)


phrenic nerve

Phrenic nerve

C3C5

referred pain

3. B. apoptosis
esophagus
1. esophageal

atresia esophagotracheal fistula

esophagus trachea
esophagotracheal septum
posterior
2. esophageal

stenosis

lumen esophagus

1/3 esophagus
rcanalization 8

Abdominal Organs System


Part Anatomy 2006
1. E. SER

enzyme P450 oxidase

drug

hydrolyze

2. C. esophageal reflux
lower
esophageal (gastroesophageal)
sphincter

esophageal
reflux
3. D. Gastric parietal cell

Peptic ulcer


(Gastric ulcer)
mucosal barrier
parietal cell
HCl

fundus body

pepsinogen pepsin

pepsin
mucosal barrier
peptic ulcer
4. B. Splenic vein

Gastric varices
vein thrombosis

Isolated splenic

left sides portal hypertension venous


hypertension

gastro splenic gastric

varices esophageal varices


5. B. Splenic vein
isolated splenic vein thrombosis
6. B. Middle colic artery
Descending colon left
colic artery (
inferior mesenteric artery ) middle colic artery (
superior mesenteric artery)

(anastomosis)
7. D. Tracheoesophageal septum
Tracheoesophageal septum membrane
forgut

-ventral : respiratory primordium


-dorsal : forgut esophagus
8. A. real agenesis
amniotic fluid abnormalities
Oligohydramnios < 0.5 L of amniotic fluid; associated with
bilateral renal agenesis or posterior urethral valves(in males)
and resultant inability to excrete urine. Can give rise to
Potters syndrome.
Polyhydramnios > 1.5 2 L of amniotic fluid; associated
with esophageal duodenal atresia, causing inability to swallow
amniotic fluid, and with ancephaly.
Abdominal Organs System
Part Anatomy 2007
1. E. external anal sphincter
oral canal

2. C. pancreas
3. D. Transversalis fascia
Anteolateral abdominal wall

1. skin
2.

superficial fascia

3. external abdominal oblique


4. internal abdominal obique
5. tranversus abdominis
6. transversalis fascia
7.extraperitoneal fat
8.parietal peritoneum
4. D. pluroperitoneal membrane

-septum transversum

-pleuroperitoneal membrane

2 Posterolateral septum
transversum

peritocardioperitonial

canals

pleuroperitoneal membrane

- -

diaphragmatic hernia
5. A. Urachal fistula
Keyword
urachal
fistula
Omphalocele
umbilical cord amnion
Gastrichisis

loop

amnion lateral
6. C. 1/3 ASIS

1/3

anterior

superior iliac spin (ASIS)


(Umbilicus) McBurneys point
7. C. Zona fasciculata

glucocorticoid cortisol

keyword
moon face, central obesity,
hyperpigmentation
Cushing

syndrome

Abdominal Organs System

Ureter

Brain and Nervous System


Part Anatomy 2005
1. C. right optic tract

A. left optic nerve

= total blindness

B. optic chiasma = bitemporal hemianopia


C. right optic tract

= left homonymous

hemianopia
D. left optic radiation = right homonymous
hemianopia
E. occipital lobe

= homonymous hemianopia

)
2. D. suprascapular artery
Suprascapular artery
trunk

thyrocervical

st

1 part

subclavian artery
anastomosis

dosal

subscapular artery
collateral

circulation

Suprascapular artery

scapular
scapula

ligation
axillary artery
3. A. upper outer quadrant
Sciatic

nerve

piriformis

gluteal

region

gluteal

gluteus


region

inferior gluteal nerve

maximus

superolateral
part of buttock upper
outer quadrant
4. D. nerve artery vein ring
femoral
triangle

lateral

N(nerve)

medial
A(artery)

V(vein)

5. D. deep peroneal nerve

Tibialis anterior

muscle
deep peroneal nerve

Tibialis anterior

muscle
6. A. basilar artery

lesion CN III

lesion CN

III

lesion

CN VII
nucleus CN VI,III,VII
pons

basilar artery

7. C.

8. C. corticospinal tract
Keyword
motor

Corticospinal tract


9.
A. ptosis of right eyelid: CN III (Levator
palpebrae superioris m. )

B. loss of corneal reflex:

corneal reflex

V ) CN V CN VII
C. engorgement of the retinol vein

Sinus

D. constrictive pupils response: intact CN III


E. loss of all extraocular movement of right
eye:

CN III

10. C. right anterior cerebral artery


right anterior cerebral artery

11. C. neuromuscular junction


myasthemia gravis
acetylcholine receptor

antibody

neuromuscular junction

12.
13. D. CN V
CN VII

Corneal reflex CN V

14. B. vestibular nucleus


Nystagmus


nystagmus
involuntary movement
15. D. umbilicus
Dermatome
1
T10

Brain and Nervous System


Part Anatomy 2006
1. A. Broca area
Motor area :
Angular gurus

: language and cognition

Hippocampus :

Wernicke area

2.
3. B. Basal ganglion
(resting tremor)
substantia nigra

basal ganglion
4. B. Midbrain
weber syndrome
1.

= light reflex

midbrain
2.hyperreflexia

= UMN lesion cerebral preduncle

midbrain
5. B. Neural Tube Defect

6. A. Hydrocephalus
Hydrocephalus
Obstruction
Blood clot , pus , tumor
fibrosis after meningial infection
Increase production
Tumor-producing ( choroid plexus papilloma )
Loss of brain tissue

post-infarction , brain atrophy


Brain and Nervous System
Part Anatomy 2007
1. D. middle cerebral artery left
63

keyword Right

hemiparesis() with motor


aphasia()
2. E. left middle cerebral artery

corticospinal tract

cecussation

medulla

lesion (left side)


Motor aphasia lesion Brocas area
( lateral side )
choice d e
d

medial side cerebral hemisphere


e
3.C. hippocampus
4. C. basal ganglia
Parkinson disease
- (rigidity)
- (Mask face)
- substantia nigra basal ganglia
5. C. hippocampus
Brain and Nervous System

1. CN IV
2. torticullis

sternocleidomastoid

muscle

(CN XI)
muscle
Musculoskeletal System
Part Anatomy 2005
1. B. deep dermis
Dermis

2. B. flexion
Flexor digitorum superficialis middle phalanges
5
Finger flexion
3. E. bone
2 ( Nerve
branch
Nerve )

4. B. quadriceps
Patellar ligament

5. D. peronius longus, peroneus brevis

lateral part

of leg
peronius longus, peroneus brevis

lateral

part of leg
6. A. axillary nerve
Axillary nerve surgical neck of
humerus deltoid teres minor

axillary nerve
overstretch
7. B. recurrent laryngeal nerve
8. A. Superficial inguinal lymph node
Superficial inguinal lymph node
Penis Scrotum Perianal

9. B. deep branch of ulnar nerve


deep branch of ulnar nerve Interossei
Palmar interossei m.
Dorsal interossei m.
10. D. gluteus maximus

= major hip extension muscle

11. E. interosseous membrane


Interosseous membrane
ulna radial
12. A. apoptosis
13. D. lateral side of arm
Axillary nerve upper lateral
brachial cutaneous nerve
deltoid
Musculoskeletal System
Part Anatomy 2006

( arm)

1. A. L2
L1
2. B. Flexion, lateral rotation, adduction
Head of femur
acetabulum
3.F. abduction full extension
4. D. Median nerve
Median nerve

median nerve

5. A. Flexion + medial rotation + adduction


6. C. adduct vocal cord

Recurrent laryngeal nerve

7. E. parathyroid gland
2+

Ca
Ca

2+

Parathyroid gland parathyroid thyroid


gland thyroid
gland

parathyroid gland

8. A. Pudendal
9.

1.Stratum corneum
2.Stratum lucidum
3. Stratum granulosum
4. Stratum sipinosum
5. Stratum basalae

10. E. hemidesmosome
A-D lateral cell
11. A. Desmosome

12.

C. Stratum lucidum

5
1.Stratum corneum
2.Stratum lucidum
3. Stratum granulosum
4. Stratum sipinosum
5. Stratum basalae

4 Stratum
lucidum
13. A. medial nasal and maxillar
Medial nasal prominence ,
philtrum
(Median portion)
Lateral nasal prominence

Maxillary prominence 2
lateral
Portion
Mandibular prominence ,
Medial nasal
prominence maxillary
Prominence
Musculoskeletal System

Part Anatomy 2007


1. A. flex, medial rotate, adduct
2. E. Iliac spine
3. B. radial nerve
Radial nerve radial groove

humorus radial nerve

action extensor
muscle

radial

nerve
4. E. Alpha motor neuron
Alpha motor neuron lower motor neuron
brain stem spinal
cord fasciculation hyporeflexia

Hyperreflexion upper
motor neuron
Fasciculation spasticity motor
neuron
UMN

LMN


E
5. D. neural crest

(melanocyte) f orm neural crest cell
6. C. apocrine gland
Apocrine gland

7. D. Capillary

2552

17

( .
)
2. 1 Barbinskis reflex

dorsiflexion

Frontal cortex Barbinskis reflex

dorsiflexion

upper motor neuron frontal lobe

motor upper
motor neuron

4. vertical diplopia
nerve

Left trochear nerve


9. bone marrow biopsy

hip bone (iliac crest)


(sternum

aspiration)

The types of biopsy


There are two main types of bone marrow test a bone marrow aspiration and a
bone marrow trephine biopsy.
marrow cells up into a syringe.

Aspiration means the doctor sucks some bone


A bone marrow trephine means that the doctor

removes a 1 or 2cm core of bone marrow in one piece.


You may have both of these tests done at the same time.

They give some of

the same information to the doctor, but there are differences.

The bone marrow

biopsy shows the doctor the structure of the bone marrow inside the bone,
whereas the aspiration takes just the bone marrow cells.
Having the test
Either test only takes a few minutes and you can have them done as an
outpatient.
bone.

The doctor usually takes the bone marrow sample from your hip

You can have a bone marrow aspiration from your breast bone (sternum)

but not a bone marrow biopsy.


First you have a local anaesthetic injection into the skin over the biopsy site
usually your hip bone to numb it. When this has worked, the doctor puts the
needle in through the skin, into the hip bone or sternum

http://www.cancerhelp.org.uk/help/default.asp?
page=3948
10. carpal tunnel syndrome

Palmar branch of median nerve



16. thalassemia

Spleen
white pulp red pulp




thalassemia

24.1/3 shaft of humerus bone

Extension wrist ( 1/2 shaft of humerus bone

) radial nerve
27.

Schwann cell


28.

Supraspinatus tendon

29.

ulnar nerve

ulna

33. carpal turnel


syndrome
nerve
palmar cutaneous branch of median nerve

superficial flexor retinaculum



34.

abductor pollicis longus

radial nerve

adductor pollicis ulnar nerve

abductor pollicis brevis median nerve

( ..
)

. 15 28 4

A. Apoptosis
B. Hydrolysis
C. Proteolysis
D. Aplasia
E. Necrosis
E

secretory phase ischemic phase


. gonad testis
A. Y chromosome
B. Sertoli cell
C. Leydig cell
D. Paramesonephric duct
E. Mesonephric duct
A H-Y antigen
. 2 testis scrotum
inguinal canal
testis
A. Iliac fossa
B. Pelvic cavity
C. Spermatic cord

D. Abdominal cavity
E. Processus vaginalis
B
(
.. )
.

A. Midgut volvulus
B. Esophageal atresia
C. Pyloric stenosis
D. Duodenal atresia
E. Annular pancreas
D. Duodenal atresia

(Duodenal atresia)

lumen

recanalization lumen

common bile duct


2-3

common bile duct


common bile duct
polyhydramnios
(amniotic fluid)

(amniotic sac)

.

A. hepatoduodenal ligament
B. hepatogastric ligament

A.

hepatoduodenal ligament

Cystic artery is located in which of the following


peritoneal structures?
A.Falciform Ligament
B.Gastrocolic Ligament
C.Gastrohepatic Ligament
D.Hepatoduodenal Ligament

E.Splenorenal Ligament
. duodenum

A. left gastric a.
B. right gastric a.
C. esophageal

a.

D. epigastric a.
E. gastroduodenal a.
E. gastroduodenal a.

. diaphragmatic herniation
( )
A. anterior
B. posterior lateral
C. lateral
D. central
B. posterior lateral

Congenital diaphragmatic hernia



pleuroperitoneal membrane




(left posterolateral site)



pulmonary hypoplasia

.
( )
A.pyloric stenosis
B.esophageal atresia
A.pyloric stenosis
Congenital

hypertrophic pyloric stenosis

pylorus
(pyloric muscle

hypertrophy) pyloric canal

(projectile

vomiting)
Duodenal atresia

.
stridor
A.trachea
B.bronchus
C.bronchiole
D.alveoli
A.trachea
. asthma wheezing

Rhonchi (Wheezing)
bronchi bronchioles

.
( )
A. gallbladder
B. pancreas
A. gallbladder referred pain gallbladder

( .
)
.
dermatome
A.

T12-L1

B.

L1-L2

C.

L2-L3

D.

L4-L5

E.

L5-S1

. 40 3

A. Detrusor muscle
B. Bladder trigone

C. Levator ani + pelvic fascia (Cilinically oriented anatomy


ed. 4 p. 345)

D. Pudendal nerve
E. Perineal fascia
. eversion
(
)
A. superial peroneal nerve
B. deep peroneal nerve
C. common peroneal nerve

C superficial
deep peroneal nerve
injury common peroneal
dorsiflex eversion injury
superficial peroneal eversion
injury deep peroneal dorsiflex
. dorsiflexion foot
)
A. deep peroneal nerve

B. superial peroneal nerve


A 31

nerve (

.

fundus lumbar lymph node
external iliac lymph node

round ligament of uterus superficial inguinal lymph


node
body external iliac lymph node
cervix internal iliac sacral lymph node
( . )
. folate Embryogenesis
A. Cell division
B. Gene mutation
C. DNA methylation
D. Histone acetylation
E. Neural tube folding
. Sympathetic trunk

A. Ectodermal cell
B. Mesodermal cell
C. Endodermal cell
D. Neuroectodermal cell
E.

Neural crest cell

.
A.ectoderm
B.splanchnic mesoderm

C.endoderm
D.neurocrest
.greater splanchnic
neural crest

nerve

(Physiology)

Part Physiology 2005


1.

respiratory rate 12/min tidal


volume 500 cc PaCO2 45 mmHg
PaCO2
A. cardiac output
B. lung compliance
C. dead space ventilation
D. functional residual capacity
E. FEV1

2.

36 hypertension hypokalemia
aldersteronism serum aldersterone

A. serum rennin
B. serum ACTH
C. serum prolactin
D. serum cortisol
E. urine Na

3.

25 HIV positive CD4+ count = 250 ml


protinase inhibitor nucleoside polymerase inhibitor
Hb 12.8 8.2 g/dl anemia

A. folate
B.
C. antibody RBC
D. RBC

E. RBC
4. poor skin turgor
Na 131 mEq/l ( 135-145 )

A. increased ADH
B. increased atrial natriuretic peptide
C. increased aldersterone
D. increased effective circulatory volume
E. ruine osmolarety < serum osmolarity
5.

1 rickets Ca P

1,25 ( OH )2D3. alkaline phosphatase parathyroid


hormone rickets
A. vitamin D intake
B. vitamin D absorption
C. activation of vitamin D

D. target cell receptor


E. catabolism of vitamin metabolite
6.

18


Serum:

Na
Cl

Urine:

140
85

Na

mEg/l

mEg/l
40

2.1

mEg/l

HCO3 35 mEg/l

mEg/l

140 mEg/l

A.
B.

anabolic steroid

C. aldosterone deficiency
D. diabetic ketoacidosis
E. anxiety with hyperventilation
7.

alcohol
coronary artery disease
A. HDL
B. LDL
C. triglyceride

D. thromboxane B2 platelet
E. contractile protein coronary artery
8.


prolactin hormone
A. insulin cortisol
B. thyroxine dopamine
C. estrogen progesterone
D. growth hormone oxytocin
E. HCG prostaglandin

9.

10 burn 2
nitrogen loss
A. insulin
B. cortisol
C. thyroxine
D. erythropoietin

E. parathyroid hormone
10.

cardiac output Fick O2 tension

O2 tension
venous blood
A. jugular
B. saphenous vein
C. foramen ovale
D. pulmonary artery
E. pulmonary vein
11.

A. ankyrin
B. myosin
C. tubulin
D. integrin

E. spectrin
12.

A. O2 affinity
B. O2 capacity
C. hematocrit
D. Pa O2
E. viscosity
13.

initial velocity (V) substrate

MicalisMenten kinetic enzyme


( 4 )

14.

A.

B.

C.

D.

control

70 NaCl 200 mEq 4

mineralocorticoid 4
urine NaCl
Day 1

30 mEq

Day 2

90 mEq

Day 3

180 mEq

Day 4

200 mEq

NaCl 1 l 1kg NaCl 150 mEq 4

A. 66 kg

B. 68 kg
C. 70 kg
D. 72 kg
E. 74 kg
15.

cardiac catheter 14

mmHg 26 mmHg
O2 saturation 55%
A. left atrium
B. right atrium
C. foramen ovale
D. ductus artheriosus
E. pulmonary artery
16.

4 2

oxidized

A. serum glucose
B. serum fatty acid

C. muscle glycogen
D. muscle triglyceride
E. muscle creatine phosphate
17.

35 6

gastrocnemius
A. conversion of fast fiber
B. decreased fiber
C. decreased myofibril
D. increased satellite cell
E. increased mitochondrial content
18.

30

BP 112/76 mmHg PR
88/min BP 80/60 mmHg PR 120/min

A. dopamine

B. isotonic saline
C. colloid solution
D. 5% dextrose in water
E. fresh frozen plasma
19.

40 20


, , severely restricted horizontal eye
movement and ataxia 2
vitamin
A. folate
B. retinol
C. thiamine
D. pyridoxine
E. cyanocobalamine

20.

50




A. amygdala
B. hippocampus
C. locus ceruleus
D. mammillary body
E. caudate nucleus
21.

platelet- derived growth factor cell

receptor signal transduction


A. DNA
B. GTP-coupled G protein
C. calcium
D. ion channel
E. thyroxine kinase
22.


A. -adrenergic blockage
B. -adrenergic blockage
C. muscarinic blockage
D. inhibition of acetyl cholinesterase
E. sympathetic denervation
23.

25 10

A. 2
B. 3
C. 1
D. 4
E. 6

24.

intracellular domain insulin receptor enzyme

activity
A. tyrosine kinase
B. adenylyl cyclase
C. phospholipase C
D. phosphodiesterase
E. phosphoprotien phosphatase
25.

30

BP 90/60 mmHg wheezing


vasoactive effect
A. IgE eosinophil neutrophil
B. IgA mast cell basophil
C. IgE mast cell basophil
D. IgA eosinophil neutrophil

E. active T-lymphocyte

26.

35
antivenom
100ml 2
RBC

A. hypersensitivity type I
B. hypersensitivity type II
C. hypersensitivity type III
D. hypersensitivity type IV
E. venum-antivenum reaction
27.

20 20 100ml
immediate response
A. hematocrit
B. heart rate

C. vagal activity
D. ADH
E. plasma hydrostatic pressure
28.

blood flow mean arterial pressure (MAP)


autoreguration blood flow

29.

severe atherosclerosis 10
Compliance

30.

pulse pressure

A.

B.

C.

D.

E.

50 aerobic


A. liver glucose
B. serum lactic acid
C. muscle protein
D. muscle glycogen
E. muscle phosphocreatine

31.

30 respiratory rate 20/min tidal volume 500 ml


anatomical dead space 200 ml respiratory rate
30/min tidal volume 350 ml anatomical dead space

A. PaCO2
B. PaCO2
C. PaCO2
D. pH
E. pH

32.

30

A. oxytocin
B. prolactin
C. estrogen

D. progesterone
E. HCG
33.

20 sex
physiology
A. sexual ambiguity
B. sexual immaturity
C. sexual perversion
D. sexual dysfunction
E. sexual inadequacy

34.

25 32

A.
B.
C. Hb 11
D.

E. urine protein 2+
35.

10 2

generalized edema, ascites positive


pH 6.0,
specific gravity 1.015, protein 4+, glucose negative

urine sediment
A. RBC cast
B. broad cast
C. oval fat body
D. urate crystal

E. numerous WBC
36.

30 chronicotitis
media tuning folk
A. Rene test right positive

B. Rene test right negative


C. Rene test left negative
D. Waber no lateralization
E. Waber lateralized to the left
37.

aterial blood gas pH 7.2

PaCO2 80 mmHg

Pa O2 50 mmHg
A. renal failure
B. metabolic acidosis
C. ventilatory failure
D. oxygenation failure
E. combine ventilatory and oxygenation failure
38.

18


PR 100/ min, BP 80/60 mmHg,abdominal

distention, decreased bowel sounds

A. splenic rupture
B. stomach perfotation
C. colonic perforation
D. renal pelvis rupture
E. hepatic subcapsular hemorrhage
39.

Specific gravity
1.035,Glucose negative, Protein negative

A. medullary blood flow


B. urea availability
C. solute free water
D. collecting tubule ADH
E. NaCl reabsorption thick ascending limb
40.

50 1



hormone

A. inhibin
B. estradiol
C. FSH & LH
D. progesterone
E. androstenedione
41.

25 2


A. night urine specimen
B. random urine specimen
C. midstream urine specimen
D. first- voiding morning urine specimen
E. 24 hours urine specimen

42.

30
postural hypotension poor skin
turgor

A. angiotensin II
B. ADH
C. ANP
D. aldosterone
E. sympathetic activity
43.

QRS complex

A. atrial depolarization
B. atrial repolarization
C. SA node depolarization
D. Ventricular depolarization
E. Ventricular repolarezation
44.

40 specific
gravity 1.015, sugar 2+ ,protein negative
blood sugar 100 mg/dl

A. afferent arteriole
B. glomerular basement membrane
C. proximal convoluted tubule
D. Henles loop
E. distal convoluted tubule

45.

HR 2 SV
TPR MAP ( mean arterial
pressure )
A.
B.
C.
D.
E.

46.

25
100dB 5

A.
B.
C.

D. +
E.
47.

60

A. cardiac index
B. GFR

C. renal blood flow


D. fasting blood sugar
E. maximum O2 uptake
48.


receptive relaxation

A. parietal chief cell


B. antral G cell

C. secretin
D. somatostatin
E. vagus nerve
49.

2 ( x y )
Y
X

A. isotonic
B. hypotonic
C. hypertonic
D. isosmotic

E. hyperosmotic
50.

( circadian varian )

melatonin MRI lesion

A. accessory optic nucleus


B. supraoptic nucleus
C. suprachiasmatic nucleus
D. lateral preoptic nucleus
E. preoptic nucleus
51.

35 shock BP 80/66
mmHg compensate
A. vagal activation
B. sympathetic activation
C. rennin-angiotensin-aldosterone system
D. higer center

52.

5
deleyed bone growth recombinant growth
hormone 3
A. insulin-like growth factor
B. activity PO4
C. parathyroid hormone
D. Ca

53.

2+

dehydration
sp.gr. 1.030 .protein negative, sugar 4+,
ketone-positive, Na 148 mEq/l, Cl 92 mEq/l HCO3 5
mEq/l, blood sugar 560 mg/dl
A. ADH
B. insulin
C. Hyperthyroidism
D. Cortisol

54.

18 cardiac catheterization

Blood pressure

O2 saturation (%)

(mmHg)
Right atrium

95 (75)

Right ventricle

95 (75)

Pulmonary artery

95 (75)

Left Atrium

90 (90)

Left ventricle

90 (90)

Aorta

90 (90)


A. mitral stenosis
B. aorticstenosis
C. artrial septal defect
D. tricuspid insufficiency

E. patent ductus arteriosus


55.

3
arterial blood gas
A. pH=7.5, pCO2= 55, HCO3 = 34
B. pH=7.4, pCO2 = 40, HCO3 = 24

Part Physiology 2006


1.

60
3




clinic

A. Glycine
B. Serotonin
C. Dopamine
D. Acetylcholine
E. Gamma aminobutyric acid

2.

10

A. c fiber
B. A beta fiber
C. transmittion cell in spinal cord
D. substance P nociceptor
E. enkepalin periaqueductal gray
3.

30
( myasthenia gravis)
A. Cholinesterase
B. Cholinesterase

C. ACh Receptor

4. 70 6



A. splanchnic arteriole
B. symphathetic activity

C. intercranial pressure

D. BP
E. coronary parasymphathetic activity
5.

100 10

6.


A. Ach
B. Ach receptor
C. AchE Synaptic cleft
D. Ach presynap
E.
turnover Ach receptor

7.


()
A. T4
B. T3
C. rT3
D. TSH

8.

40
PR 120/min Thyroid
gland PTU
PTU
A. Iodine transport
B. Thyroglobulin synthesis
C. TSH receptor synthesis
D. Thyroid peroxidase activity
E. Thyroglobulin binding activity

9.


Thyroid enlarge
,tachycardia
A. Osmotic diarrhea
B. Bacteria overgrowth
C. Increase GI motality
D. Inflammatory
E. Loose sphincter tension

10.

30 BP 150/100 Na+ = 150 K+ = 3.0

hormone
A. aldosterone
B. ACTH
C. ANP
D. ADH
E. Glucocorticoid
11.

30

negative MRI Pituitary


tumor hormone
A. Prolactin
B. Estrogen

C. Progesterone
D. LH
E. FSH
12.

45
fasting blood sugar 220

A. glycolysis
B. ketonegenesis
C. gluconeogenesis
D. lypolysis
E. glucogenolysis

13.

20
4-5 fasting blood
glucose = 250 mg/dl

A. Albumin
B. C-protein
C. Chylomicron
D. Ketone bodies
E. Triacylglyceride
14.

30


BP 150/100
FBS 180 hormone
A. GH
B. LH
C. TSH
D. ADH
E. ACTH

15.

45 BP PR


A. rennin

B. aldosterone

C. ADH

D. Epinephrine

E. ANP

16.

hormone
A. Leptin
B. Insulin
C. Thyroxin
D. Growth hormone
E. Insulin like growth factor

17.

BP
+

90/65 Na = 120, K = 5, HCO 3 = 20 , Fasting blood


glucose = 60 Hormone
A. vasopressin
B. thyroxin
C. adenocorticotrophin H.
18.

60

Hb 10 Hct 30 MCV 120

A. Gastrin
B. Secretion
C. Intrinsic factor

D. CCK
E. VIP
19.

terminal ilium 2/3 ilium valve


A. bile acid

B. transient time colon


C. transient time intestine
D. bile salt

20.

30 10


shooting star
RBC WBC
toxin



second messenger
A. cAMP
B. DAG
C. phospholipase
D. tyrosine lanase
E. IP3

21.

albumin edema

A. osmotic p

B. osmotic p
C. Increase permeability
D. increase hydrostatic pressure
22.

30
post hypertension pure
skin turgor Na+ 130 mEq

A. ADH
B. ANP

23.

pH = 7.55 HCO3 = 22 PCO2 = 25

24.

Serum osmolarity 320 mOsm/H2O


Urine osmolarity 40 mosm/H2O urine sp. Gr. = 1.002 Na+
K+
A. Na+
B.
proximal tubule
C. Na+ proximal tubule
D. Na+ distal tubule
E.
collecting

25.

60 4(multipartum)


A. Pelvic muscle contraction
B. Detrusor
C. bladder
D. micturatio center
26.



12

Yellow color,clear,specific gravity 1.035,


AlbuminGlucose negative
A. Urea renal medulla
B. Medulla blood flow

C. Solute-free water excretion


D. Collecting duct ADH

E. NaCl reabsorption loop of Henle



(Na -K

reabsorption Henles loop )


27.

60 6

A. splanchnic arteriole
B. sympathetic activity

C. cadiac pressure
D. BP

E. GFR

28.

3 HCG
hormone
A. HCG Corpus luteum
B. LH Corpus luteum
C. Estrogen Corpus luteum
D. progesterone Corpus luteum
E. Estrogen secretory endrometrium

29.

Hormone


A. LH
B. FSH
C. HCG
D. Progesterone
E. estrogen

30.

25 3
1
Hormone
A. Estrogen
B. Progesterone
C. FSH

D. LH
E. HCG
31.

1 ovulation

32.

Estrogen endometrium
A.
cAMP
B. GTP G-protein
C. gene expression
D. tyrosine kinase
E. adenylate cyclase

33.

50 6



1
Estrogen


A.
B.


C.
D. Estrogen
E.
34.

1 RR = 15 , Tidal volume = 500


, Anatomical dead space = 200 2 RR = 30

, Tidal volume = 300 , Anatomical dead space = 200


2

A. PaCO2
B. PaCO2
C. PaCO2
D. pH
E. pH
35.

pulmonary function Pa CO2 , Pa


O2 () 70 20
( 20 )
Pa O2

Pa CO2

A.

B.

C.

36.

D.

E.

25 O2
effector
A. Proton
B. CO2
C. 2, 3 - DPG
D. Erythropoietin
E. F 1-6

37.

Arterial blood gas


pH 7.55 , Pa CO2 25 mmHg , HCO3 = 22

A. hyperventilation
B. hypoventilation
+

C. H

D. HCO3
E. Titratable acid
38.

10 midbrain

pH 7.25 , Pa CO2 55 , Pa O2 60 , HCO3


= 25
A. diffusion defect
B. intrapulmonary shunt
C. alveolar hypoventilation
D. decrease perfusion
E. V/Q mismatch

39.

ARDS

A. compliance of lung
40.

10
dull percussion Breath sound

A. pleural effusion
B. pneumothorax
41.

A. cell atrophy
B. mucus

42.


Pulmonary defense mechanism
A. Cilia movement
B. Cough reflex
C. Secretory IgA
D. Alveolar fluid
E. Alveolar macrophage
F. Mucous blanket

43.

5 25 kg.

A. Chest wall muscle tone


B. Thoracic muscle tone
C. Upper respiratory muscle tone
D. Smooth muscle tone
E. Skeleton
F. Pressure upper respiratory trac

44.

45.


consolidation right lower lobe
hypoxemia
A. Hypoventilation

B. Anatomical shunt
C. Intra-alveolar shunt
D. Diffusion
E. V/Q mismatch
46.

2
Exudate Exudate

A. respiratory permeability
B. plasma oncotic pressure
C. plasma hydrostatic pressure
D. plasma oncotic pressure
E. lymphatic drainage visceral pleura
47.

50
Engorge jugular vein crepitation lower
lung field 2 2cm. costalmargin

pretibial edema

A. Oncotic Pressure
B. Oncotic Pressure
C. Hydrostatic Pressure
D. Hydrostatic Pressure
E. Vascular Permeability
48.

2 32
RDS
(Respiratory Distress syndrome)

A. surfactant
49.

murmur S1&S2 intercostal


space5 midclavicular line

A. aortic stenosis
B. aortic regurgitation

C. mitral stenosis
D. mitral regurgitation
E. tricuspid regurgitation
50.


A. ASD
B. VSD
C. tetralogy of fallot
D. PDA
E. Coarctation of Aorta
51.

60 aortic perfusion pressure 80


atherosclerosis aortic perfusion
pressure

52.

60 coronary thrombosis
dysfunction endothelial cell

A. Endothelium
B. NO
C. Angiotensin
D. Epinephrine
E. PG
53.

20 BP 90/50 PR 120

A. Cerebral
B. Coronary
C. Cutaneous muscle
D. skeleton muscle

54.


enlarge jugular vein crepitation lower lung
area 2 2 cm

pretibial myxedema

A. oncotic pressure
B. oncotic pressure
C. hydrostatic pressure
D. hydrostatic pressure
E. vascular permeability
55.

20

A. Splanchnic artery
B. Sympathetic activity
C. Increase Intracranial pressure
D. Blood pressure
E. Cardiac parasympathetic
56.

20 BP 160/100 BP 110/80

57.

36


A. stroke volume

B. venous return ( IVC )


C. respiratory rate
D. Heart rate
E.
F. Tidal volume
G. FRC
58.

55 BP 140/85 20 BP
115/80 BP
A. HR
B. SV
C. Aortic compliance
D. Vascular ..

59.

50 coronary perfusion pressure 80 mmHg


50% () coronary
perfusion pressure
A. 40
B. 160
C. 640
D. 1280
E.

60.

20 30 BP 70/50

A. Heart rate
B. Splanchnic blood flow
C. Cutaneous blood flow
D. Skeleton M.
E. Carotic baroreceptor activity

61.

EKG

A. P wave
B. PR interval
C. QRS complex
D. T wave
62.

Carotid Aortic Chemoreceptor

A. Low O2
B. High CO2
C. Severe anemia
D. CHF
E. CO poisoning
63.

Hemoglobin
O2 tissue effector
A. 2,3-DPG

B. CO2
C. Temperature
D. Fluid
E. Surface tension of alveoli
64.

5 PP BP poor skin
turgor , eye ball () pH 7.2 HCO3- 10 PCO2
60 PO2 100
A. metabolic acidosis
B. metabolic alkalosis
C. respiratory acidosis
D. respiratory alkalosis
E. combine metabolic and respiratory acidosis

65.

1
murmur intercostals
space 2
A. aortic stenosis

B. pulmonic stenosis
C. ductus arteriosus
66.

30
cardiac murmur apex
Rheumatoid heart disease
A. Aortic stenosis
B. Aortic insufficiency
C. Mitral stenosis
D. Pulmonic stenosis
E. Tricuspid stenosis

67.

50 30 10
30
45
BP 160/110 acute myocardial infarction
non modifiable risk factor coronary artery
disease

A. family history
B. hypertension
C. hyperlipidemia
D. Cigarrete smoking
E. DM
68.

26

ultrasound
Part Physiology 2007
1.

10 lower esophagus
sphinter manometry LES pressure

A.
B.
C.
D.

E.
2.

60 5
6 postural hypotension
poor skin turgor

A. aldosterone
B. plasma angiotensin II
C. ANP
D. Sympathetic activity
E. Efferent arteriole resistant
3.

2 10 2
serum CO2 = 12 mmol/L urine pH= 5.0

compensation nephron
A. Glomerulus
B. Proximal tubule

C. Loop of henle
D. Distal tubule
E. Collecting duct
4.

35 16 .

urine sp.gr 1.025 hormone

A. insulin
B. Oxytocin
C. Vasopressin
D. Glucocorticoid
E. ANP
5.

FEV1 (force expiration in 1 second)

FEV1(liter) = (1.052 x height) - (0.244 x age) -0.561

A. 1 ,FEV1 0.244 liter


B. 0.244 FEV1 1 liter
C. 1 FEV1 0.244 liter
D. 0.244 FEV1 1 liter
6.

25 2



A.
B.
C.
D.
E.

7.


A. FEV1/FRC
B. Elascity

C. Compliance
E. Restrictive fibers
8.


A. micelle
B. chylomicron
C. cholesterol
D. fatty acid
E. monoglyceride

9.

30 tidal volume 500 mL,anatomical dead


space 200 mL, RR15 RR 30,tidal volume 350
anatomical dead space

A. Paco2

B. Paco2
C. pH

D. pH
E. paco2
10.

70 40

2 x-ray emphysema
A. airway resistance
B. lung compliance
C. intrapleural P
D. FRV
E. FEV1
11.

bile salt

A. bilirubin
B. lecithin
C. fatty acid
D. triglyceride

E. cholesterol
12.

200 m

A. mitochondria
B. glycolytic enzyme
C. muscle blood flow
D. Fat composition
13.

35


rennin-angiotensin system BP
A. GFR
B. renal blood flow
C. Na reabsorption
D. ACE activity

14.

17
A.
Baroreceptor
B.
Low Pressure receptor
C. rennin
D.
chemoreceptor
E. Hydrostatic pressure

15.


A. Na+
B. K+
C. Renin
D. Aldosterone
E. ANP

15.

2 serum CO2 Urea pH 5.0 nephron

compensate

A. Glomerulus
B. Proximal tubule
C. Loop of Henle
D. Distal tubule
E. Collecting duct
16.

20 5-6


A. Hypoxia
B. Hyperoxia
C. Hypocapnia
D. Hypercapnia
E. Pneumothorax

17.

25 HR

66 RR 20 Pressure 210/100 CVS

A. Blood volumn

B. Preload
C. Afterload
D. Sym
E. Peripheral Vasoconstriction
18.

50 Ascites furosemide 1


Na 130 K 2.5 pH 7.58 PCO2 50
acid-base
A. uncompensated respiratory alkalosis
B. compensated respiratory alkalosis
C. uncompensated metabolic acidosis
D. compensated metabolic acidosis
E. mixed respiratory and metabolic alkalosis

19.

15 ER

10

A. Ach
B. NE
C. AchE
D. muscarinic Receptor
E. adrenergic receptor
20.

50 . New York City


hormone
A. anterior Pituitary
B. posterior pituitary
C. pineal gland
D. hypothalamus
E. adrenal gland

21.

30

thymus

A. immediate hypersensitivity
B. delay type hypersensitivity
C. immune complex hypersensitivity
D. Ab-cytotoxic hypersensitivity
E. Complement activation
22.

65 Alzheimer



A. nerve growth hormone
B. intracellular ca2+
C. glycogenolysis
D. gluconeogenesis
E. ......

23.

25

Hormone
A. Prolactin
B. Oxytocin
C. Vasopressin
D. Catecholamine
E. Corticosteroid
24.

35


A. FSH
B. testosterone
C. estrogen
D. Progesterone
E. Estrogen

25.

35 16 .

urine specific gravity = 1.025 hormone

A. insulin
B. oxytocin
C. vasopressin
D. glucocorticoid
E. atrial natriuretic peptide
26.

35 PR

110/min BP 80/50 24 . BP 70/50-80/60

A. Venous return
B. Central vasoconstriction
C. Central venous pressure
D. End diastolic pressure

E. Total peripheral resistance


26.

vagotomy
vagotomy parietal cell
A. Cl- channel
B. H+ channel
C. nicotinic receptor
D. muscarinic receptor
E. histamine receptor
27.

30



A.
B.
C.
D.

E.
28.

ECG

A. PR interval
B. RR interval
C. P wave
D. P wave
E. QRS complex
29.

35
BP
rennin angiotensin system
BP
A. Vasodilation
B. renal blood flow
C. sodium reabsorption

D. GFR
E. angiotensin
30.

20

A. Tidal volume
B. RR
C. Minute ventilation
D. Dead space ventilation
E. Inspire reserve volume(IRV)

31.

65 COPD+bronchietasis

A. FEV1/FVC
B. Airways resistance
C. Residual volumn

D. Lung compliance
E. Lung capacity
32.

10 2 Hct 25% Hb 7
WBC 5000 ( N 65% L 35% M 5% ) Plt 250000
Heinz body
A. Precipitate Ribosome
B. Precipitate Hemoglobin
C. Iron inclusion
D. Nucleus
E. Aggregation ER

33.

3 1
hormone
A. estrogen
B. progesterone
C. FSH

D. LH
E. HCG
34

.
RR (), BP 200/100 BP
A. Preload
B. afterload
C. Sympathetictone
D. vasoconstriction of peripheral vessels

35.

800

A. J-receptor

B. Irritant receptor
C. peripheral chemoreceptor
D. central chemoreceptor

36.

30
8 urine pregnancy test

A. prolactin
B. estradiol
C. progesterone
D. leutinizing hormone
E. follicular stimulating hormone

37.

A. ADP
B. Prostacycline
C. Activation COX enz
D. Activation thrombomodulin
E. Activation glycoprotein VWF

38.

20
(unconscious) 8 /
arterial blood gas : pH = 7.07 paO2 = 32paCO2 = 88
HCO=24

A. oxygen
B. minute ventilation
C. diffusing capacity
D. resistance
E. intrapulmonary shunt

39.

100 ml/min
50%
A. 6.25
B. 25
C. 50

40.

8 Ca 7.5 mg/dl,phosphate
2.8mg/dl, alkaline phosphatase 300microI/L

A. Ca
B. Vit D
C. Vit C
D. Calcitonin
E. PTH

41.

40

A. Ca

B. phosphate
C. Creatinine phosphate
D. ADP
E. ATP

42.

20 1 unit
BP 80/50 , PR 50/min

A. cardiogenic shock
B. hypovolumic shock
C. vasovagal attack
D. orthostatic hypotention
E. stroke adams
43.

5000 ml
urine graviy 1.015

A. .
B. reabsorb sugar
C. ...
D. reabsorb water collecting duct
E. reabsorb Na PCT

44.

50 2 1



hormone GI
A. Motilin
B. Gastrin
C. Secretin
D. CCK
E. VIP

45.

30 3
Hct 23% WBC
3,200 Neu 30% Lymphocyte 70%Platelet 50,000 MCV 89

A. RBC
B. RBC
C. Ab RBC
D.

E.
46.

35 peptic ulcer

Hb 8 g/dL Hct 30
Reticulocyte 8% Platelet 250,000/mm3 MCV 86 fL

A. autoimmune
B. Vit B12
C. Hb
D. enzyme
E. Post hemorrhage
47.

20
urlicaria rash generalized lymphadenopathy ,arthritis

A. Serum complement
B. Absolute white blood cell
C. Serum IgG

D. Lymphocyte
E. ESR
48.

25 alcohol
Gastroscope Gastric
mucosa Gastric biopsy Neutrophil

A. Autoimmune
B. Direct mucosal toxicity
C. Stimulate acid secretion
D. Alcohol-induced Thiamine deficiency
E. Stimulate growth of H.pylori

49.

60 2
rigid

A.

DA

Cholinergic

50.

B.

C.

D.

25 5 PCR
20
higher cortical function
A.
B.
C.
D.
E.

51.

45 Cardiac
murmur pulmonaric valve Hct 20%
Cardiac murmur

A.
B.
C.
D.
E.
52.

50 ...
TSH TRH stimulation test TSH

A. Thyroid gland
B. Thalamus
C. Hypothalamus
D. Anterior pituitary gland
E. Posterior pituitary gland

53.

Blood pressure 110/70 100/70

A. venous pooling

B. stroke volume
54.

20


A. Ca2+
B. K+
C. HCO3-
D. Na+
E. Mg

55.

30 24 ultrasonography
anencephaly
A. insulin
B. Thyroid hormone
C. TSH
D. GH
E. HCG

56.

DM Insulin-dependent Insulin
15 Insulin 10

A. insulin
B. .........
C. sensitivity insulin
D. absorb insulin

57.

20 midbrain 10 /min pH7.25


PaCO2 =55 HCO3 = 20
A. alveolar hypoventilation
B. Q-V mismatch



- p53 suppress
- Barrett esophagus adaptation
- insulin

- MI EKG
- EKG
- Hormone
- mediator

Physiology
Part Physiology 2005
1. c. dead space ventilation
RR tidal volume


dead space

Dead space
1.anatomical dead space: conducting zone trachea,
bronchus,bronchiole

2.alveolar dead space: alveolar

3.physiologic dead space : dead space


anatomical dead space + alveolar dead space
b. lung compliance(V/P) lung compliance
tidal volume tidal volume
b.
d. functional residual capacity

e. FEV1 : force expiratory volume in 1 second


2.

A. serum renin (activity)


aldosteronism primary secondary


aldosterone RAAS renin angiotensin


aldosterone LAB
plasma renin activity plasma aldosterone level

primary secondary primary renin


secondary renin ACTH
Aldosterone
3. D. RBC
AIDS 3
1) Nucleoside reverse transcriptase inhibitor (NRTIs)
Zidovudine(AZT) , Stavudine(d4T) , Lamivudine(3Tc)
reverse transcriptase enz. side
effect RBC
anemia granulocytopenia

, peripheral

neuropathy pancreatitis
2) Non-nucleoside reverse transcriptase inhibitor
(NNRTIs) Nevirapine , Delavirdine
reverse transcriptase enz.

HIV-1 side effect

rash
3) protenase inhibitor Ritonavir , Saquinavir ,
Indinavir protease enz. of HIV-1

enz. viral protein precursor


functional protein maturation of viral

side effect

asthenia, malaise, diarrhoea, nausea and vomiting,


abdominal pain, dizziness, insomnia, sweating , taste
abonormality
4.

A. increased ADH
: poor skin turgor
ECF volume extraction

Posm
+
ADH release
(serum

~ 285 295 mOsm/kgH2O)

urine

hypertonicity

urine

osm >

serum osm

5.

D. target cell receptor


rickets Ca P 1,25 ( OH )

D3, alkaline phosphatase parathyroid hormone


Impaired action of 1,25-dihydroxycholecalciferol on target

tissues

certain anticonvulsants
1,25-dihydroxycholecalciferol receptor defects
Uremia

Vit D metabolism
PTH vitamin D
target cell
receptor
6.

A.
Hypokalemia
Hypochloremia

Flurosemide
+

K plasma
E.
B.
C.

anabolic steroid

aldosterone deficiency

aldosterone K cell,

K plasma
D.

diabetic ketoacidosis

metabolic acidosis; HCO3 ,


+

PaCO2 insulin, insulin K cell; K


E.

anxiety with hyperventilation anxiety loss


-

CO2, PaCO2, HCO3 respiratory alkalosis


7.
8.

C. estrogen progesterone
estrogen, progesterone

hormeone
9.

B. cortisol
Cortisol stress hormone

Stress cortisol
cortisol

metabolism catabolism
Nitrogen Nitrogen loss

EPO and PTH


Insulin

10. D. Pulmonary artery


Fick C.O.
C.O.

O2 consumption
A-V O2 Difference

A-V O2 Difference [O2]


Pulmonary vein Pulmonary artery [O2]
Pulmonary vein (O2 )
Radial artery

[O2] [O2]

Pulmonary artery ( O2 Venous blood


) Pulmonary
artery

11.

E. spectrin
(peripheral protein)

spectrin, ankyrin, protein 4.1, actin and adducing


spectrin
polypeptide 2 alpha and beta chains

2
Spectrin
actin,
ankyrin

adducin

(circulatory shear force)

2 (biconcave shape)

lipid
2
12.

D. Pa O2

d. PaO2 HbF affinity O2


O2 () O2

a. O2 affinity fetus HbF


b. O2 capacity fetus HbF O2
HbA1 ()
c. Hct fetus fetus PaO2 Hb
O2 affinity fetus chronic hypoxemia
erythropoietin RBC
Hct
e. viscosity fetus Hct
fetal blood O2(Pa O2) = 25-30 mmHg Pa
CO2 = 45-50 mmHg adult
O2
cardiac output
O2 Hb

Hb(hematocrit)
O2

(viscosity)
20
15 g/dL
18 g/dL
RBC Hb HbF O2 (O2affinity)
RBC HbA Hb adult HbA
2,3-diphosphoglycerate HbF 2,3diphosphoglycerate Hb O2
RBC

2,3-diphosphoglycerate O2
RBC 2,3-diphosphoglycerate
13.

B. 2

Enzyme (Vmax)
Enzyme 1
2 (Vmax Enzyme)

Enzyme
Enzyme Substrate
Substrate Product
2 Vmax/2 2
Normal
14.

15. E. Pulmonary artery


Pulmonary
artery ( 24/9 mmHg)
** systolic diastolic
pressure ( = 120/80
mmHg) Pulmonary
circulation Right atrium , Right ventricle ,
Pulmonary artery , Pulmonary capillary Pulmonary vein
1. diastolic pressure
0 secondary pressure wave
aorta large artery (diastolic pressure

secondary pressure wave) Right atrium , Right


ventricle Foramen ovale

2. O2 saturation 55% Pulmonary


vein Ductus arteriosus ( ductus arteriosus
Left atrium O2 )
Pulmonary artery
16.

B. serum fatty acid


1-3

glucagon
free fatty acid
glucose
17. C. decreased myofibril

myofibril

disuse muscle atrophy


18. B. Isotonic saline


(Dehydrate) Electrolyte Venous
return C.O. BP drop

Isotonic Saline
electrolyte
19.

C. thiamine

Alcohol Ethanol

Thiamine Alcohol
Thiamine Thiamine
Thiamine Thiamine
diphosphate (ThDP) Coenzyme Enzyme
metabolism Pyruvate
Dehydrogenase (PDH)
Acetylcholine , Neurotransmitter myelin sheat
Ataxia

20.

D. mammillary body

Korsakoffs syndrome amnestic


confabulatory syndrome 1


(Confabulation)
mammillary bodies
dorsomedial thalamic nuclei
mammillary body
papez circuit recent memory
remote memory

cerebral cortex
recent remote memory


Limbic system
(medial margin; margin = limbus)
cerebral hemisphere limbic system

limbic lobe, fornicate
gyrus, visceral brain, vital brain emotional brain

Limbic System
. Telencephalon
. Hippocampal formation
.. Hippocampus (Ammons horn, cornu
ammonis)
.. Dentate gyrus (fascia dentate)
.. Subiculum
. Parahippocampal gyrus
. Cingulate gyrus
. Retrosplenial cortex (isthmus)
. Septal area
. Basolateral amygdaloid nucleus
. Diencephalon
. Mammillary body

. Anterior thalamic nucleus


. Habenular nucleus

http://www.colorado.edu/intphys/Class/IPHY3730/image/figure58.jpg

http://www.med.wayne.edu/diagRadiology/Limbic%20System
%20RSNA%202004/Overview.html
HIPPOCAMPAL FORMATION
Hippocampal formation temporal lobe
inferior horn lateral ventricle
Hippocampal formation archicortex

. Hippocampus: Hippocampus
temporal lobe
hippocampus inferior horn

lateral ventricle hippocampus archicortex


. Molecular layer
. Pyramidal cell layer
pyramidal fimbria-fornix
. Polymorphic cell layer
. Dentate gyrus:
hippocampus parahippocampal gyrus dentate
gyrus archicortex hippocampus
granule Dentate gyrus
hippocampus
. Subiculum: parahippocampal gyrus
hippocampus

Dissection of the left cerebral hemisphere demonstrating


structures of the limbic system. The body of the corpus
callosum has been divided sagitally. The frontal, temporal and
occipital lobes have been sectioned horizontally and their
superior parts removed. The left lentiform complex and
thalamus have been removed and the floor of the inferior
horn of the lateral ventricle opened.

http://www.neurosurvival.ca/ComputerAssistedLearning/neuroana
tomy/neuroanatomy_epilepsy/neuroanatomy_epilepsy_data.htm
Hippocampus

. entorhinal cortex olfactory association


cortex hippocampus perforant pathway

. septal area
fornix hippocampus
. parahippocampal gyrus temporal lobe
association areas parietal, occipital
temporal lobes

fimbria fornix:
Axon pyramidal hippocampus
ALVEUS floor inferior horn lateral ventricle
alveus FIMBRIA
splenium corpus callosum
FORNIX Fornix

corpus callosum anterior commissure


anterior column posterior column

http://spinwarp.ucsd.edu/NeuroWeb/Text/br-800epi.htm

Anterior column fornix septal area, preoptic


area, anterior ventromedial hypothalamic nuclei
hypothalamus
reticular formation
hypothalamotegmental tract dorsal longitudinal
fasciculus

Posterior column fornix


hypothalamus mammillary body

anterior thalamic
nucleus mammillothalamic tract (bundle of Vicq dazyr)
anterior thalamic nucleus cingulated gyrus
thalamocingular fibers anterior thalamic radiations
cingulated gyrus cingulum
entorhinal area entorhinal area
hippocampus
Papez (Papez loop)

Papez Circuit
http://commons.wikimedia.org/wiki/File:Papez_Circuit.jpg

http://www.indianjpsychiatry.org/articles/2007/49/2/images/Indian
JPsychiatry_2007_49_2_132_33264_3.jpg
mammillary body
mammillotegmental tract
CINGULATE GYRUS
Cingulate gyrus cerebral hemisphere
corpus callosum

archicortex neocortex mesocertex Cingulate gyrus


Thalamocingular thalamocortical fibers anterior
thalamic nucleus

. Corticostriate fibers caudate nucleus
putamen
. Corticothalamic fibers anterior dorsomedial
thalamic nuclei

. Corticohypothalamic fibers anterior, lateral


posterior hypothalamic nuclei
Papez circuit Limbic system

21.

E. thyroxine kinase
Critical molecular determinant of endothelial cell biology :

angiogenic endothelium expresses a number of receptor not


found on resting epithelium. These include receptor thyrosine
kinases (RTK) and integrins that bind to extracellular matrix
and mediate endothelial cells adhesion, migration and
invasion. Endothelial cells (ECs) also express RTK (i.e., the
FGF and PDGF receptors) that are found on many other cell
types. Critical function mediate by activated RTK include
proliferation, migration and enhanced survival of endothelial
cells, as well as regulation of the recruitment of perivascular
cells and bloodborne circulating endothelial precursors and
hematopoietic stem cells to the tumor. Intracellular signaling

via EC-specific RTK utilizes molecular pathway that may be


targets for future antiangiogenic therapies.
22.

E. Sympathetic denervation

.
23.

A. 2
2 hCG


24.

A. tyrosine kinase
The insulin receptor is a dimer with two different types

of subunits. It spans the membrane.


The -subunits are tyrosine kinases. When insulin binds
the subunits phosphorylase themselves at tyrosine residues.
The subunits also phosphorylate other protein kinases at
tyrosine residues. These kinase produce the cellular
responses reversal of glucagon stimulated
phosphorylation, phosphorylation of proteins, induction and

repression of specific genes, stimulation of general protein


synthesis and stimulation of glucose transport.
second messenger
cAMP

Ca -DAG
2+

Tyrosine kinase

Increase cAMP
ACTH

Acetylcholine

-adrenergic

1-adrenergic

catecholamines

catecholamines

Insulin

Calctonin

Angiotensin

Insulin-like growth

Glucagon

Cholecystokinin

LH

Gastrin

FSH

Oxytosin

TSH

ADH(V1receptor)

HCG

TRH

Parathyroid

Epidermal growth
factor

factor I
Fibroblast growth
factor I
Platelet-derived
growth factor

hormone

GnRH

Secretin
Decrease cAMP
Acetylcholine
2-adrenergic
catecholamines
Dopamine
Opiate peptides
Somatostatin

Insulin receptor family insulin
like growth factor 1
insulin IGF-I
metabolism (growth promoting effect)
IGF-1 polypeptide Insulin


Growth
hormone Androgen
insulin IGF-1 Glycoprotein
tyrosine specific protein kinase
4 (tetramer) 2
disulfide linked heterodimers ( )2
Cystein
membrain

membrane
phosphorylation Tyrosine kinase
tyrosine kinase

SH domain (Src homology domain)


SH2 domain SH3 domain
transcription factor
leucine zipper
heterodimer c-fos/c-jun AP-1 DNA response
element gene

..

25. C. IgE mast cell basophil


BP wheezing
hypersensitivity type 1
hypersensitivity type 1 allergen
IgE mast cell basophil mast cell
basophil histamine vasodilate
skin (urticaria and eczema), eyes
(conjunctivitis), nasopharynx (rhinorrhea, rhinitis),
bronchopulmonary tissues (asthma) and gastrointestinal
tract (gastroenteritis) BP ( vasodilate )
26. C. hypersensitivity type III
(-bungaro toxin ) block

nicotinic

receptor postsynaptic NMJ Ach ptosis


diaphragm paralysis ( )
serum ( antivenom )

excess Ag

Ab immune complex
complement
chemetatic factor neutrophil
, () ,
nephritis ( RBC )

Serum sickness
27. B. Heart rate

(hemorrhage)

Cardiac
output
Stretch on carotid sinus
baroreceptors
Firing rate of
carotid sinus nerve
Parasymp
Symp activity to heart
Heart
activity to heart
ana
blood rate
vessels
Hea
Contractility
rt rate
Mean arterial
Arterioles constrict
pressure
TPR

Venous return
28. D. Sigmoid curve

MAP 2
(Blood

flow )
(Central control)

central control
blood flow sigmoid
curve
29. B. Compliance

pulse pressure

severe atherosclerosis
Compliance Systolic pressure


secondary
pressure wave Diastolic pressure Pulse
pressure
30. E. muscle phosphocreatine
ATP
creatine

phosphate

Time Course of Contributions from Different Energy


Sources
Taken from Gleim, Anaerobic Testing and Evaluation,
Med Exerc Nutr Health 1993;2:27-35

31.

A. PaCO2

alveolar ventilation(VA) = RR x (VT-VD)
RR 20 / VT 500 ml VD 200 ml : VA = 20(500-

200) = 6000 ml
RR 30 / VT 350 ml VD 200 ml : VA = 30(350200) = 4500 ml

RR VT VA

PaO2
32.

PaCO2

A. oxytocin

33. .
34
35.

A. RBC cast
pH 6

PCT H

specific gravity 1.015 concentrate


urine ( concentrate urine renal
failure)
glucose -ve

pct glucose

protein 4+ glomerulus
RBC, WBC cast
RBC cast

cast : are formed in tubular lumen in kidney; they are


composed of a protein matrix(Tamm-Horsfall protein) within
which are entrapped cell, debris or protein leaking through
the glomeruli; Their presence proves a renal origin of
rd

disease.[ Rapid review pathology 3 edition, Edward F.


Goljan]
urate crystal:
oval fat body: renal tubular cell with lipid
numerous WBC(pyuria): urinary tract infection (
)
36.

B. Rene test right negative


Webers test Y



hair cell bone conduction
conductive hearing loss
(lateralizes to the affected ear)

sensory-neural hearing loss/deafness


/
Rinnes test Y
mastoid process bone conduction
air
conduction air conduction
bone conduction amplification

air conduction bone
conduction (AC BC positive)

air conduction bone


conduction (BC AC negative )
conductive hearing loss/deafness

chronic

otitis media BC AC negative


37.

C. ventilatory failure
pH 7.2 (acid) , PaCO2, PaO2

a.

renal failure reabsorp HCO3

PaCO2, PaO2
b.

metabolic acidosis HCO3


+
H CO2 PaCO2, PaO2
c.ventilatory failure PaCO2, PaO2
pH
CO2
d.oxygenation failure: CO2 exchange
O2 exchange PaCO2
PaO2
38.

C. colonic

perforation

bowel sounds
39.

c.

solute free water


a. medullary blood flow medullary blood
flow interstitial medulla

specific gravity (urine


specific gravity 1.001-1.035)
b. urea availability urea availability
concentrate urine urine specific gravity
1.035 higher normal urea availability

d. collecting tubule ADH : ADH


ADH

e. NaCl reabsorption thick ascending limb



40.

C. FSH & LH
A. inhibin
B. estradiol
D. progesterone
E. androstenedione

41.

D. first voiding morning urine specimen

42.

C. ANP
blood volume VR Stretched atrium

ANP
Intravascular volume
ANP
RAAS renin ,AII ,aldosterone ADH
AII Hypothalamus
Sympathetic activity AII

Vasomotor center
43. D. Ventricular depolarization
EKG
44.

C. proximal convoluted tubule


glomerulus

PCT
2Na-glucose cotransporter ( secondary active
transport Na-K ATPase Na

gradient Na cell lumen


Na lumen gradient cell
glucose cell

glucose electrochemical gradient


capillary GLUT1,GLUT2)

blood glucose

glucose blood

glucose 200 mg/dL glucose reab


transporter glucose
glucosuria

Textbook of Medical Physiology, 11th edition (Guyton,


2006)

45. C.

MAP =

SV x HR x TPR

MAP=

(1/2SV) x (2HR) x TPR

MAP
46. C
Bern physiology
1oo hair
cell cochlear

47. A. Cardiac index


acidosis
O2
anaerobic metabolism
Hyperventilation

O2 Cardiac index (
Maximum O2 uptake )
**Cardiac index (CI)

BSA = Body Surface Area


48.

E. vagus nerve

: receptive relaxation vagovagal reflex (


afferent efferent pathway vagus nerve)

food bolus
bolus lower esophagus esophagus

mechanoreceoptor vagus
nerve CNSCNS lower
esophageal sphincter stomach relax
a. p
arietal cell : parietal cell HCl
intrinsic factor Ach(parasymp:vagus nerve),
Histamine, gastrin

b. a
ntral G cell: G cell gastrin
small peptide, amino acid, distention of stomach and vagal
+

stimulation H secretion growth of


gastric mucosa
c. secretin : secretin S cell duodenum
+

H fatty acid duodenum


-

pancreatic HCO3 secretion , biliary HCO3 secretion ,


+

gastric H secretion inhibit trophic effect of gastrin on


gastric mucosa
d. somatostatin : endocrine cell
gastrointestinal

mucosa pH

H
( physiology 2
49.

nd

edition, Linda constanzo)

B. hypotonic
2

y
2

1. x Hypertonic
leak x
2. y Hypotonic
leak y
50.

C.suprachiasmatic nucleus

hypothalamus

Function

Afterents From

Integrating Areas

Temperature

Temperature receptors in

Anterior hypothalamus,

regulation

the skin,deep

response to heat;

tissues,spinal

posterior hypothalamus,

cord,hypothalamus,and

response to cold

other parts of the brain


Neuroendocrine
control of.

Limbic areas concerned

Dorsal and posterior

Catecholamines

wih emotion

hypothalamus

Vasopressin

Osmoreceptors, volume

Supraoptic and

receptors, others

paraventricular nuclei

Touch receptors in

Supraoptic and

breast, uterus, genitalia

paraventricular nuclei

Temperature receptors in

Paraventricular nuclei

infants,perhaps others

and neighboring areas

Oxytocin

Thyroid
stimulating hormone
(thyrotropin,TSH) via
TRH
Adrenocorticotropi
c hormone(ACTH)
and -lipotropin( LPH) via CRH

Limbic system (emotional


stimuli); reticular
formation(systemic
stimuli); hypothalamic and

Paraventricular nuclei

51. C. rennin-angiotensin aldersterone system (RAAS)


symp
(HR)

compensate

symp hemorrhage RAAS

compensate
compensate RAAS
vagal activation parasymp BP
HR
52.

- IGFs

bone

remodeling


GH

tyrosine kinase
GH IGF-I
GH

IGF
GH

metabolite
1. growth-promoting action:

GH IGF-I


epiphyseal plate
(chondrocyte,cartilage cell) long bone


collagen chondroitin matrix
2+

bone Ca

chondrocyte

osteocyte
epiphyseal plate bone shaft
GH
osteoclast osteoblast

(hyperplasia)


(hypertrophy) osteoblast
osteocalcin osteocalcin

osteocalcin
GH GH


2. metabolic action: GH



GH
(glucostatic effect) stress

GH (protein anabolic
effect)

lean body
mass nitrogen IGF insulin

GH
GH
insulin IGF
GH
IGF
GH PO4
2+

Ca

3-

GH

D Ca3(PO4)2 GH


RAAS ANP (
)
53.

B. insulin
ketoacidosis

urine ketone

positive
sp.gr.
ketone Na
complication DM type I diabetic
ketoacidosis DM type I type insulin
insulin

IDDM
insulin osmotic diuresis
polyuria polydipsia
polyphagia

hypovolemia GFR H excretion


acidosis
insulin
ketone bodies
DKA (. )
54. C. Atrial septal defect

( Left atrium)
Pulmonary artery +
Left atrium + %O2 saturation (
)
Left atrium
Right atrium
O2 Left to right shunt
ASD
55.

A. pH = 7.5,

pCO2 = 55, HCO3 = 34


-

= , = HCO3


a. pH 7.5 Pco2 55

HCO3

34 pH,
-

Pco2, HCO3
Part Physiology 2006
1. C. Dopamine

(Parkinsons disease)

3. C. ACh Receptor
myasthenia

gravis

antibody

autoimmune

Ach

receptor NMJ

Ach Ach receptor (

receptor block

nicotinic receptor)
action potential

muscle fatigue , weakness ,ptosis ( ),
( )
4. D. BP


Vulsava
Manuver Intrathoracic pressure
Venous return C.O.

(Phase1) Venous return


C.O. BP
( MAP = C.O. x TPR) Baroreceptor

BP drop


Syncope
ATP

5.

-
--

ATP = creatine phosphate <

glycogen < fatty acids


( )

Glycogen Ca


< 30

2+

100 (sprinter)

ATP ( 1.2 )

creatine phosphate (
9 )

creatine regenerate ATP


(ATP buffer)

Creatine phosphokinase

creatine phosphate

6. E.

turnover Ach receptor
myasthenia
antibody
NMJ

Ach

gravis

receptor

Ach Ach receptor

( receptor block nicotinic receptor)


action potential

E. turnover

Ach receptor

7.

D. TSH
Negative feedback

th

Williams Textbook of endocrinology 11 edition


Thyroid T3 T4
negative feedback Hypothalamus Anterior pituitary
gland TSH TRH
8.

D. Thyroid peroxidase activity


Deiodinase activity

Antithyroid drug Thioamides methimazole


(MMI) propylthyouracil(PTU)
Thyroid peroxidase catalyzed step
Thyroid hormone synthesis (oxidation, iodination,coupling)
PTU Extrathyroid action
T4 T3 (periperal tissue)
PTU Thyroid strom
Thioamides immunosuppressive
Gravesdisease thyroid stimulating
immunoglobulin(TSI) lymphocytic infiltration
Thyroid
9.

C. Increase GI

motility

Thyroid hormone, hyperthyroidism diarrhea


E. loose sphincter tension smooth muscle
10.

A. aldosterone
Na

K
aldosterone

- Reabsorption

Na Plasma Na

- K circulation cell Plasma K


+

- Plasma Na

ECF

volume BP

hyperaldosteronism
11.

A. prolactin

Pituitary tumor prolactinoma tumor


function prolactin Dopamine
prolactin
prolactin


prolactinoma galactolia
12.

C. gluconeogenesis
DM type II insulin

Fasting bood sugar cell resistance


insulin insulin
mild to moderate
Gluconeogenesis Gluconeogenesis

FBS () insulin
lipolysis lipolysis

ketogenesis ketoacidosis
insulin
FBS
C
13.

E. Triacylglyceride

DM lipolysis TG TG

14.

E. ACTH
ACTH dependent - Cushing Syndrome

ACTH MSH
ACTH dependent - Cushing Syndrome
ACTH MSH melanin
15.

E. ANP

volume
+

Na A C D
BP
ANP
blood volume Atrium ANP

16.

D. Growth hormone


Growth hormone

arginine (sleep stage
III,IV)
DA,NE(-adrenergic receptor), serotonin, enkephalin
GHRH GHIH
GH (REM sleep)
GH
GH
17. CAT epinephrine

cell, K

BP, metabolic
18.
21.

C. Intrinsic factor

B. osmotic pressure : oncotic pressure

osmotic pressure

PIF (interstitial
hydrostatic pressure) C (plasma oncotic pressure)

PC (capillary hydrostatic
pressure) IF (interstitial oncotic pressure)


albumin
C(plasma oncotic pressure)


edema
23.
pH

pCO2

HCO3

Resp. acid.

Resp. alk.

Met. acid.

Met alk.

pH 7.55 (base), pCO225(; normal 40),


-

HCO3 22 (; normal 22-26)

resp. alk.

compensate HCO3

24.

E. collecting
serum osm (normal: 275-300), urine

osm(normal: 50-1400), urine specific gravity :lower


normal(normal: 1.001-1.035)

e-lyte (
c.

Na proximal tubule d. Na distal


tubule )

collecting duct

ADH e.

26. E. NaCl reabsorption loop of Henle

(Na -K reabsorption Henles loop )
+

1.002-1.003
1.030-1.040 (osmolarity 50-100 1000-1400
mOsm/kg.H2O )
=

(particle)

ADH collecting duct


renal tubule


+

furosemide Na -K -Cl thick


ascending limb of Henles loop

total solute clearance osmolar clearance


(Cosm) free water clearance (CH2O)

CH2O < 0 CH2O > 0
(solute free water excretion)
1) countercurrent
multiplication system Henles loop urea recycle
hyperosmolarity medullary interstitium 2)
countercurrent exchanger vasa recta

hyperosmolarity medullary interstitium

3)

ADH

Distal tubule collecting duct
27.

D. BP

28.

A. HCG Corpus luteum


HCG corpus luteum progesterone


29.

A. LH
Ovulation predictor kits are widely avaible in pharmacies

LH LH peak
30.

E. HCG

31.

14

32.

C. gene expression
Estrogen steroid cell membrane

gene expression
33.

E.

34.

A. Pa CO2

alveolar ventilation(VA) = RR x (VT-VD)


Pt.1: RR 15 / VT 500 ml VD 200 ml : VA =
15(500-200) = 4500 ml
Pt.2: RR 30 / VT 300 ml VD 200 ml : VA =
30(300-200) = 3000 ml
Pt.2 VA Pt.1
Pt.1

Pt.2

Pt.2 PaO2

PaCO2

Pt.1
35.

A. Pa O2

Pa CO2

alveolar, elastin, lung capillary


fibrosis
gas

alveolar capillary

PaO2 20

chest diameter
diaphragm weak

pattern of breathing, lung expansion gas


exchange.
36.

C. 2, 3-DPG

37. D. HCO3

pH7.55: acid, PaCO2 25:, HCO3 22:lower


normal

Resp. alk. compensate


-

HCO3
pH

pCO2

HCO3

Resp. acid.

Resp. alk.

Met. acid.

Met alk.

38.

C. alveolar hypoventilation

pH7.25: acid, PaCO2 55:, PaO2 60 : HCO3

25: Resp. acid. compensate


-

reabsorb HCO3 HCO3

Resp. acid
midbrain
center alveolar
ventilation

39.

A. compliance of lung
surfactant

40.

A. pleural effusion

dull percussion fluid


()

b. peumotharax : gas
pleural cavity alveoli chest wall
pleural cavity



41. cough reflex

stimulant

42. C. Secretory IgA


: virus infection receptor





secretory IgA
receptor
d. alveolar fluid : alveolar fluid secretory IgA
IgG virus receptor tracheobronchial
epithelium

alveolar fluid

airway secretory IgA

43. F. Pressure upper respiratory tract


44.

IRDS(infant respiratory distress syndrome)

surfactant
45.

e. V/Q mismatches

consolidation rt. Lower lobe Lower lobe


ventilation & perfusion ( gas exchange
) consolidation gas diffuse
perfusion v entilation V/Q mismatches
a. hypoventilation
hypoven
b. anatomical shunt conducting
Zone of resp. tract
d. diffusion a.
diffuse diffuse
c onsolidation
46.

A. respiratory permeability

exudate
exudate respiratory epithelium

permeability WBC, complement


infectious agent
exudate
b. plasma oncotic pressure

c. plasma hydrostatic pressure


exudate

d. plasma oncotic pressure

e. lymphatic drainage visceral pleura


ECF circulation

47. B. oncotic pressure


Liver 2 cm below costal margin
neoplasm, cirrhosis, infection, inflammation

neoplasm
infect inflam

cirrhosis
pretibial edema
fluid leak cirrhosis
oncotic pressure
cirrhosis albumin
48.

A. surfactant

: surfactant24
RDS
RDSglucocorticoid
surfactant
49. D. Mitral regurgitation
murmur S1-S2
Systolic murmur 2 Aortic stenosis
Mitral regurgitation
intercostals space 5 midclavicular line

Mitral valve (
apex) Mitral regurgitation

50. C. tetralogy of fallot


Cyanosis
(Right to
Left shunt)

O2 Cyanosis
choice tetralogy of fallot Right
to Left shunt ( Left to Right shunt
)
**Tetralogy of Fallot (TOF)
4
1. (right ventricular
outflow tract obstruction)
2.

(Ventricular septal
defect : VSD)

3. aorta (Aorta
overrides the right ventricle) aorta 2

4. (hypertrophy

of

right ventricle)
pulmonary artery ( 1)
Right ventricle Right ventricular
pressure VSD
Left ventricle Right to Left shunt
51. choice Aortic perfusion
pressure
Aortic perfusion pressure aorta
(Aortic pressure)

aorta
aorta Aortic
perfusion pressure aortic
perfusion pressure artery

atherosclerosis
atherosclerosis Compliance
+ vasoconstriction total
peripheral resistance (TPR) mean arterial pressure
(MAP) ( MAP = C.O. x TPR)

52. E. PG ( )
Thrombosis PG
PG PGI2 (Prostacyclin)
Endothelial cell
Vasodilate platelete aggregation
PGI2 platelete
Coronary thrombosis
** Thrombosis
? NO
Thrombosis Endothelial cell function

pathway NO pathway
NO platelete aggregation

53. A. Cerebral
BP drop
Baroreceptor
Sympathetic activity BP


( Cerebral blood flow) Hypoxia
54. B. oncotic pressure
Liver 2 cm below costal margin
neoplasm, cirrhosis, infection, inflammation

neoplasm
infect inflam
cirrhosis
pretibial edema
fluid leak cirrhosis

oncotic pressure
cirrhosis albumin
55. D. Blood pressure
74
56. Bernoulli

57.

B. Venous return

(IVC )

IVC venous return


cardiac output

58. C. Aortic compliance


Aorta collagen tissue elastic
tissue compliance


BP
59. E.

Perfusion pressure
(
224 Aortic perfusion pressure) organ
perfusion pressure = organ pressure

Coronary perfusion pressure


pressure
60. A. Heart rate

Hemoorhage Stroke
volumn BP drop Baroreceptor
Sympathetic activity HR , contractility
+ Vasoconstrict internal organ
Splanchnic vessels vasodilate

61. D. T wave
T wave Ventricular repolarization
ST segment Isoelectric line MI
( MI) ST
elevation ST segment T wave
Isoelectric line
62. B. High CO2
Chemoreceptor 2
1.Central chemoreceptor Medulla Oblongata
response PCO2 blood pH
response PO2
2.Peripheral chemoreceptor Carotid body response
PO2 PCO2 response pH
Aortic body response 3

Chemoreceptor
O2 CO2

** Chemoreceptor
PCO2
63.

A. 2, 3-DPG.: hypoxemia

2,3-DPG

Rt. Shift

O2
b. CO2 Rt. Shift CO2
Bohrs
effect affinity O2 Hb
CO2
c. temperature Rt. Shift


d. fluid
e. surface tension of alveoli alveoli (
. alveoli)
O2
64.

E. combine metabolic and respiratory acidosis

pH: acid, HCO3 , pCO2, pO2


-

acidosis HCO3
-

Met. acid. pCO2 Resp. acid.


combine resp. met. acidosis
pH

pCO2

HCO3

Resp. acid.

Resp. alk.

met.

acidosis
Met alk.

65 A. Aortic stenosis
intercostals space 2 Aortic
Valve
murmur Aortic
stenosis
66. C. Mitral stenosis

S. Pyogenes
Rheumatoid heart disease
murmur
apex mitral valve
67. A. Family history
MI Risk factor 2
1.non-modified : age, sex, family history
2.modified : smoking, hypertension, DM, obesity, etc.
non-modified Risk factor Family
history
Part Physiology 2007

ANP

1.
2.

D. sympathetic activity

ECF volume blood volume


CO GFR renin

AII

BP
baroreflex
aldos

Symp

3. B. proximal tubule HCO3


-

HCO3
-

HCO3 proximal tubule


4.

C. Vasopressin

5. c.
1 FEV1 0.244 L
H

A1= A

A2=A +1
A1 : FEV1 = (1.052xH) (0.244xA) 0.561
A2 : FEV2 = [(1.052xH) (0.244(A+1)) 0.561
A2- A1: FEV2 - FEV1 = [(1.052xH) (0.244(A+1)) 0.561]
[(1.052xH) (0.244xA) 0.561]
= -0.244 L
1 FEV1 0.244 L
6.

C.

Heat stroke
7.

B. Elasticity
elastic fiber,

alveoli alveoli
elastic recoil gas
emphysema
a. FEV1/FRC
emphysema FRC(barrel-shape chest)
(emphysema obstructive disease
trap (
)

) FEV1 elatic
recoil obstuct
FEV1/FRC = / =
c. compliance elastic recoil
alveolar

alveolar
compliance
e. restrictive fiber : restrictive disease

fibrosis

( emphysema
obstructive disease)
8.

B. chylomicron

chylomicron (CM) apoprotein C-II

lipoprotein lipase


triglyceride chylomicron
apoprotein C-II
chylomicron remnant apoprotein apo E
apo B-48 ligand

9.

E. PaCO2

alveolar ventilation(VA) = RR x (VT-VD)


RR 15 / VT 500 ml VD 200 ml : VA =
15(500-200) = 4500 ml
RR 30 / VT 350 ml VD 200 ml : VA =
30(350-200) = 4500 ml
VA .PaCO2

10.

E. FEV1
FEV1

FRC

FVC

RV

FEV1/ FVC

TLC

Elastic recoil

VC

Lung compliance

IC

Airway resistance
Work of breathing(resistive work)

11.

E. cholesterol

12.

B. glycolytic enzyme
200

fast
fiber

fast fiber

anaerobic glycolysis

glycolytic
enzyme
13.

C.
Na reabsorption

14.

B. Low Pressure Receptor

Na + ANP, ADH + osmoreceptor, baroreceptor


H2O
15.

A. Na

15.

b. proximal tubule =
-

compensate HCO3
proximal tubule

a. glomerulus
c. loop of

Henle

d. distal tubule & e. collecting duct Na

ADH
16.

A. Hypoxia

D. hypercapnia

17. D. Sympathetic

Trauma Intracranial pressure
Cerebral perfusion pressure cerebral blood
flow CNS ischemic response
Hypothalamus Sympathetic activity
Peripheral vasoconstriction + C.O.

MAP
Cushing reflex
Cerebral perfusion pressure (CPP) = MAP Intracranial
pressure
18.

19.

A. Ach (Botox Clostridium

botulinum Snare protein End bulb Presynapse)


20.

C. pineal gland
Jet lag

pineal

21. D. Ab-cytotoxic hypersensitivity


Myasthenia gravis



40

acetylcholine






Myasthnia gravis
75%
Hypersensitivity Reactions, Immediate
The immune system is an integral part of human protection
against disease, but the normally protective immune
mechanisms can sometimes cause detrimental reactions in
the host. Such reactions are known as hypersensitivity
reactions, and the study of these is termed immunopathology.
The traditional classification for hypersensitivity reactions is
that of Gell and Coombs and is currently the most commonly

known classification system. It divides the hypersensitivity


reactions into the following 4 types:

Type I reactions (ie, immediate hypersensitivity reactions)


involve immunoglobulin E (IgE)mediated release of
histamine and other mediators from mast cells and
basophils.

Induction and effector mechanisms in typeI hypersensitivity

Type II reactions (ie, cytotoxic hypersensitivity reactions)


involve immunoglobulin G or immunoglobulin M
antibodies bound to cell surface antigens, with
subsequent complement fixation. Examples; Rh

incompatibility, myasthenia gravis, Rheumatic fever,


Graves disease

Type II hypersensitivity mechanisms

Type III reactions (ie, immune-complex reactions) involve


circulating antigen-antibody immune complexes that
deposit in postcapillary venules, with subsequent
complement fixation. Examples; Arthus reaction,SLE,
Rheumatoid arthritis

Mechanism of damage in type-III hypersensitivity

Type IV reactions (ie, delayed hypersensitivity reactions,


cell-mediated immunity) are mediated by T cells rather
than by antibodies. Examples; tuberlin (PPD) test,
chronic transplant rejection.

Mechanisms of damage in delayed


22.

hypersensitivity

B. A



glycogenolysis gluconeogenesis C D

Alzheimer
Pathophysiology of Alzhimers Disease

Alzheimers disease dementia


genetic disposition
1,12,14,19,21 19
Apolipoprotein E
21
-amyloid precursor small amyloid
peptides protein fibrils

senile plaques fibrin


-amyloid fibril

receptor for advanced glycation and products (RAGE) ,
scavenger receptor (RA)
depolarization
NMDA receptor

RAGE RA microglia cells


NO, Prostaglandins, excitotoxins, cytokines, tumor
necrosis factor (TNF-), tumor growth factor (TGF-1),
fibroblast growth factor (b-FGF) inflammation
NGF NGF
recoptor
Cholinergic neurons hippocampus entorhinal
cortex
cerebrum

neurotransmitter
Acetylcholine
cerebral cortex hippocampus
choline-acetyltransferase 90%
acetylcholine

neurotransmitter

cerebrum


(Anterograde amnesia) ,
, ,
limbic system
, lethargy, ,

2+

Ca

Ca
Exitable cell
Ca
Homeostasis

nerve growth factor


A B nerve growth
factor A B

Ca

23. A. oxytocin
Oxytocin


hypothalamus

stretch receptor


24.

D. progesterone
progesterone


ferning

25.

C. vasopressin

Stress, sp.gr.

26. E. Total peripheral resistance



Negative feedback
Baroreceptor
Vasoconstrict BP
BP drop Vasoconstrict
blood flow organ Metabolite
Local control
+

metabolite CO2 , H , K
vasodilate
metabolite
Vasodilate TPR BP drop
BP
Shock
** 2
1.Central control BP
Reflex Baroreceptor

2.Local control
constrict dilate
metabolite
27.

C.

Hemotoxic

DIC (Disseminated Intravascular Coagulation)


Fibrinogen
Nephrotoxic

28. B. RR interval
P wave , PR interval , QRS complex
Arrythmia
RR interval RR interval
HR RR interval HR
Hyperthyriodism

**- RR interval ( HR)


HR HR
Respiratory sinus arrhythmia (
Cardiac inhibitory center HR
)
29. C. Increase sodium reabsorption
RAAS
+

RAAS Aldersterone Na reabsorption


30. C. minute ventilation
minute ventilation =RR x VT

RR VT

a. tidal volume & d. inspiratory reserve volume


(inspiratory reserve volume)
(tidal volume)
a.

b.

RR a.

d. dead space ventilation


31. B. Airways resistance


COPD(Chronic obstructive
pulmonary disease)

airway
pleural cavity

pleural cavity
airway

airway pleural

cavity elastic tissue


airway air way
COPD, emphysema

airway

airway
airway trap
FRC residual volume

a. FEV1/FVC : FEV1/FVC
FEV1 airway obstruction airway resistance
1
pleural
cavity airway obstruction
c. residual volume : residual volume
airway obstruction

trap

residual volume
d. lung compliance : lung compliance
elastic recoil alveolar
alveolar
compliance
e. lung capacity : lung capacity



32. B. Precipitate Hemoglobin
Heinz bodies (unstable
hemoglobin)

1-3 m Wrights stain


()
brilliant cresyl blue Heinz bodies


phase contrast

Heinz bodies


G-6-PD
pyruvate kinase

33.

E. HCG

34. D. Vasoconstriction of peripheral vessels


55 Choice
55 choice
C. Sympathetic activity
Vasoconstriction of peripheral vessels
(Sym + Vasoconstrict)
Sympathetic activity

35.

C. peripheral chemoreceptor

36.

prolactin

amenorrhea galactorrhea
38.

A.
oxygen

:
RR
alveolar RR
alveolar O2 CO2
b. minute ventilation : minute ventilation RR x VT
RR VT

c. diffusing capacity

A : surface area

T: thickness of membrane

diffusion constant( gas)

D:

P1 P2 : gas
A, T, D
P1 P2
a. ()
d.

resistance :

39. A. 6.25
Poiseuille
2 (50%)
4

2 =

Central control
blood flow 16
blood flow 100/16 = 6.25 ml/min
40.

B. Vit D

41. E. ATP

ATP
myosin

affinity myosin

actin

actin

myosin

ATP
42. C. Vasovagal reflex
BP Drop

Baroreceptor Sympathetic activity **


Sym

Vagal tone Sym Peripheral vasodilate +


HR Vasovagal syncope
**

Stroke adams


(Heart block) HR Blood flow
(
)

** Vasovagal syncope
Blood perfusion (
) , ,
43.

( frontal lobe micturition center )


D. CCK

44.

: I cell of duodenum and ileum pancreatic


-

enzyme and HCO3 secretion, biliary secretion, growth


of exocrine pancreas and gallbladder ,delay gastric
emptying
a. motilin : upper duodenum during fasting state
gastrointestinal motility
b. gastrin : G cell of stomach gastric H
+

secretion and stimulate growth of gastric mucosa

c. secretin : S cell of duodenum pancreatic


-

HCO3 secretion,biliary HCO3 secretion, gastric H

secretion and

inhibit trophic effect of gastrin on gastric

mucosa
e. VIP : neuron of mucosa and smooth muscle
relaxation of smooth muscle, pancreatic and intestinal
secretion
45. D.
Hct, WBC Neu
lymphocytes , Platelet MCV
(The normal reference range is typically 80-97fL)
granulocyte
platelet

Hematopoietic stem cells give rise to two major progenitor


cell lineages, myeloid and lymphoid progenitors
46. . D Enzyme
Glucose-6-Phosphate
Dehydrogenase G6PD Deficiency

G6PD (Glucose-6Phosphate Dehydrogenase)

Pentose Phosphate Pathway


(PPP.) NADP NADPH
Glutathione reductase
Glutathione peroxidase
(Oxidants) H2O2
G6PD

(Oxidants)

(Hemolysis)
G6PD X-linked
recessive
Acute hemolytic anemia (
)


(Acute
renal failure)
Electrolytes ()
Hyperkalemia

Hb 8 g/dL Hct 30 Reticulocyte 8%
CBC
Hb , Hct


Reticulocyte



47. A. serum complement

hypersensititvity type 3

Type III reactions (ie, immune-complex reactions) involve


circulating antigen-antibody immune complexes that
deposit in postcapillary venules, with subsequent
complement fixation. Examples; Arthus reaction,SLE,
Rheumatoid arthritis
hypersensitivity type

complement

serum complement

48. B,C


Alcohol
Alcohol

Alcohol in large Quantities or in high concentration


damage the mucosa. While moderate drinking of wine or
beer increase

gastric secretion through their alcohol

component.
( Color Atlas of
pathophysiology)
- Excessive alcohol consumption causes damages to the
stomach or duodenum by impairing the integrity of the
mucosal barrier.
- alcohol may also cause irritation and erosion of the
mucous lining of the stomach and increase the production of
stomach acid.

- alcoholic drinks may stimulate the overproduction of


pepsin and gastric acids.
49.

A. DA
Cholinergic
( DA agonist )
C. DA Cholinergic
DA

(Parkinsons disease

Amantadine

Ach improve tremor)


50.

B C

Hypoxia oxygen

cerebral blood

flow oxygen
( )

Function
Attention
()
51. D.

anemia Hct + O2
carrying capacity metabolic demand
C.O.
( )
Aortic + pulmonic valve
Systolic murmur
** Reynolds number
Tubulance flow

52.

C. Hypothalamus

Hypothyroidism
primary secondary serum TSH
secondary hypothyroidism Hypothalamus
pituitary TRH stimulation test TSH
TRH Anterior pituitary

Hypothalamus(.
Thyroid axis )

LAB TSH
TRS stimulation test TSH
basal Hypothyroidism

Hypothalamic hypothyroidism

Diagnostic Tests for Hypothyroidism


b) Secondary

c) Hypothalamic

Hypothyroidism

Hypothyroidism

T4 low, TSH response

T4 low, No TSH

T4 low, Delayed TSH

exaggerated

response

response

a) Primary Hypothyroidism

The TRH stimulation test used to make the differential


diagnosis between primary hypopothyroidism and secondary
hypothyroidism.

TRH stimulation test


primary hypothyroidism Thyroid gland
/ negative feedback
TRH TSH
Thyroid hormone
TSH TSH
Lab secondary hypothyroidism
TRH stimulation test
TSH TSH Thyroid hormone

Hypothyroidism
TRH
TRH
TSH TSH

Thyroid hormone thyroid hormone


TSH TRH TSH Thyroid
hormone
http://www.islandnet.com/~rinfocan/thyroid.htm
53. B. Stroke volumn

Bernoulli BP
venous return SV

55.

A. insulin
Insulin

B. Thyroid hormone >>> anencephaly T


SH

C. TSH >>>
anencephaly
D. GH >>>maternal GH+fetal GH have no role in
regulation of fetal growth

E. HCG >>>
57. A. alveolar hypoventilation
midbrain
RR alveolar
hypoventilation



- gastroenteric reflex

(Biochemistry)

Part Biochemistry 2005


1.

DNA fragment GGACGG mutation GGCGG

mutation
A. silent

B. nonsense
C. transition
D. frame shift
E. transversion
2.

carnitine
A. glucose
B. glycogen
C. amino acid
D. fatty acid
E. triglyceride

3.

gene expression in RT-PCR RNA

A. transfer RNA
B. mitochondrial RNA
C. messenger RNA

D. heteronuclear RNA
E. ribosomal RNA
4.

Metabolism carbohydrate 100 g lipid 25 g protein

20 g
A. 300 kcal
B. 500 kcal
C. 700 kcal
D. 900 kcal
E. 1100 kcal
5. 5
Down
syndrome karyotype 46.XY

A. somatic mutation
B. deletion
C. translocation

D. undetected trisomy
E. mosaicism
6.

endonuclease AAUAAA
3 heterogenous nuclear RNA (hnRNA)

A. splicing
B. capping
C. transport
D. hybridization
E. polyadenylation

7.

A. riboflavin
B. biotin
C. thiamine
D. pentathonic acid

E. folate
8.

A. ketolysis
B. ketone bodies
C. gluconeogenesis
D. fatty acid
E. glycerol adipose tissue

9.

cytoplasmic organelle power house of cell

A. mitochondria
B. Golgi apparatus
C. lysosome
D. rough endoplasmic reticulum
E. centriole

10.

DNA

duplex

phosphorus 4 4 enzyme
type II restrictive endonuclease Bc/l 6 base
pair palindrome DNA duplex symmetry

DNA

11.

A. CCCGGT

GGGCCA

B. CCGGTT

GGCCAA

C. CGGTTG

GCCAAC

D. TTGATC

AACTAG

E. TGATCA

ACTAGT

55 40

chromosome abnormality
trisomy 21

A.

B.
C. endometrium
D. fallopian tube
E. non disjunction oocyte
12.

48
2
sigmoid colon biopsy neoplastic
glandular lesion invade colon
marker
monitor
A. -fetoprotein
B. CA- 125
C. alkaline phosphatase
D. lactic dehydrogenase

E. carcinoembryonic agent
13.

35 liver
function test total bilirubin 10 mg/dl,direct bilirubin 8
mg/dl, AST 700 U/L, ALT 1,000 U/L HBsAg
positive
A. viral toxicity
B. cyto
C. immune response
D. intranuclear multiplication
E. intracytoplasmic multiplication

14.

20 3

A. Cori cycle
B. beta oxidation

C. sorbital pathway
D. glucose-alanine pathway
E. hexose monophosphate shunt
15.

3
20

A. DNA cloning
B. DNA sequence
C. DNA fingerprint
D. Wentern blot
E. in situ hybridization
16.

A. sutosimal dominant
B. autosomal recessive
C. X-linked recessive
D. Non penetrance
E. Sporadic
17.

messenger RNA

uridine 135
136 amino acid
A. 134
B. 135
C. 136
D. 137
E. 138

18.

double stranded DNA nucleotide


5GGAACCATT3 nucleotide

19.
gluconeogenesis
Part Biochemistry 2006
1.

lipid


A. cholesterol
B. sphingolipid
C. phospholipids
D. fatty acyl
E. triglyceride

2.

3.

25
potassium cyanide
cyanide poisoning

A. glycolysis
B. Krebs
C. Electron transport chain
D. phosphooxidative

4.

Paraquat superoxide cell

injury

O2 free radical
A. B1 Thiamine
B. B6
C. A (Retinoic acid)
D. C (Ascorbic acid)
E. B5 Panthetoic acid
5.

Organelle steroid

6.

30 35

Downs syndrome 46 XY 45 XX
t(q14,q21) Downs
syndrome

A. 0 %
B. 25 %
C. 33.3%
D. 50 %
E. 99.9 %
7.

G6PD deficiency
1 16
chromosome 46,XX G6PD
deficiency
A. 0
B. 25
C. 50
D. 75
E. 100

8.

- - - GCSCGG - - - - - - GCCGG- - -

A. non sense
B. missense
C. frame shift
D. silence
9.

5
chromosome
A. Down syndrome
B. Turner syndrome
C. Edward syndrome
D. Klinfelter syndrome
E. Cri du chart syndrome

10.

20 DM lab
A. C-peptide

B. ketone body
C. TG
11.

Albinism
A. Tyrosinase
B. Hexokinase
C. Galactase
D. Lactase
E. Maltase

12.

60 10
Hepatic encephalopathy

A. Lipogenesis
B. Bile formation
C. Urea metabolism
D. Ketone bodies formation

E. Estrogen biotransformation
13.

1 6
2

A.
B.
C.
D.

14.

( )
A. B1
B. B2
C. B5
D. folic
E. B12

15.

30

A. DNA repair
B. cell membrane repair
C. Hb synthesis
D. free radical scavenger
E. 2,3-DPG
16.

60
Hepatic encephalopathy
A. Lipogenesis
B. Bile formation
C. Urine formation
D. Ketone formation
E. Glycolysis

17.

LFT, SGOT, SGPT ALP 3

18.

50

A. urea
B. Albumin
C. Ammonia
D. Ketone body
19. 25 5

A.
B. detergent

20.

40 56 trisomy 21
A.

B.
C. non-disjunction of ovum
D. endometrium
E. Fallopian tube

21.

25


(myocardial infarction)

A. Saturated fatty acid


B. Polyunsaturated fatty acid
C. Monounsaturated fatty acid
D. Medium chain fatty acid
E. Long chain fatty acid
Part Biochemistry 2007
1.

3
kwashiorkor amino acid
A. Threonine
B. Tryptophan
C. Methionine
D. Aspartic acid

E. Phenyl alanine
2.

5 glioma 4 cm
right frontal lobe
cell
A. inactivation of bcl2 gene
B. inactivation of p53 gene
C. inactivation of precursor of MDL-2 gene
D. activation of BCL-XL gene
E. activation of retinoblastoma gene

3.


ethanol ethanol

A. Product inhibitor
B. Allosteric inhibitor
C. Irreversible inhibitor

D. Competitive inhibitor
E. Feedback inhibitor
4.

2
Bitots spot ()

A. Retinol
B. Thiamine
C. Riboflavin
D. Pyridoxine
E. Ascorbic acid

5.

deep tendon reflex


metabolic pathway
A. glycolysis
B. Krebs cycle
C. Glucogenesis

D. Glycogenesis
E. Oxidative phosphorelation
6.

acute lymphoblastic leukemia


uric acid metabolism

A. heme
B. purine
C. pyrimidine
D. amino acid
E. glucoronide
7.

50

5 . Hb

A. sulfhemoglobin
B. methamoglobin

C. oxyhemoglobin
D. carboxyhemoglobin
E. oxidized hemoglobin
8.

50
1 lab creatinine kinase
troponin-t
A. dissociation of ribosome
B. swelling mitochondria
C. clumping nuclear chromatin
D. Fragmentation plasma membrane

9.

5
platelet 200,000/
cu.mm.
A. Deamination of alanine
B. Hydroxylation of praline

C. Phosphorylation of serine
D. Glycosylation of hemoglobin
E. Carboxylation of glutamic acid
10.

apoptosis
A. P53 protein
B. bcl-2
C. IL-6
D. tyrosine kinase
E. TNF

11.

Pathway RBC
A. Urea cycle
B. Krebs Cycle
C. Beta oxidation
D. Lactose fermentation

E. Glycolysis
12.

50 1
2


A. Retinol
B. Thiamine
C. Riboflavin
D. Cobalamine
E. Niacin
13.

45 MTP joint
uric 8.2 mg/dl
metabolism
A. purine
B. protein
C. glucose

D. cholesterol
E. pyrimidine
14.

mutation ...ATGCCAAA...
DNA probe
A. UUUGGCAU
B. UACGGUUU
C. TTTGGCAT
D. TACGGTTT
E. TTTGGUAT

15.

X telomerase


A. DNA stability
B. DNA replication
C. DNA degradation
D. Gene rearrangement

E. Spindle fiber attachment


16.

Vitamin
A. thiamine
B. folate
C. Riboflavin
D. B12
E. niacin
17.

Phagocytosis
neutrophil enzyme O2-dependent mechanism

A. Lysozyme
B. Defensin
C. Hydrolaze
D. Lactoferrin

18.

5 down syndrome
karyotype 46,XY
A. deletion
B. mosaicsm
C. translocation
D. somatic mutation
E. undetected trisomy

19.

1
hydrolytic enzyme enzyme organelle

A. acrosome
B. lysosome
C. ribosome
D. microsome
E. peroxysome




1.Western blot ...
? ()
2.alpha-fetoprotein
3.gene apoptosis

1
2552
17

1. 60 BUN 80 ml/dl

creatinine 8

transamination
nitrogen waste product
A. Leucine
B. Lysine
C. Threonine

D. Glutamate
E. Aspartate
2.
A. Oxytocin
B. prolactin
3. 45 specific gravity 1.001

A. Aldosterone

B. Antidiuretic hormone
4. 35

A. Muscle protein
B. Lipid
C. Liver glycogen
D. Muscle glycogen
E. Ketone body
5. Epigenetic inactivation X chromosome
DNA

A. Phosphorylation
B. Mutation
C. Gene rearrangement
D. Gene deletion
E. Methylation

6. autosomal recessive

PCR

Band carrier
A.A
B.B
C.

D.

7. 20
BP 130/80
of both legs

PR 110/min moist skin, muscle weakness

A. Insulin
B. Glucagon

C. Aldosterone
D. Epinephrine
E. Thyroxine
8. ketoacidosis

A. Protein metabolism
B.

Protein degradation

C. Fatty acid oxidation


D. Fatty acid synthesis
E. glycogenolysis
9. 25 3
1

A. Estrogen
B. Progeterone
C. LH
D. FSH
E. HCG

10. 25 3
blood sugar 35 mg/dL

A. lipolysis
B. gluconeogenesis
C. glycogenolysis
D. glycogenolysis
E. pentose phosphate pathway
11. 5

A. Arginine
B.
C. Tryptrophan
D. Methionine
E. Hydroxyproline
12.

A.alcohol dehydrogenase

B.acetaldehyde dehydrogenase
13. epigenetic gene
A.methylation
14.

A.transduction
B.translocation
C.conjugation
D.transposition
15. anticodon tRNA 5 - - - 3 3
codon
16.keloid
A. fibroblast
B. granulation tissue
C. collagen

17. ketone bodies (


ketone bodies
)
A. lipolysis
B. gluconeogenesis
C. fatty acid metabolism
D. fatty acid synthesis
18. xy,+21
A. translocation
B. meiosis

nondisjunction

C. deletion
19. labia majora
gamete
A. xy
20.

A. 69,xxy
B .46,xx
C. 46,xy
D.23,xx
E. 23,xy
21. ( )
A. tryptophane
B. tyrosine
22. log michealis Y
X antagonist
23. PG type
A. PGI2
B. PGF2
C. PGD2
24. posttranslation

A.ER

B.ribosome
C.nucleus
D.vesicle

(Biochemistry)
Part Biochemistry 2005
1. D. Frame shift
2.

D. fatty acid

Carnitine

Lysine Methionine
Fatty acyl CoA mitochondria
beta-oxidation cell
3
1.

Carnitine-acyl transferase 1
outer membrane of
mitochondria
Fatty acyl CoA LCarnitine O-acylcarnitine
intermembrane space of mitochondria

2.

Acylcarnitine translocase
innermembrane of
mitochondria
O-acylcarnitine
matrix mitochondria

3.

Carnitine-acyl transferase 2
innermembrane
of mitochondria
L-carnitine Fatty
acyl-CoA beta-oxidation

carnitine
Malonyl-CoA

AcetylCoA + CO2

---- MalonylCoA

enzyme

AcetylCoA Carboxylase Biotin ( Vitamin B6 )


Co-enzyme
MalonylCoA inhibit Carnitine-acyl transferase

3.

C. messenger RNA

4.

C. 700 kcal

(100

4)+( 25

9)+(20

4) = 705

5.

C. translocation

6.

E. polyadenylation
hnRNA transcript 3

AAUAAA
enzyme endonuclease
polyA
7. C. thiamine

tannic acid

tannic caffeic acid


Vit B1
Vit B1
8.

transketolase

B. ketone bodies

( Starvation ) fat adipocyte

hormone cortisol , glucagon

Triglyceride Fatty acid Glycerol


Fatty acid albumin
ketone body ketone
body
substrate


preserve RBC ,
renal medullary cell
ketone body ( Glycerol
DHAP enzyme Glycerol
kinase
9.
10.

gluconeogenesis )

A. mitochondria
E. TGATCA ACTAGT

Bc/l restriction enz.


11. E. non disjunction oocyte

35
non-disjunction meiosis
oocyte
12.

E. carcinoembryonic agent

Carcinoembryonic antigen tumor marker

follow up colorectal cancer (


non-neoplastic lesion
ulcerative colitis , pancreatitis , cirrhosis)
-fetoprotein

- fetus

adult

tumor marker Hepatocellular


carcinoma
CA-125

tumor marker ovarian tumor

lactic dehydrogenase

RBC

marker hemolysis
follow up cancer
lymphoma turn over

Dysgerminoma elevate LDH first sign


13.

C. immune response

parenchyma of liver

infected hepatocyte


14.

E. hexose monophosphate shunt




G6PD activate
oxidative stress antibiotic
sulfonamide

15.

C. DNA fingerprint

16.

C. X-linked recessive

17. E. 138

UUU

CUG

CCA

UAU

UUU AUG

UGU

UAA(stop

codon)
18.

3 CCTTGGTAA 5

19.

Ethanol metabolize

enzyme alcohol
Dehydrogenase cytoplasm acetaldehyde (
metabolize
mitochondria )
NADH NADH cytoplasm
NADH enzyme Lactate
dehydrogenase pyruvate lactate
pyruvate
substrate gluconeogenesis

NADH enzyme
gluconeogenesis (lactate

peripheral tissue skeleton muscle


lactic acidosis
)
Part Biochemistry 2006
1.

E. triglyceride

2.

Adipose tissue

3.

C. Electron transport chain


potassium cyanide

cytochrome c oxidase
Electron transport chain

tissue

oxygen anaerobic glycolysis -> lactic acidosis


,
cyanosis cardiac arrest

2
4.

D. C(ascorbic acid)

5.

D.

steroid cholesterol
cholesterol
(
acetylCoA
LDL-receptor)

mitochondria

pregnenolone
pathway
steroid substance ( SER
pregnenolone)
6.

C. 33.3%

Robertsonian Translocation
6

A.

14
21

Monosomy 21

21
B.

C.

D.

21
14

Monosomy 14

14

(spontaneous abortion)

14 21

Robertsonian

Translocation ( 21 14 )
balanced translocation
E.

Robertsonian
Translocation 21
trisomy 21

F.

(Down

syndrome)

Robertsonian
Translocation

14

trisomy 14

carrier Robertsonian Translocation


t(14q;21q)

(live birth)

Down syndrome 1: 3
7.

A. 0

G6PD deficiency X-link

recessive
2
1 allele


) 3
1

XX
XX

XY

XY
*

XX
XX

XY

XX

XX

XY
*

XY
*

XY
*

XX

XY

XX XY

XX

XY

3
X-link recessive

2
1

recessive

8.

C. frame shift

9.

D. Klinfelter Syndrome
Turner

syndrome monosomy x

Edward syndrome

trisomy 18

Klinfelter Syndrome karyotype 47,XXY or 48,XXXY


10.

A. C-peptide
DM

type1 autoimmune

beta-cell pancreas insulin deficiency


C-peptide C-peptide
exocytosis insulin marker
endogenous insulin
11.

A. Tyrosinase
Albinism
autosomal recessive Tyrosinase deficiency
Tyrosinase enzyme tyrosine melanin
melanocyte hypopigmented skin eye hair

12.

C. Urea

metabolism

Hepatic Encephalophathy

metabolize

deamination amino acid


aromatic amino acid phenylalanine

metabolize
urea urea cycle

ammonia alpha-ketogutarate
neuron alpha-ketogutarate neuron
aromatic amino acid
neuron channel tryptophan
tryptophan neuron 5-HT (
Tryptophan 5-HT )
13.

D.

CHO

14.

E. B12

Vit B12

bacteria yeast
15.

D. free radical scavenger


G6PD
deficiency NADPH G6PD
enzyme pentose phosphate pathway

NADPH free radical


Glutathione
G6PD free radical
16.

C. Urea

17.

Cholestasis

formation

ALP ( Alkaline Phosphatase ) enzyme


enzyme

clinic marker
cholestasis

18.

C. Ammonia

urea cycle urea NH3

NH3
19.

B. detergent

20.

C. non-disjunction of ovum

21.

C. Monounsaturated fatty acid

- (MUFA)
cholesterol (LDLc)
- w-6
1 LDLc
1.35 100 10
HDLc
- w-3
a linolenic triglyceride
(ADA
US RDA RDI 2007 etc.)

Part Biochemistry 2007


1.

D. Aspartic acid

Aspartic acid

oxaloacetate
enzyme AST ( Aspartate
transaminase)
kwarshiorkor

2.

D. activation of BCL-XL gene

BCL-XL Bcl-2 family

apoptosis mitochondrial
pathway

expression

gene induce
apoptosis
3.

D. Competitive

inhibitor

methanol

structure ethanol

inject ethanol

enz. Alcohol DHnase enz.


methanol toxic product
( formaldehyde formic acid toxic
)
4.

A. Retinol

= night

blindness retinol
Rod & Cone cell
Biots spot = Xerosis of epithelium conjunctiva
5.

B. Krebs cycle

1
Thiamine pyrophosphate coenzyme enzyme
metabolism

1.

Pyruvate dehydrogenase
pyruvate
acetylCoA

2.

-ketoglutarate dehydrogenase
ketoglutarate SuccinylCoA Krebs cycle

3.

Transketolase pentose phosphate pathway

(beri beri ) deep


tendon reflex neuron
6.

B. purine

purine uric acid

7.

D. carboxyhemoglobin

8.

B. swelling mitochondria

9.

B. Hydroxylation of proline

matrix protein collagen loose

connective tissue
Platelet 200,000/ cu.mm.
platelet

confirm

10.

B. bcl-2

11.

E. Glycolysis

12.

D. Cobalamine

B12

B12 ( cobalamine)
coenzyme enzyme 5-Methyltetrahydrofolatehomocysteine methyltransferase ( MTR )
5

N -methyl-tetrahydrofolate Tetrahydrofolate
coenzyme thymidine
B12



megaloblastic anemia
( pernicious
anemia)

13.

A. purine

14.

C. TTTGGCAT

15.

A. DNA stability
Telomerase enzyme DNA
telomere DNA replication
DNA
replication ( senescence )
breakdown chromosome enzyme
telomerase DNA stability

16. D. B12
17.

E 4 choice
phagocytosis

phagocyte 2
1.

O2-dependent mechanism enzyme



free radical

2.

oxidation

O2-independent mechanism enzyme



Lysozyme bacterial cell wall 1,4beta-linkages between N-acetylmuramic acid Nacetyl-D-glucosamine


peptidoglycan

Defensin

cell wall bacteria

Hydrolase

lactoferrin mucosa innate


immunity
( ETS )
18.

C. translocation

19.

B. lysosome



1. Protein
2. Neural Tube Defect
3. Bcl-2

1
2552

17

1. Ans D
glutamate dehydrogenase
ketoglutarate


glutamate dehydrogenase

-ketoglutarate

glutamate + H2O + NAD

NH4 + NADH + H

2. Oxytocin
(Hypothalamus) (Posterior
pituitary gland)

(Persistalsis)
Oviduct Oxytocin

Prolactin Lactogenic hormone (LTH)

androgen

3. (vasopressin) ADH (antidiuretic


hormone)

Antidiuretic
hormone increases the permeability of the distal portion of
the nephron allowing more water to be reabsorbed from the
nephron back into the blood stream which dilutes the blood
and concentrates the urine. Alcohol interferes with the
production of antidiuretic hormone which is why beer drinking
leads to frequent trips to the bathroom.
aldosterone mineralocorticoid

(K )
(distal renal tubules)
+

(Na )





+

(H )



-

(Cl ) (HCO3 )


4.
1
glucose-6-phosphatase
G-6-P glc
G-6-P

glc Glc
glucose-6-phosphatase Glc

5. epigenetic
5'
(CpG) (-CH3) 5'


(methylation)

6.
7. Thyroxin

metamorphosis

Hyperthyroid





Adrenalin hormone Epinephrine hormone

( )
glycogen
(adrenalin
)
Insulin cell
insulin
insulin
glycogen

insulin ( diabetes

mellitus )





(Ketone Body)



Glucagon
( insulin )

8. Protein degradation

branch chain

amino acid ketone body


ketoacidosis
Fatty acid oxidation
acetyl co A acetyl co A

oxaloacetate TCA cycle


acetyl co A

oxaloacetate

acetyl co A

ketone bodies acetoacetate, D--hydroxybutyrate


acetone
TCA cycle
renal cortex
ketone bodies
ketone bodies ketone bodies
pH acidosis
ketone bodies ketosis

9. The hormone human chorionic gonadotropin (better

known as hCG) is produced during pregnancy. It is made by


cells that form the placenta, which nourishes the egg after it
has been fertilized and becomes attached to the uterine wall.
Levels can first be detected by a blood test about 11 days
after conception and about 12 - 14 days after conception by
a urine test. In general the hCG levels will double every 72
hours. The level will reach its peak in the first 8 - 11 weeks
of pregnancy and then will decline and level off for the
remainder of the pregnancy.

10. 3


gluconeogenesis glucogenic
amino acid

Lipolysis acetyl co A

acetyl co A Glc
glycogenolysis 24
glc
gluconeogenesis
glycogenolysis

pentose phosphate pathway

NADPH
-5 -

11. Vitamin C hydroxylation


proline lysine

12. acetaldehyde dehydrogenase


acetaldehyde

Ethanol----1

acetaldehyde

-2

acetic acid

1: alcohol dehydrogenase; 2: aldehyde dehydrogenase


13. epigenetic


methylation

14. Bacterial conjugation is the transfer of genetic

material between bacteria through direct cell-to-cell contact.

[1]

Discovered in 1946 by Joshua Lederberg and Edward Tatum,


[2]

conjugation is a mechanism of horizontal gene transferas

are transformation and transductionalthough these


mechanisms do not involve cell-to-cell contact.

[3]

15.
16. collagen

keloid granulation
tissue


exuberant granulation
fibroblast
connective tissue
desmoid aggressive fibromatosis.
17. fatty acid metabolism ketogenesis
ketone bodies acetyl CoA
18. meiosis
19.

nondisjunction Down syndrome

20. hydatidiform mole 46,XX


genomic imprinting
21. tyrosine melanin pigment
22. Competitive inhibition
Lineweaver-Burk plot of competitive inhibition. Note the
complete relief of inhibition at high [S] (ie, low 1/[S]).

23.
24.
Hydrophobic signal peptide appears at the N-terminus of
protein.
Signal recognition particle (SRP) binds to signal peptide,
ribosome, GTP and stops elongation.

Ribosome-SRP cpx is bound by receptors on ER.

Newly synthesized protein is translocated into the ER


lumen.

The signal peptide is cleaved from protein by peptidase


within the lumen.
Ribosome dissociates and is recycled.
The protein is made into the ER, then folded and
packaged into a vesicle.

(Pharmacology)

Part Pharmacology 2005


1.

55 gout allopurinol

enzyme
A. xantine oxidase
B. adenine deaminase
C. guanine deaminase
D. adenine phosphoribosyltransferase
E. guanine phosphoribosyltransferase
2.

55 recurrent ventricular

arrhythmia 5
increased

ESR pulmonary function

diffuse lung capacity diffuse


interstitial pneumonia
A. atenolol
B. verapamil

C. propanolol
D. amiodarone
E. procainamide
3.

50 chronic obstructive pulmonary

disease theophline
14
2 theophyline
theophyline
A. rifampin
B. isoniazid
C. cimetidine
D. hydrothiaxide(hydrochlorothiazide?)
E. pyrazenamide
4.

mydriasis c ycloplegia
A. atropine

B. pilocarpine
C. neostigmine
D. phenylamine
E. phenylephrine
5.

38 amphetamine

BP

170/110 mmHg
amphetamine BP
A. potent -1 adrenergic agonist
B. potent -2 adrenergic agonist
C. inhibition of catecholamine metabolism
D. release of internal catecholamine
E. metabolism to false neurochemical transmitter

6.

30

BP

112/76 mmHg PR

88/min BP 80/60 mmHg

PR 120/min

A. dopamine
B. isotonic saline
C. colloid solution
D. 5% dextrose in water
E. fresh flozen plasma
7.

effect digitalis Na-K pump axon


+

A. Na
+

B. K
-

C. Cl
D. inhibit propagation of action potential
E. hyperpolarization membrane potential

8.

4 tuberculin test

positive
isonizid prophylaxis 4

isoniazid ( 4

)
A. heme
B. metabolism hepatocellular injury
C. billirubin albumin
D. lymphatic obstruction
9.

mean arterial

pressure
peptidase
A. histamine
B. bradykinin
C. serotonin

D. angiotensin ll
E. neuropeptide Y
10.

46 cholesterol

CPK

A. captopril
B. propanolol
C. nicotinic acid
D. simvastatin
E. hypochlorothiazide
11.

40 deep vein thrombosis warfarin

A. kininogen
B. prothrombin

C. Hageman factor
D. thrombomodulin
E. antihemophilia A factor
12.

39


impressed vertebral fracture (osteoporosis)

A. vitamin D
B. calcium
C. estrogen
D. progesterone
E. prednisolone
13.

antrax
spore

A.
B.
C.
D. hypochlorite 400 ppm
spore
E. Phenol 1 %
14.

bronchiolar resistance

epinephrine
x x epinephrine

x
A. nadolol
B. pindolol
C. propanolol
D. metropolol
E. isopoterenol
F.

15.

50 dyslipidemia antilipid drug

A. probucol
B. gemfibrozil
C. simvastatin
D. nicotinic acid
E. cholestyramine
16.

40

A. dimenhydrinate
B. domperidone
C. metoclopamide
D. scopolamine
E. cyclizine

17.

50 digitalis

A. adrenylate cyclase
B. Na-K ATPase
C. adrenergic receptor
D. dopamine receptor
E.
18.

30 3 thin

film RBC
trophozoit, Shuffner dot
A. quinine + primaquine
B. quinine + atezunate
C. quinine + cloroquine
D. cloroquine + atezunate
E. cloroquine + primaquine

Part Pharmacology 2006


1.

30 50 kg 20 mg/kg half- life

A. 2 .
B. 4 .
C. 8 .
D. 9 .
E. 10 .
2.

30 L/kgs

apparent of volume distribution (Vd)


A.
B. hydrophilicity
C. protein-binding
D. blood brain barrier
E.
3.

Azotimycin afferent volume of distribution [V d] = 50


l/kg

A. binding protein
B.

4.

rapid metabolism
loading dose

5.

maintain dose

A.

B.

C.

D.

E.

vaccine

1
A. multistep trial
B. pharmacokinetic activity
C. pharmacodynamic activity

D. pharmacoeconomic activity
E. none of the above
F. double blind effect
6.

glaucoma bete-blocker

A. aqueous humor
B. trabecular membrane
C. iris dilator muscle
D. papillary. Muscle
7.

antiemetic drug brain


A. cerebrum
B. midbrain
C. Pons
D. medulla

8.

diuretic
glaucoma

9.

50
prolonged muscle relax LAB Abn. Plasma
cholinesterase
A. Atracurium
B. vercuronium
C. Pancuronium
D. Suopotamia
E. Sucrinylcholine

10.


A. Ergotamine
B. ASA
C. Ibuprofen
D. Diazepam
E. Morphine

11.

35


A. Heroin
B. Alcohol
C. Gasolene
D. Morphine
E. Amphetamine

12.

Long acting relax


A. Tubocurarine
B. Pancuronium
C. Romum
D. Scopolamine
E. Succinylcholine

13.

45 type 2 2

glipizide

fasting blood glucose 110 mg/dL, HbA1C 6.5


%
A.
insulin
B. insulin receptor
C. cortisol
D. ........
14.

45 7

4 24
A. water diureis
B. solute diuresis
C. osmotic diuresis
D. ADH
E. ADH

15.

estrogen

hormone

A. Interleukin
B. interferon
C. calcitonin
D. Thromboxane A2 (TXA2)
E. TNF
16.

50

moon face Na+


+ plasma sugar
A. Estrogen
B. Progesterone
C. hydrocortisone
D. Thyroid

E. ACTH
17.


3
1
moon face lab
Na+ 120 K- 5 C l- 105
Blood glucose 60 globulin20

A. estrogen
B. hydocortisone
C. ADH
D. ACTH
E. TSH
18.

40 20

deep tendon reflex

A. B1
B. B2
C. B3
D. B6
E. B12
19.

A. B1
B. B2
C. B6
D. B12

20.

1 hyoscine
enteric nervous plexus
A. afferent fiber

B. meissner plexus
C. mucosal plexus
D. myenteric plexus
E. submucosal plexus
21.

30 Nephrotic syndrome

Prednisolone WBC
Lab : WBC 14,000 Platelet 200,000 (Neutrophil
Lymphocyte )
A.
Bone Marrow
B. Neutrophil
C. maturation Neutrophil
D.

permeability / migrate vessel

circulation
E.
granulocyte CFS

22.

congestive heart failure furosemide

digitalis
furosemide digitalis toxicity
A. hypo K+
B. hypo Na+
C. hypo Cl
D. hypo Ca2+
E. metabolic alkalosis
23.

A. free radical scavenger


24.

20


A. R..
B. tamoxifen
C. Ergonovine

D. Mifedistone(Mifepristone?)
25.

endometriosis Danazole

irreversible effect
A.
B.
C.
D.
E.
25.

50

Tamoxifen

A. Estrogen negative feedback
B. Gonadotropin
C. FSH
D. Estrogen receptor

E.
Progesterone
26.

Benign Prostate Hyperpasia finesteride

hormone
A. DHT
B. Estrogen
C. LH
D. Didydro( Dihydrotestosterone)
27.

Salbutamol
A. mucus
B. beta - agonist

28.

60
isosorbide dinitrate

A.
guanylate cyclase
B.
B2-adrenergic receptor
C. ionized calcium cell

D. B1-adrenergic receptor
E. ACE
29.

transient myocardial infarction aspirin

A. decarboxilation
B. COX enzyme
C. Hexokinase
30.

50 3 1 T 39 C BP 80/50

fluid therapy
BP 80/60
A. Dopamine
B. Vasopressin
C. Epinephrine
D. Zsopetunal
E. hydrocortisone

31.

25 30 AMI

A. Plasmin
B. Protein C
C. Antithrombin
D. Prostacyclin
E. Phospholipase
32.

acute MI ER fibrinclytic drugs


A. protein C
B. plasmin
C. antithrombin
33.

multiple myeloma Ca2+ 12 mg/dL

Ca 2+

A. Thiazide
B. Kay
33.

A. 2 nd wk
B. 4 th wk
C. 12 th wk

Part Pharmacology 2007


1.

cyclosporine cell
A. B-cell
B. T-cell
C. Macrophage
D. NK cell
E. PMN

2.

Nanosum


A. Phase IIa Clinical trial
B. Phase IIb Clinical trial
C. Phase III Clinical trial
D. Phase IV Clinical trial
3.

60 80

Number needed to
treat
A. 5
B. 20
C. 33.3
D. 50
E. 10

4.

Terbutaline sulfate

A. alpha1 antagonist
B. alpha2 agonist
C. beta1 agonist
D. beta2 agonist
E. antimuscarinic

28

respiratory distress syndrome surfactance


surfactant

A. lung compliance
B.
6.

25
discharge

intracellular gram negative

diplococci penicillin

A. Imipenam
B. Gentamicin
C. Erythromycin
D. Ciplofloxacin
E. Bactrim

7.

20 Barbiturate

1
RR 12/min arterial blood gas

A. pH 7.5
B. PaCO2 55 mmHg
C. PaO2 100 mmHg
D. Bicarbonate 18 mEq/L
E. Base excess +4 mEq/L

8.

systematic review amoxicillin


Amoxylin clavuranic acid sinusitis
causal association
A. temporalily
B. reversibility
C. consistency
D. dose-effect
E. biological plausibility

9.

X max 5 mg 1 hr

max 25
mg 2 X Y
A. 5
B. 10
C. 25
D. 40

E. 50
10.

N-acetylcystein paracetalmol toxicity


liver
A. N-acetylation
B. N-hydroxylation
C. sulfer conjugate
D. glutathione conjugate
E. glucolonide conjugate

11.

5 3 Stifness of neck

CFS Gram
Gram negative diplococci
A.
B.
C.

D.
E.

12.

60 BMI=30

A. Metformin
B. Acarbose
C. Meglitinide
D. Glitazone
E. Glipizide
13.

K= 6, Na = 135 , Cl

A. Spinoloractone
B. Priqnerene
C. Amiloride
D. Furosemide

E. Eplereone
14.

20 Haloperidol

A. inhibit dopaminergic
B. inhibit cholinergic
C. inhibit serotonin
D.
dopaminergic
E.
cholinergic
15.

25 40

A. Oxytocin
B. Ritrodine
C. Ergotarmine
D. Vasopressin
E. Bromocriptine

16.

50

acute myocardial
infarct
mechanism activity
A. plasmin
B. Protein C
C. AntiThrombin
D. Prostacyclin
E. Phospholipid
17.

27 carbamet ,muscle
fasciculation, , , PR 60 beat/min
Atropine
A.
B. Muscle fasciculation
C.

D.
E. 60 beat/min
18.

50

KOH
preparation hyaline septate hyphae
A. Fluconazole
B. Clotimazole
C. Amphotericin B
D. Bemzyl benzoate
E. Sodium thiosulfate
19.

35


single treatment
A. Ca
B. Vit D

C. Prednisolone
D. Progesterone
E. Estrogen
20.

5 Hemophilia A

coagulation
A. Aged plasma
B. Cryoprecipitate
C. Fresh frozen plasma
D. Platelet
E. Fresh whole blood
21.

Point mutation Ribosome Subunit 50 Bacteria


A. Cloxacillin
B. Ceftriaxone
C. Norfloxacin

D. Tetracycline
E. Erythromycin
22.

25


A. paraquat
B. organophosphate
C. ethylene glycol
D. arsenical compound
E. sodium hypochlorite
23.

Organophosphate
A. Ach
B. Ach
C. ACh esterase
D. reuptake Ach

E. sensitivity Ach receptor


24.

Venous thromboembolism
INR = 5



- asthma
- succinylcholine action
- organophosphate action
- first line drug of DM
- haloperidol action
- penicillin
- ciprofloxacin
- fasciculation flaccid

Pharmacology
(
)

Part Pharmacology 2005


1.

A. xanthine oxidase
Key word gout
gout uric acid

Uric acid

metabolic product purin


enzyme xanthine oxidase hypoxanthine
& xanthine
Purine bases Inosine Hypoxanthine Xanthine
Uric acid
allopurinol xanthine oxidase inhibitor
uric acid hypoxanthine & xanthine 2

2.

D. amiodarone
Key word arrhythmia, pulmonary

function, pneumonia
recall amiodarone arrhythmic
adverse
effect
atenolol


Side

-blocker 1-selective

effect

indigestion,

constipation,

fatigue,

dizziness, faintness
verapamil

Antiarrhythmic agents class IV (Ca

2+

channel blockers)
Side effect : constipation, CHF LV dysfunction,
hypotension
propanolol

-blocker Nonselective

Side effect : fatigue, bronchoconstriction

procainamide Antiarrhythmic agents class IA (Na

channel blockers)
Side effect : proarrhythmic effects (e.g., torsades de
pointes)
3.

C. cimetidine (enzyme inhibitor )


Key concept Drug Interaction
Key word theophyline
Cimetidine, ciprofloxacin, erythromycin

enzyme

inhibitor theophylline theophylline


enzyme inducer phenobarbital, phenytoin,
rifampin
4.

E. phenylephrine
Key word mydriasis cycloplegia
Key concept autonomic control of the eyes

Mydriasis 1-adrenoceptor radial muscle


muscarinic receptor circular muscle (
contract miosis)
Cycloplegia = loss of power in ciliary muscle
cycloplegia
1-adrenoceptor

ciliary muscle

phenylephrine

1-selective adrenoceptor agonist


phenylramine H1-receptor antagonist
allergic conjunctivitis
atropine

Antimuscarinic drugs

ciliary muscle
pilocarpine

Cholinomimetics M3

receptor ciliary muscle


neostigmine

Cholinesterase-inhibiting drugs

ciliary muscle

5.

D. release of internal catecholamine


Key word
amphetamine, BP 170/110

mmHg
Amphetamine
cathecholamines nerve terminals
6.

B. isotonic saline
electrolyte


isotonic saline
electrolyte colloid solution


dextrose


dextrose electrolyte dextrose
Fresh flozen plasma
factor
7.

B. K
+

Key concept mechanism of Digitalis action &


action potential
Digitalis enzyme Na+/K+ ATPase pump Na+ 3
K+ 2 K+
K+ gradient resting
membrane potential
action potential: Na+ depolarization;
K+ repolarization
hyperpolarization

K+

Na+/K+ ATPase K+

membrane potential

hyperpolarization Digitalis hyperpolarization of


membrane potential
B

membrane potential resting potential


hyperpolarization
8.

B.

metabolism hepatocellular

injury
Key concept
metabolite
of isoniazid acetylhydrazine hepatic damage
B.
9.

Key word peptidase

mean arterial pressure


peptidase

protein/peptide

peptidase peptide bond peptide/protein


Histamine = amine
bradykinin = peptide

serotonin = 5- hydroxytryptamine
angiotensin ll = octapeptide, vasoconstrictor
neuropeptide Y = vasoconstrictor
10.

A,C

D. simvastatin
Key word

CPK
D

recall

Simvastatin HMG CoA

reductase inhibitor cholesterol


first choice hypertension

nicotinic acid

triglyceride & hepatotoxicty &


common side effects: flushing & dyspepsia

11.

B. prothrombin

Key concept Coagulation Factors & Mechanism


of warfarin action
Mechanism of action warfarin enzyme
Vitamin K epoxide reductase Inactive vitamin K epoxide
Vitamin K (active hydroquinone
form) Precursors of clotting factors II, VII, IX, X
Active clotting factors II, VII, IX, X
Prothrombin Thrombin
Prothrombin Extracellular
prothrombin B. Prothrombin
. Extracellular prothrombin Intracellular
prothrombin
( : Journal of the American society of hematology)
12.

C. estrogen
Key concept Osteoporosis in menopausal

women

estrogen deficiency estrogen


bone resorption
13.

A.


disinfection

anthrax C. anthrax spore


B., D. E.
bacterial spore disinfection (
bacteria, virus )
, hypochlorite phenol
14.

A. pindolol
Key concept

Sympathetic drugs & -

adrenoceptors
heart rate & bronchiolar resistance
(= bronchodialtation)
1- adrenoceptor heart rate

2 -adrenoceptor bronchial smooth

muscle
adrenoceptors

Epinephrine 1 & 2-

X 1- adreoceptor

100 % 2- adrenoceptor

2-

adreoceptor 100 % intrinsic sympathomimetic activity


(
sympathetic nerve)
X - adreoceptor
(nonselective antagonist)

antagonist
recall

()
Propranolol, nadolol, timolol & pindolol nonselective
- adrenoceptor antagonists

pindolol

intrinsic sympathomimetic activity ( )

Metopolol

1- selective adrenoceptor antagonist intrinsic


sympathomimetic activity

Isoproterenol

adrenoceptor agonist
15.

D. nicotinic acid
Probucol Side effect : decrease HDL level (20-30%)
Gemfibrozil

Side effect : GI side effects

Simvastatin

Side effect :

Nicotinic acid Side effect : Flushing of body and face


Cholestyramine

Side effect : GI side effects

D. nicotinic acid
16.

Key concept motion sickness


- Antihistamines: Cyclizine (long duration of

action),

hydroxyzine

(antipruritic*),

promethazine,

dimenhydrinate (Dramamine, most effective OTC drug) &


diphenhydramine
-

Anticholinergic

agents:

scopolamine

(p.o.

or

transdermal patch: i.v., most effective prescription drug)


A., D. E.
B., C. B., C.
Dopaminergic antagonist GERD, vomiting,
gastroparesis

17.

18.

A. quinine + primaquine
Key concept maralial diagnosis & treatment
Key word trophozoit, Schuffner
Schuffner dot P. vivax & ovale malaria

malariae Ziemanns dot

[P.

P. falciparum Maurers dot]

hypnozoites relapse hypnozoites


Primaquine

A & E

chloroquine > quinine A


Part Pharmacology 2006
1.

half-life : Y

maximum

X
X
half-life
2.

C. protein-binding

Key concept
Drug distribution
Vd

= Amount of drug in body/ Drug concentration in

blood or plasma
Vd
tissues e.g. muscle, adipose tissue &
drug receptor

tissues

membrane

( B

)
Plasma proteins tissue
components Vd plasma volume (3 )

bound drug

C. protein-binding (
)
* protein-binding Vd

3.

4.

D.

Key concept Pharmacokinetics & Drug Dosing

Loading dose
minimal effective concentration

IV Loading dose Vd
metabolize

maintenance dose

MEC
rate of drug elimination drug metabolism &
drug excretion
D.

IV loading dose
maintain dose

prodrug

loading dose
& maintain dose
5.

C. pharmacodynamic activity
Key concept Drug development Clinical

trials (testing, study)

Phase I normal volunteers/ special population (renal and


hepatic impairment)
- biological effects, safety, metabolism, kinetics, drug
interaction
Phase II- selected patients
- therapeutic efficacy, dose range, kinetics, metabolism
Phase III- large sample of selected patients
- safety, efficacy
NDA review NDA approved marketing
Phase IV- post-marketing surveillance
- patients given drug for therapy
- adverse reaction, pattern of drug utilization, additional
indications discovered
=
. .. C. pharmacodynamic activity

Pharmacodynamics is the study of the biochemical and


physiological

effects

of

drugs

on

the

body

or

on

microorganisms or parasites within or on the body and the


mechanisms of drug action and the relationship between drug
concentration and effect
6.

A. aqueous humor
Key concept autonomic control of intraocular

pressure
(ciliary epithelium & blood vessel) receptors
( 2)

receptor

aqueous humor ( )
7.

A.

D. medulla
Emetic center medulla (lateral reticular formation)

chemoreceptor trigger zone (CTZ) base


of fourth ventricle

(in area postrema of medulla)

CTZ

receptors antiemetic
dopamine

receptor

(Metoclopramide,

Promethazine),

5HT3

receptor (Ondansetron), muscarinic receptor (Scopolamine or


hyoscine), opioid receptor
8.

D.

Key concept Mechanism of action of diuretics


acetazolamide (carbonic anhydrase inhibitor)

aqueoushumor intraocular pressure


Osmotic diuretics (Glycerin, Isosorbide, Mannitol,
Urea) plasma osmotic pressure
intraocular pressure acute attack of
glaucoma, ocular surger
9.

E. Succinylcholine
(RECALL: Succinylcholine)
Key word ,

Abn. Plasma

cholinesterase
Succinylcholine (SCh) Depolarizing drugs
2 phase

Phase I :

Depolarization of motor end-plate

Phase II :

Muscle relaxation

prolonged paralysis
Plasma ChE
(Cholinesterase) Abnormal plasma
cholinesterase
Abnormal plasma
cholinesterase
10.

A. Ergotamine
Key concept Migraine therapy
Ergotamine migraine severe

50-70%

5-HT1-receptor partial agonist, vasoconstrictor, affect adrenoceptors blocks trigeminal nerve transmission
side effect increase BP,
myocardial infarction, stroke

11.

E. Amphetamine
Key concept CNS stimulation
Amphetamine Phenethylamine derivatives

Methylphenidate
Clinical effects Amphetamine Cocaine
Amphetamine increase the
concentration of catecholamines transmitter in the synaptic
cleft (
)
alertness
fatigue insomnia
12.

A. Tubocurarine B. Pancuronium
Long-acting muscle relaxants onset of action & clinical

duration of action Tubocurarine onset 4-6

duration

80-

120 Doxacurium 4-6, 90-120


120-180

Pancuronium 4-6,

Pipercuronium 2-4, 80-100

Succinylcholine short-acting muscle relaxant: onset 1-1.5


duration 5-8

A. B. long-acting drugs

13.

A. insulin
Recall : glipizide sulfonylurea

insulin
14.

C. osmotic diuresis
Key words

Key concept : dehydration


hyperglycemia osmotic diuresis
15.

A. Interleukin

Key concept Estrogens & Bone remodeling


Estrogen

- ***number & activity of osteoclasts
osteoclast- stimulating cytokines: IL-1, IL-6, TNF-

osteoblast & stroman cell


antiresorptive

insulin-like growth factor (IGF)-1, bone

morphogenic protein (BMP)-6, & transforming growth factor


(TGF)-
- bone organic matrix type I
collage, osteocalcin, osteopontin, osteonectin markers of
differentiated osteoblast alkaline phosphatase
- osteocyte survival apoptosis

Estrogen Osteoprotegerin (OPG)


RANKL RANKL RANK
hematopoietic osteoclast precursor cells & mature osteoclast

RANK gene expression differentiation of


osteoclast precursor cells, maturation of osteoclast
Estrogen bone resorption
Cytokines differentiation of
osteoclast precursor cells, maturation, activation of osteoclasts
& apoptosis
2 Interleukin TNF
IL osteoporosis ( )

TNF in vitro bone loss in arthritis &


periodontal disease
16.

C. hydrocortisone (cortisol)
Key word moon face

glucocorticoid effect

effect

moon face key word key


word

17.

E. TSH
Key concept

[ Na+ 120 glucose 60

(?)]
. Acute hyponatremia
< 135 mEq/L;

(Hyponatremia : plasma [Na ]


+

[Na ] < 120 mEq/L

severe weakness, consciousness


the National Institute for Diabetes and Digestive
and Kidney Diseases (NIDDK)
120 mg/dL

blood sugar 60 -

60 mg/dL

shakiness, dizziness,

sweating, hunger,

headache, irritability, pale skin color,


, clumsy or jerky movement,
, tingling sensation .
[ Drugs and
Fluids

used

disorders]

to

treat

volume,

electrolyte

and

acid-base

Hypovolemic hyponatremia
, , Addisons disease,
burns, , hypotonic
fluid isotonic NaCl (0.9%)
Euvolemic hyponatremia
syndrome

of

inappropriate

hypoadrenalism,

secretion

hypothyroidism,

polydipsia

renal

of

ADH

failure,

(SIADH),

psychogenic

SIADH Lithium

Demeclocycline (
ADH)
Hypervolemic hyponatremia


CHF, hepatic cirrhosis renal failure

,

dialysis

advanced renal failure

Plasma sodium menopausal women


Estrogen
. moon face glucocorticoid
hydrocortisone
ADH hyponatremia
ACTH sodium retention
Aldosterone

cortisol

hypothyroidism hyponatremia
thyroid
hormone permissive action aldosterone

ADH
E. TSH thyroid
hormones
18.

A. B1
Key concept Thiamine deficiency

. Alcoholics

19.

A. B1
.

alcohol intoxication .

unconscious (hypoglycemia)

IV thiamine (vitamin B1) 50% Dextrose


solution hypertonic solution

. Alcoholic

malnutrition thiamine coenzyme


glucose metabolism

glucose B1

Wernickes encephalopathy
Wernickes encephalopathy time- dependent reversibility
. 40 % residual deficit

Encephalopathy

triads ocular abnormality (primarily horizontal


nystagmus or bilateral sixth nerve palsy), ataxia, and global
confusion

pyridoxine(B6) supplement .
Cirrhosis 50 .
B1
20.

D. myenteric plexus
Recall: anatomy & parasympathetic drugs
Enteric nervous system 2 plexuses:
Meissner

plexus

(submucosal)

sensing

the

environment within the lumen, regulating GI blood flow and


controlling epithelial cell function (secretion)
Auerbachs (myenteric) plexus circular
longitudinal smooth muscle layers tunica muscularis
digestive motility

plexus [myenteric
mys (Greek, ) + enteron (intestine)]


hyoscine (scopolamine)
muscarinic receptor antagonist ( plexus muscarinic
receptor)
21.

B. Neutrophil
Glucocorticoids apoptosis neutrophils

22.

A. hypo K+
Recall: Digitalis Na+/K+ ATPase

hypokalemia receptor toxicity


Furosemide Inhibit Na+,K+,2Cl- cotransport (symport)
thick ascending
limb of Henles loop Na+ tubules
late distal convoluted tubules collecting ducts
aldosterone reabsorb Na+ K+
hypokalemia
23. -

,dehydrate ,
, rifampin
24.D. mifepristone
Mifepristone progesterone and glucocorticoid receptor
antagonist (RU486) emergency contraception
PGE analog early abortion
25.A.
Danazol testosterone side effect
liver dysfunction, virilism (acne,
hirsutism, oily skin, reduced breast size), HDL
side effect irreversible deep voice , enlargement of
the clitoris, increased body hair
25.D. estrogen receptor
Tamoxifen selective estrogen receptor modulator
(SERM) D receptor ( A)
26.A.

DHT

fenasteride -Reductase inhibitors

testosterone dihydrotestosterone (DHT)


27.B.

beta-agonist

Salbutamol beta2 agonist (short acting)

28.A. gualnylate cyclase


Isosorbide dinitrate nitrate angina
NO
cGMP gualnylate cyclase
endothelial vasodilation
29.B.

COX enzyme

Aspirin antiplatelet NSAID COX


TXA2 platelet aggregation

aspirin analgesic

(),antipyretic (),antiinflammation
30.A.

dopamine

sepsis shock ()
mediator vasodilation dopamine

vasoconstriction ( epinephrine
web dopamine BP sepsis)
epinephrine anaphylactic shock steroid
sepsis antiinflam
31.A.

plasmin

Fibrinolytic plasminogen plasmin

fibrin 2 1. Tissue
plasminogen activator alteplase 2. Streptokinase
32. - furosemide ()
diuretic Na K Cl cotransport channel
lumen-positive potential --> -->
lumen -->
Ca, Mg
Thiazide Ca Na/Ca exchange
basolateral membrane Ca
Kayexalate (sodium polystyrene sulfonate)
hypokalemia

33. B. 4 th week
upper limb bud 26

Pharmacology 2007
1. B. T-cell
Cyclosporine immunosuppressant
helper T cell IL ,IFN gramma
2. D. phase IV Clinical trial
Phase IV phase
long term effect
3. A. 5
number needed to treat (NNT)
1
NNT = 1/ARR ARR absolute risk

reduction CER EER CER control group


event rate EER experimental group event rate
60 80 60% 80%
40% 20%
CER 40% 0.4 EER 20%
0.2
NNT = 1/(0.4 - 0.2) = 5
4. D. beta2 agonist
Terbutaline beta2 agonist (short acting)

5. A.
lung compliance
Surfanctant
compliance

6. D. ciplofloxacin ()
Imipemam carbapenam Gram positive
cocci,Gram neg rod ,anaerobe, etc

Gentamicin aminoglycoside Gram neg


aerobe
Erythromycin macrolide Gram + cocci
and bacilli
Ciprofloxacin fluoroquinolone Gram neg
UTI GI
Bactrim beta lactam inhibitor
choice cephalosporine
7. B. PaCO2 55 mmHg
Barbiturate sedative-hypnotic drug overdose
severe respiratory and cardiovascular depression
-

respiratory acidosis PaCO2 HCO3 side effect


liver enzyme inducer, psychomotor dysfunction, CNS
depression
8. C. consistency
epidemiology occupational health

Criteria for determining Causation


1. Temporality: Does the presumed cause precede the
effect?
2. Reversibility: Does removal of a presumed cause lead to
a reduction in the risk of ill-health?
3.

Dose response relationship (dose- effect): Is increased


exposure (dose) to the possible cause associated with
an increased response (i.e. an increased likelihood of an
effect) ?

4. Biological plausibility: Is there a reasonable postulated


biologic mechanism linking the possible cause and the
effect?
5.

Consistency: Have similar results been shown in other


studies?

Elsewhere you can learn how to critical

appraise literature. It follows that if a number of good


studies using different approaches lead to the same
interpretation of a cause-effect relationship it is more
likely to be a valid one.
c. consistency
9. A. 5

max
2 efficacy 5 (duration
)
10. D. glutathione conjugate
N-acetylcysteine

glutathione

paracetamol 98%
conjugate glucuronide sulfate nontoxic

metabolize P-450

mixed function oxidase toxic metabolites


glutathione (GSH)
toxic metabolite paracetamol
paracetamol toxic
metabolites GSH
11. A.
Neisseria meningitides A.



12. A. metformin
Metformin
hepatic gluconeogenesis

glycolysis BMI (
)
13. D. furosemide
hyperkalemia Furosemide
Na/K/Cl channel --> K --> K
( K sparing diuretic)
14. A. inhibit dopaminergic
Haloperinal dopamine neuron
extrapyramidal symptom
adrenergic (alpha)
15. A. oxytocin
Oxytocin
ergotamine

postpartum bleeding bromocriptine


dopamine agonist Parkinson
16. A. plasmin
216
17. B. muscle fasciculation
Carbamet acethylcolinesterase inhibitor

Ach muscarinic nicotinic
Atropine muscarinic receptor antagonist
muscarinic nicotinic neuron NMJ
muscle fasciculation
18. B. Clotimazole
(local) -->
Clotrimazole
19. E. estrogen
--> estrogen
and progesterone --> osteoporosis --> hormone

replacement therapy estrogen (progesterone


osteoporosis)

20. B. Cryoprecipitate
hemophilia A factor VIII
cryoprecipitate factor VIII,XIII
21. E. :Erythromycin ( macrolide)
antibiotic ribosome 50s
Chloramphenical macrolide
Tetracycline ribosome 30s
22. paraquat arsenic
paraquat arsenic herpecide paraquat
GI irritate, pulmonary impairment arsenic
RBC renal failure

Organophosphate insecticide , ethylene glycol


antifreeze , sodium hypochlorite antiseptic pesticide
23. A. Ach esterase
muscarinic parasympathetic

nicotinic NMJ

24. vit K
prothrombin time (PT), prothrombin ratio (PR)
international normalized ratio (INR)
extrinsic pathway of coagulation

warfarin, liver damage and vitamin status.


reference PT 12-15 INR 0.8
-1.2 ( = 0.9 1.3)
warfarin 2.0-3.0

INR
INR

bleeding
ISI

INR = [PT()/PT(normal)]

ISI (International Sensitivity Index) tissue


factor
sensitivity internationally
standardized sample
1.0 1.4

INR extrinsic
pathway Factor VII
half-life


vitamin K


Vitamin K



Platelet blood vessel

(Microbiology)

(Parasitology)

Part Microbiology 2005


1.

5 2 pharynx

submandibular area beta-

hemolytic colony blood agar


bacitracin
A. Haemophilus influenza
B. Coryneacterium diptheriae
C. Streptococcus pyogenes(group A)
D. Mycoplasma pneumoniae
E. Mycobacterium tuberculosis
2.

35 right upper quadrant


1 BT

38 C

A. HBV
B. CMV
C. Entamoeba histolytica
D. Mycobacterium tuberculosis

E. Salmonella typhi
3.

35
antibiotics 10
toxin
A. Salmonella typhi
B. Shigella dysenteriae
C. Escheridchia coli
D. Helicobacter jejuni
E. Clostridium difficile

4.

40 1
cavity at right upper lobe
positive acid fast bacilli

A. cell wall mycolic acid


B. cell wall protein carbohydrate

C. cell wall
D. cytoplasm , ribosome
E.
5.

antrax
spore
A.
B.
C.
D. hydrochoride 400 ppm
spore
E. Phenol 1 %
6.

16
gram positive rods

A. Chlamydia sp.
B. Clostridium sp.
C. Bacterioides sp.
D. Actinomyces sp.
E. Pseudomonas sp.
7.

CSF cell count

A. acanthosis
B. spongiosis
C. ballooning degeneration
D. cyotolysis
E. liquafactive degeneration
8.

8
2

gram positive cocci in chain


complete hemolysis blood agar
A. Staphylococcus aureus
B. Staphylococcus epidermidis
C. Streptococcus pyogenes
D. Streptococcus viridans
E. Streptococcus pneumoniae
9.

30

3-4 24

Gram positive rod spore

A. Botulium toxin

B. Clostidium enterotoxin
C. heat-labile enterotoxin

D. heat-stable enterotoxin
E. toxin c.perfingens type A
10.

specimen

A. pus
B. urine
C. sputum
D. throat swab
E. CSF
11.

30


A.

Vibrio sp.

B.

Shigella sp.

C. Salmonella sp.
D. Campyrobacter jejuni

E.
12.

Yersinia enterocolitica

25

A. Shigella sp.
B. Salmonella enteriditis
C. Clostridium difficile
D. Bacillus cereus
E. Staphylococcus aureus
Part Microbiology 2006
13.

SLE 3 X-Ray Cavity

Upper Right Lung


modified acid fast
..

A. Norcardia sp.
B. Aspergillus sp.
C. Acitinomyces sp.
D. Streptomyces sp.

14.

antibiotics 10



A. E. coli
B. Clostridium difficile
C. Shigella
15.

3
gram

positive cocci enzyme


A. elastin
B. laminin
C. fibronectin
D. hyaluronic acid
E. collagen type 4
16.

2 alpha-

hemolytic group, diplococci, lancet shape, gram positive



A. Klebseilla pneumoniae
B. Staphylococcus aureus
C. Neisseria meningitidis
D. Streptococcus pneumoniae
E. Pseudomonas aeruginosa

17.

Salmonella typhi reservoir


A. Intestinal lymphatic tissue
B. Gall bladder
C. Liver
D. Spleen
E. Thymus

18.

10

KOH

A. Pityriasis versicolor
B. pediculosis
C. scabiasis
D. larva migrane
E.
19.

eruptive

KOH preparation septate

hyphae

A. Aspergillus sp.
B. Malassezia furfur
C. Exophiala werneckii
D. Trichosporum (Tricophyton sp.?)
E. Candida albicans
20.

Cryptococcal meningitis

A. Ionic
B. lipid solubility
C. anionic acidic
D. low MW
E. low lipid H2O coefficient
21.

20 2 ........

lymphocyte


A. CMV
B. EBV
C. Adenovirus
D. Coxsackie B virus
E. Influenza virus
22.


fecal contamination
A. Salmonella sp.
B.

E.coli

C. Staphylococcus aureus
D. Vibrio cholera
E. Clostridium perfringens

F. Streptococcus fecalis
23.


discharge


A. E.coli
B. Streptococcus group B
C. Neisseria species
D. Lactobacillus species
E. Staphylococcus sp.
24.

20 discharge

gram negative rod



A. E. coli
B. Neisseria gonorrhea
C. Chlarmydia trachomatis
D. Pseudomonas aeruginosa
25.

35 cervicitis
cervical discharge



A. Mobiluncus spp.
B. Lactobacillus spp.

C. Gardnerella vaginalis
D. Chlamydia trachomatis
E. Staphylococcus epidermidis
26. 30 5





sperm analysis


A.
B.
C.

D.
E.
27.


(
)
A.
B.
C.
D.

E.


28.



lymph node
A. Actinomyces israelii
B. Clamydia trachomatis
C. Neisseria gonorrhoeae
D. Treponema pallidum

29.

gram

A. Syphilis
30.





A.

Dientamoeba fragilis

B. Staphylococcus aureus
C. Pseudomonas aeruginosa
31.

20


Respiratory epithelium
A. cell lysis

B. cell degeneration
C. mucous production
D. cell fusion and shrinkage
E. cell necrosis and exudates formation
32.

10 pneumonia
gram positive bacteria
(
)
A. Haemophilus influenzae
B. Staphylococcus aureus
C

Staphylococcus epidermidis

D. Streptococcus pyogenes
E. Streptococcus pneumoniae
33.

3 Mitral valve

vegetation

A.

Viridans streptococci

B. Staphylococcus aureus
C. Staphylococcus epidermidis

34.

Cell count 10,000 (N 90 L 10)


Glucose 30 Protein 80

A. Neisseria meningitidis
B. Haemophilus influenzae
C. Listeria monocytogenes
D. Streptococcus agalactiae
E. Streptococcus pneumoniae

Part Microbiology 2007


35.

Urinalysis lab

14.00 .
A. 14.30 .
B. 15.00 .
C. 16.00 .

36.

D. 18.00

E. 20.00

25

polymyositis bacterial

coagulase catalase

A. Staphylococcus aureus
B.

Staphylococcus epidermidis

C. Staphylococcus hemolyticus
D. Staphylococcus pyogenes
E.
37.

Staphylococcus pneumoniae

red vesicle,

erythema and papule 2


histopathology intranuclear halo,
multinucleated giant cell
A. HSV
B. Gonococcal infection
C. CMV
D. Treponema pallidum

E. HPV
38.

20 8 murmur

echo 0.5 cm vegetative mitral 7


Bacteria
A. Enterococcus faecalis
B. Staphylococcus aureus
C. Haemophilus aphrophilus
D. Pseudomonas aeruginosa
E. Staphylococcus epidermidis
39.

25 2

Gram +ve diplococci


Pneumonia Criteria sputum gram
strain
Epithelium (/HPF)
A. 10

WBC(/HPF)
40

40.

B. 25

30

C. 30

50

D. 50

10

E. 30

40

Squarmous cell CA cervix


A. Parvovirus
B. Herpes
C. EBV
D. CMV
E. HPV

41.

5 stiffness of

neck CSF WBC 1,200 cell/cu.mm. N 90%


MN 10% protein 150 g/dl sugar 20 g/dl(blood sugar
100)
A. Enterovirus

B. Neagleria floweli
C. Neisseria meningitidis
D. Cryptococcus neoformans
E. Mycobacterium tuberculosis
42.

2
x-ray ureteral
calculi
A.

Esterichia coli

B.

Proteus mirabilis

C. Enterococus faecalis
D. Krebsiella pneumoniae
E.
43.

Staphylococcus saprophyticus

40


normal flora
A.

Clostridium difficile

B.

Shigella dysenteriae

C. Salmonella enterica
D. Yernisia enterocolitica
E.
44.

Plesiomonas shigelloides


modified acid fast
A.

Nocardia asteroides

B.

Penicillium maneffii

C. Clostidium perfringen
D. Mycobacterium chelone
E.

Streptomyces spp.

45.

30 6


A. E.coli
B.

Vibrio cholerae

C. Shigella
D. Bacillus cereus
E.
46.

Aeromonas hydrophilia

E.coli
Colony forming unit / mL

47.

17
LAB ........., Nitrite +
A. E. coli
B. Enterococcus spp.
C. Trecomonas vaginalis
D. Candida albicans

48.

peritonitis
A. Bacteriodes fragilis

49.

5 stiffness of
neck CSF WBC 1,200 cell/cu.mm. N 90%,
MN 10% protein 150 g/dl, sugar 20 g/dl (blood sugar
100)
A. Enterovirus
B. Naegleria fowleri
C. Neiseria meningitidis
D. Cryptococcus neoformans
E. Mycobacterium tuberculosis

50.


A. Erysipelothrix fhusiopathiae
B. Aeromonas hydrophila

Parasitology
Part Parasitology 2005
1.

35
2

2.

40

A. tiiasis Taeniasis

Trichinosis (
Taeniasis solium saginata
)
B. spargadiasis sparganosis
C. gnathostomiasis
D. cystececosis cysticercosis
E. fasicolopiasis fasciolopsiasis

3.

RBC,
WBC ()

A. Isopora belli
B. Giadia lambria
C. E. coli
D. E. histolitica
Part Parasitology 2007
4.

10 occult blood positive

stool exam


A.
B.
C.
D.

E.

5.

A. praziquentel
C. metronodazol

B. albendazol

D. ivermectin

E. amphotericin B

6.
A. Opisthorchis Viverrini



- scotch tape technique

Microbiology

Parasitology

Part Microbiology 2005


1.

C. Streptococcus pyogenes (group A)


A. Haemophilus influenza
B. Coryneacterium diptheriae

----

Gram ----

cocobacilli

Gram + bacilli ,

Chinese letter
C. Streptococcus pyogenes (group A)--- Gram + cocci
D. Mycoplasma pneumoniae
bacteria

----

Cell wall-less

E. Mycobacterium tuberculosis

----

Acid fast bacilli

-Streptococcus group A Streptococcus


pyogenes
Blood agar
( -hemolysis)

Lancefield group B

Acute (Suppurative) Streptococcus pyogenes Infection


1. Pharyngitis
anterior cervical lymph node

2. Scarlet fever skin infection


24 . sandpaper rash, circumoral pallors,
(strawberry
tongue)
3. Pyoderma/impetigo

cellulitis
Complication

infection

1. Rheumatic fever

pharyngitis autoantibody
molecular mimicry

hypersensitivity

type II (cell-mediated type) antibody


heart tissue


pericarditis , endocarditis , myocarditis erythema
marginatum (chorea later)
2. Acute glomerulonepritis (M12 serotype)
pharyngitis

skin infection

autoantibody molecular mimicry


hypersensitivity type III immune
complexes glomeruli pulmonary edema
hypertension dark urine
Streptococcus pyogenes
catalase (Staphylococcus ) lysis Blood agar
-hemolysis bacitracin ( sensitive

bacitracin) positive pyrrolidonyl


arylamidase (PYR) test

1.

M-protein
antiphagocytosis

2.

Hyaluronic capsule (polysaccharide)


antiphagocytosis non-immunogenic

3.

Erythrogenic toxin (Streptococcal pyrogenic exotoxin)


exotoxin 3
A, B C A

-

toxin phage (lysogenic phage)


phage-encoded

(scarlet fever)
(Strawberry tongue)

endotoxin

superantigen helper
T cells T cell
receptor (TCR) MHC class II markers
processes antigen

4.

Streptolysin S non-immunogenic

hemolysin cytolysin

Streptolysin S
Blood
agar
5.

Streptolysin O immunogenic
hemolysin
cytolysin toxin

Rheumatic fever

6.

Hyaluronidase hydrolyzed ground substances


connective tissue

7.

2.

Spreading factors Streptokinase, DNAse

C. Entamoeba histolytica
Salmonella sp.
Entamoeba histolytica

10 % 90
% amoebiasis

3
-

Trophozoite

Precyst

Cyst

1.

Trophozoite

Vagetative form


10-30 normal saline
pseudopodia
cytoplasm
cytoplasm
ectoplasm endoplasm
granule

NSS iron hematoxylin



2. Precyst

(chromatoidal bar)

ribonucleic acid
acid

deoxyribonucleic

and phosphate
3.

Cyst

5 -20

cyst 1-4

cytoplasm
Chromatoid body

2
1. (Intestinal amoebiasis)
2. (Extraintestinal amoebiasis)
1. (Intestinal amoebiasis)
1.1 Acute amoebiasis

Mature cyst

Excystation amoeba 4 binary


fusion

8 invade

hyalunidase mucosa

mucosa submucosa


submucosa
(Flash shape)

electrolyte

1.2 Chronic amoebiasis


acute amoebiasis

- Amoeboma

E.histolytica

cyst

E.histolytica
- Peritonitis

- (Hemorrhage)
- (Appendciitis)

- Stricture

granuloma

(Diagnosis)
- Trophozoite cyst E.histolytica
- Proctoscope sigmoidoscope
- serology ELISA,
CIEP,IHA,IFA Latex agglutination test,Precipitin test
,Complement fixation test
2. (Extraintestinal amoebiasis)
trophozoite E.histolytica

3
1. Haematogenosus portal circulation

2. Lymphatic
3. Direct extension hepatic flexure
colon

- Amoebic liver abscess




- Cutaneous and genital amoebiasis
- Pulmonary amoebiasis
- Cerebral amoebiasis
- Spleenic abscess

3.

E. Clostridium difficile
A. Salmonella typhi

(rare case)

B. Shigella dysenteriae

(rare case)

E. Clostridium difficile (most case)


Pseudomembranous colitis ()
Clostridium difficile gram-positive,
spore-forming, anaerobic bacillus
Clindamycin, lincomycin, ampicillin, or

cephalosporin Clostridium difficile


95 5 rare case
Staphylococcus aureus, Salmonella species, Clostridium
perfringens, Yersinia species, Shigella species, Campylobacter
species, cytomegalovirus, Entamoeba histolytica, Listeria
species.
pseudomembranous colitis
pseudomembrane
offensive-smelling diarrhea,
fever, abdominal pain toxic
megacolon
4.

A. cell wall mycolic acid


Mycobacterium obligate aerobe

lipids long
chain fatty acids mycolic acids
(Acid fast)

1. Mycobacterium tuberculosis complex (tubercle bacilli)


(Tuberculosis)

(droplet)
M. tuberculosis, M. africanum M.
bovis ( BCG)
facultative intracellular bacteria



serpentine cord cord
formation

1. (pulmonary tuberculosis)
85%
2. (extrapulmonary
tuberculosis) 15%


military tuberculosis

toxin exotoxin endotoxin

facultative intracellular organism

1. Sulfatides (sulfolipids cell envelope)


phagosome-lysosomal fusion
phagocyte
2. Cord factor (trehalose di-mycolate)
-
serpentine growth
-
neutrophil

- mitochondrial respiration oxidative
phosphorylation
3. Lipoarabinomannan (LAM)
-

- (free radical)


IFN-

4. Mycolic acid

5. Antigen 85 complex mycolyltransferase enzyme


TH1
**Tuberculin (purified
protein derivative, PPD) delayed
hypersensitivity (Type IV) CMI
2. Mycobacteria other than tuberculosis (MOTTS)

Non-tuberculous Mycobacteria (Atypical Mycobacteria)


Runyon grouping
2.1 M.

kansasii Photochromogen pigment

(carotenoids) pigment

2.2 M.

scrofulaceum Scotochromogens pigment

(carotenoids)
2.3 M.

avium-intracellulare Non-chromogens

pigment
3. Mycobacterium leprae
(leprosy Hansens disease) obligate intracellular
pathogen ()
3-5

M. leprae

Histiocyte
Schwan cells
5.

A.


Anthrax

Prevention & control :

1. Disposal of animal carcasses by burning or by deep


burial in lime pits
2. Decontamination of animal products (by autoclave)

3.

Protective clothing & gloves for handling potentially


infected materials

4.

Active immunization of domestic animals with


attenuated vaccine & person with high occupational
risk should be immunized

6.

B. Clostridium sp.
A. Chlamydia sp.

>>

B. Clostridium sp. >> Gram + bacilli


C. Bacterioides sp. >> Gram - bacilli
D. Actinomyces sp. >> Gram + bacilli
E. Pseudomonas sp >> Gram bacilli
Clostridium
sp. Actinomyces sp. ( actinomycosis
Granulomatous ulcer

sulfur granule )
lesion Clostridium

Clostridium perfringens anaerobic bacteria


direct smear
( subterminal
) capsule direct smear
Blood agar
swarming 2
(complete hemolysis)
theta toxin ( )
alpha toxin ()
myonecrosis phospholipase C
Clostridium perfringens
5%

7. ()
8.

C. Streptococcus pyogenes
1

9.

D. heat-stable enterotoxin
A. Botulium toxin

>>



B. Clostidium enterotoxin

>>

C. heat-labile enterotoxin

>>

toxin
D. heat-stable enterotoxin >>

B.cereus S.aereus
E. toxin c.perfingens type A >>
7-15 30
Staphylococcus

aereus

food poisoning:

2-8 .

emetic enterotoxin B. cereus


(G+ bacilli, spore)
heat stable enterotoxin ( heat 100 C, 30 min &
acid in stomach)

Bacillus cereus

Gastroenteritis (food poisoning):


2
-

Emetic form:
heat stable enterotoxin

1-6 (~2 .)
( intoxication)

8-10 . food
poisoning entrotoxin A S. aureus
-

Diarrheal form:


heat-labile enterotoxin
6 . (~9 .) 24
.
heat-labile enterotoxin E. coli

food poisoning entrotoxin Cl. perfringens


type A ()
10.

E. CSF

CSF room temperature


Neisseria meningitidis
11.

A. Vibrio sp.

7 14
2
1. Acute watery diarrhea
enterotoxigenic E. coli (ETEC),
enteropathogenic E. coli (EPEC), enteroaggregative E. coli
(EAEC), Vibrio cholerae
2. Acute invasive diarrhea



Shigella, Salmonella, Campyrobacter, enteroinvasive
E.coli (EIEC), enterohemorrhagic E.coli (EHEC), Clostridium
difficile, Yersinia enterocolitica
Vibrio cholerae

1-5

V. cholerae

(darting movement) Oxidase





: Vibrio parahaemolyticus

NaCI 0.5 - 8%

10 -10
10 12

4 96




2 5

12. B. Salmonella enteriditis


Salmonella Salmonellosis


Salmonella enteriditis


Salmonella

Regulation (EC) No 2160/2003 of the


European Parliament and of the Council of 17 November
2003 on the control of Salmonella and other specified foodborne zoonotic
12 2546
(.. 2003)



Regulation (EC) No 2160/2003 of the
European Parliament and of the Council of 17 November
2003 on the control of Salmonella and other specified food-

borne zoonotic
Salmonella
(primary production)
(zoonoses) EU
(live poultry)
(hatching eggs) (national control
programmes)
Commission Regulation (EC) No
1003/2005 30 June 2005 implementing Regulation (EC)
No 2160/2003 as regards a Community target
salmonella serotypes -
amending Regulation (EC) No
2160/2003 Salmonella

Salmonella 5
Salmonella enteritidis, Salmonella hadar, Salmonella
infantis, Salmonella typhimurium Salmonella vircho

Part Microbiology 2006

A. Norcardia sp.

13.

A. Norcardia sp. modified acid fast

C. Acitinomyces sp. modified acid fast


D. Streptomyces sp. modified acid fast
Norcardia

sp.




Norcardia

sp.

1. Pulmonary nocardiosis: Immunocompromised


host

, necrotzing
pneumonia, lung abscess, granuloma, Cavitation
2.

Cutaneous nocardiosis:
- Mycetoma: , , sulfur granule
-

N. brasiliensis (Modified acid-fast)

Anaerobic actinomycetes: Streptomyces &

Actinomyces (Non acid-fast)


- Lymphocutaneous infection:


- Superficial cutaneous infection:





nd

- 2

cutaneous infection:





14.

B. Clostridium difficile
3

15.

D. hyaluronic acid

Staphylococcus aureus

gram positive cocci

in clusters , Coagulase-positive Catalase-positive


Virulence

factors

1. Antigens
- Capsule
- Adhesins
2. Enzymes
- Coagulase
- Catalase

- Lipase

>>

- Hyaluronidase

>>>>

Hyaluronic

connective tissue


- Staphylokinase
plasmin

>>

plasminogen

fibrin

-lactamase

>>

lactams
3. Toxin

Cytotoxin

Exfoliative toxin

Toxic shock syndrome toxin-1

Enterotoxin

Impetigo:

Folliculitis:

Furuncle:

Carbuncle:
furuncle

Wound infection:

Staphylococcal scalded skin syndrome (SSSS):


Exfoliative toxin



WBC

Bullous Impetigo:
SSSS

16.

D. Streptococcus pneumoniae
A. Klebseilla pneumoniae

>>>>

Gram - bacilli

B. Staphylococcus aureus

>>>>

Gram + cocci in

>>>>

Gram - diplococci

cluster
C. Neisseria meningitidis

D. Streptococcus pneumoniae
diplococci

>>>> Gram +

E. Pseudomonas aeruginosa

>>>> Gram - bacilli

Streptococcus pneumonia

(Gram positive diplococcic)


lancet-shaped

blood agar

(-hemolysis)


(mucoid colony)

optochin ( Viridans streptococci resistant)

bile-soluble

normal flora
5-70

(immunocompromised

host)


lobar pneumonia rusty
sputum ()

(


)

bacterial pneumonia 80-90
17.

B. Gall bladder
Salmonella typhi

Salmonella

typhi


Triple Sugar Iron Agar (TSI)


S. Paratyphi A


TSI


10




endotoxin


18.

C. scabiasis

(Scabiasis)
Sarcoptes



Scabies is a human skin infestation caused by the
penetration of the obligate human parasitic mite Sarcoptes
scabiei var. hominis into the epidermis. After the initial
exposure to the scabies mite, the pruritus and rash may take
up to 6 to 8 weeks to develop. Subsequent exposure to the
mite results in the itching and the rash developing within a
couple days, probably due to prior sensitization to the
scabies mite. The itching is severe and usually worst at
night. The lesions appear as red, scaly, sometimes

crusted

(excoriated) papules and nodules that favor the interdigital


webs sides of the fingers, volar aspects of the wrists and
lateral palms , elbows, axillae, scrotum, penis labia, and the
areolae in women. A diffuse erythematous eruption on the
trunk may be present and represents a hypersensitivity
reaction to mite antigen. The pathognomonic lesion is a
burrow, which is a thin, thread-like, linear structure that is 1

to 10 mm in length, and is a tunnel caused by the


movement of the mite in the stratum corneum. The burrow is
best seen in the interdigital webs, wrists, or elbows.
However, it can be difficult to find in early cases of the
condition, or after the patient has extensively excoriated the
lesions. Identification of a burrow can be facilitated by
rubbing a black felt-tip marker across an affected area. After
the excess ink is wiped away with an alcohol pad, the
burrow appears darker than the surrounding skin due to ink
accumulation in the burrow.
Lice



Pediculosis

Pediculus Humanus Capitis (Head Louse)

Pediculus Humanus Corporis (Body Louse)

Pthirus Pubis (Pubic Louse)

larva migrans

A larval worm, typically a nematode, that

wanders in the host tissues


but does not develop to the adult stage.

Cutaneous larva migrans may cause mild respiratory


symptoms and be associated with transient pulmonary
infiltrates and a peripheral eosinophilia (Lffler's syndrome).
Visceral larva migrans, or toxocariasis, may lead to
granulomatous involvement of the liver, lungs, heart, muscle,
brain, and eyes. Marked eosinophilia, hyperglobulinemia,
pneumonitis with wheezing, recurrent bronchitis, fever, and
tender hepatomegaly frequently occur. Skin lesions may
present as patchy urticaria or erythematous papular eruptions.
19.

C. Exophiala werneckii
septate hyphae

: Exophiala werneckii

Tinea nigra


stratum corneum


KOH preparation

(septate hyphae)

Candida albicans yeastlike fungi


cutaneous

satellite KOH
preparation budding yeast cells
pseudohyphae
epithelial cells
20.

B. lipid solubility

21.

D. Coxsackie B virus

lymphocyte

Coxsackie B virus Enterovirus, Family


Picornaviridae RNA

(Aseptic meningitis)

(Myocarditis)

(pleurodynia Bornholms disease)


22.

B. E.coli


faecal coliform
fecal contamination



Escherichia

Enterobacter, Klebsiella, Citrobacter

faecal coliform ( lactose fermenter)

facultatively-anaerobic, rod-shaped, gram-negative, non-

sporulating bacteria
bile salts oxidase negative,
lactose 48 . 44 0.5C.
23.

D. Lactobacillus species

Lactobacillus non-spore forming gram

positive bacilli ( aerobic bacteria)

pH 5.5-5.8




E. coli


enteric bacteria

24.

A. Escherichia coli

A. E. coli

>>>> Gram - bacilli

B. Neisseria gonorrhea

>>>> Gram - diplococci

C. Chlarmydia trachomatis

>>>>

D. Pseudomonas aeruginosa >>>> Gram bacilli


E. coli




(cystitis),

ascending infection E. coli


25.

D. Chlamydia trachomatis


cervicitis (, inflamed

cervix
mucopurulent discharge)

Neisseria gonorrhoeae (Gram-negative


diplococci), Chlamydia trachomatis (non-staining obligate
intracellular parasite) Herpes simplex virus


: Bacterial vaginosis (BV)

Gardnerella vaginalis ( weakly gram positive


negative thin gram variable bacilli

coccobacilli gram positive


bacilli)

anaerobes

Nonspecific vaginitis, Gardnerella vaginitis,


Amine vaginitis

BV





amine

cadaverine putrescine

BV
Gardnerella vaginalis

BV 3 4

2.
pH 4.5
1.

4.7 - 5.7



pH


10% Potassium hydroxide
3.

whiff test
whiff test
86 - 90

4.

clue cells

wet smear ( PMN)


clue cells epithelial cells Gardnerella
vaginalis gram variable coccobacilli

clue
cells 20 epithelial
cells clue cells
gram stain
clue
cells

BV
anaerobes
lactobacilli BV

1.

Metronidazole

500 2

7 Metronidazole
200
200 2
2
2.

Metronidazole gel 0.75% 5

1 - 2 5
3.

Metronidazole 2
Metronidazole

BV

anaerobes lactobacilli
Metronidazole
24



75 84

Clindamycin
1.

Clindamycin cream 2% 5

7
2.

Clindamycin 300

2 7

Metronidazole
Metronidazole
Metronidazole
3.

Clindamycin ovules 100

3



clindamycin
lactobacilli anaerobes
BV
Metronidazole
BV
- Pelvic
inflammatory disease (PID), postabortal PID, post operative
cuff infections after hysterectomy abnormal cervical
cytology

premature rupture of membranes, preterm labor and delivery,
chorioamnionitis postcesarean endometritis

BV



Gardnerella vaginalis




Gardnerella
vaginalis low grade pathogen

26.

E.

27.

D.

28.

B. Chlamydia trachomatis
Chalmydia trachomatis

oblingate intracellular bacteria





cytoplasmic inclusion


iodine


Chalmydia trachomatis

trachoma (), inclusion conjunctivitis (

), neonatal pneumonia, urethritis, epididymitis,
prostatitis, endometritis, cervicitis, salpingitis
lymphogranuloma venereum
lymphogranuloma venereum lymphogranuloma
inguinale

() 1-4


herpetiform vesicle

2-3 1 2

(inguinal lymphnodes) (venereal bubo)

perirectal lymphnodes


systemic



Treponema pallidum




(inguinal lymph node)


: 2

1. (early syphilis)

1.1 1 (primary syphilis) 9


- 90








12
.


(Chancre)




(inguinal lymph node)




3 8



2
1.2 2 (secondary syphilis)
1 6 8
2








(cervical, epitroclear, inguinal)
VDRL/RPR reactive



1.
(skin rash)



(macule)
(papule)
(pustule)
(papulosquamous)

maculopapular papulosquamous



2.


(condyloma
lata)
3.

(mucous patch)


4. (alopecia)

(moth-eaten alopecia)

(diffuse alopecia)
2





1.3 (latent syphilis)


VDRL RPR
TPHA FTA-ABS reactive 2
early latent syphilis
2 late
latent syphilis
2


VDRL non
reactive TPHA FTA-ABS reactive
2. (late syphilis)
1 3






2.1 (benign late syphilis)


gumma
tissue necrosis
granuloma


2.2

(cardiovascular syphilis)

(aorta)




2.3 (neurosyphilis)


(asymptomatic neurosyphilis)



VDRL FTA-ABS TPHA

reactive

meningovascular syphilis, tabes


dorsalis general paralysis of insane (GPI)

(congenital syphilis)











VDRL reactive VDRL reactive







2

2

30

2

1. (early congenital syphilis)
1




2 3

(prolonged jaundice)


2
(pseudo paralysis) osteochondritis
epiphyseal separation


(nephrotic syndrome)



rhagades

2. (late congenital syphilis)


2

(interstitial keratitis)

(Hutchinsons teeth)







1 3


29.

A. Syphilis
Treponema pallidum (gram negative)

Spirochaetaceae

anaerobe microaerophile

6-14


(darkfieid microscope)
contrast

phase

outer sheath

glycosaminoglycan

42

T.pallidum
subsp. Pallidum

30.

(
syphilis

28)

D. Clostridium perfringens
Clostridium perfringens anaerobic bacteia



Clostridial myonecrosis (gas gangrene)


( )






24-48 .
15-30

:
Pigmented Prevotella, Prevotella melaninogenica
Prevotella intermedia ( Bacteroides
melaninogenicus group) normal oral vaginal florae
predominant anaerobic gram-negative bacilli
(AGNB) respiratory tract infections
aspiration pneumonia, lung abscess, chronic otitis
media, chronic sinusitis, abscesses around the oral cavity,
human bites, paronychia, brain abscesses, and osteomyelitis.

Prevotella bivia Prevotella disiens (


Bacteroides) obstetric gynecologic
infections
31. E. cell
32.

necrosis and exudates formation

B. Staphylococcus aureus,

pneumoniae

E. Streptococcus

Pneumonia


Streptococcus, Staphylococcus,
Pseudomonas, Haemophilus, Chlamydia Mycoplasma,



1. Streptococcus pneumonia

(Gram positive diplococcic)

blood agar
(-hemolysis)


(mucoid colony)
optochin bile-soluble

normal flora
5-70


(immunocompromised host)




lobar pneumonia
rusty sputum ()
(


)

bacterial pneumonia
80-90
2. Staphylococcus aureus







33.

B. Staphylococcus aureus

Staphylococcus aureus (catalase coagulase

positive)
acute endocarditis
(
Staphylococcus epidermidis)
heart murmur () leukocytosis

Staphylococcus epidermidis (catalase positive


coagulase negative)
novobiocin normal

acute endocarditis

prostatic device

: Viridans streptococci

C-carbohydrate capsule antigen (



optochin bile-soluble)

Streptococcus mitis, Streptococcus mutans, Streptococcus
sanguis (-

hemolysis) (non-hemolysis)

34.

D. Streptococcus agalactiae
A. Neisseria

meningitis
CNS

3 young adult
B. Haemophilus influenzae

CNS

6 2
C. Listeria monocytogenes
CNS
renal transplant
D. Streptococcus agalactiae
CNS

E. Streptococcus pneumonae
CNS
2
young adult elderly

CNS 2


E. coli Streptococcus agalactiae
(gram positive cocci) Listeria monocytogenes
()
Streptococcus agalactiae gram positive cocci



blood agar
(-hemolysis) lancefield group b
bacitracin

hippurate camp test


normal flora vagina 15-20%
capsule -hemolysin
neonatal septicemia meningitis


E. coli

Listeria monocytogenes non-spore forming


gram positive coccobacilli facultative intracellular
pathogen listeriosis
1.


2.




granuloma

3.

Part Microbiology 2007


35.

C. 16.00 .
Urinary tract specimens

Transporting the specimen within 2 hours otherwise


o

refrigerating at 4 C
36.

A. Staphylococcus aureus
A. Staphylococcus aureus

>>>> Gram + cocci ,

catalase coagulase positive

B.

Staphylococcus epidermidis

catalase +

>>>> Gram + cocci ,

, coagulase -

C. Staphylococcus hemolyticus
D. Staphylococcus pyogenes
E.

Staphylococcus pneumoniae

C-E Streptococcus sp.

Streptococcus sp.
catalase
Staphylococcus aureus (catalase coagulase
positive)

(osteomyelitis)


metaphysic

37.

A. HSV

Herpes simplex virus (HSV)


2 Herpes simplex virus type 1
(HSV-1)
Herpes simplex virus type 2 (HSV-2)


(localized infection)
(systemic infection)


HSV gingivostomatitis, herpes
labialis, herpes genitalia, herpes keratoconjunctivitis, herpes
encephalitis, neonatal herpes infection

herpes



ganglia




20-40


80






herpes simplex



Primary Infection
Latency and Shedding
Recurring Infections

Primary Infection

2-3

24


2-3



Latency and Shedding



Recurring Infections


5


herpes simplex

herpes simplex
(HSV-1)
(HSV-2)

4-5

oral sex
type 1


HSV-2


herpes simplex

herpes simplex



Herpes Encephalitis Latency

Herpes Meningitis 4-8 primary


genital HSV-2
2-7

steroid
herpes simplex

Tzanck smear
multinucleated giant cells

38.

C. Haemophilus aphrophilus
Haemophilus aphrophilus gram negative

coccobacilli
X factor
( Haemophilus X V factors)
CO2 (
Haemophilus CO2 )

: HACEK organism Gram negative


bacteria
endocarditis

Haemophilus aphrophilus, Haemophilus parainfluenzae


Haemophilus paraphrophilus

Actinobacillus actinomycetemcomitans
Cardiobacterium hominis

Eikenella corrodens

Kingella kingae

All of these organisms are part of the normal oropharyngeal


flora which grow slowly, prefer a carbon dioxideenriched
atmosphere and share an enhanced capacity to produce
endocardial infections, especially in young children.
Collectivelly, they account for 5-10% of cases of infective
endocarditis involving native valves and are the most
common gram-negative cause of endocarditis among people
who do not use IV drugs. Because of their fastidious growth
requirements, they have been a frequent cause of culturenegative endocarditis. Culture negative refers to its inability to
produce a colony on regular agar plates, this is because they
are fastidious (require a specific nutrient).
In addition to valvular infections in the heart, they can also
produce other infections such as bacteremia, abscess,
peritonitis, otitis media, conjunctivitis, pneumonia, arthritis and
osteomyelitis, and periodontal infections.

The treatment of choice for HACEK organisms in endocarditis


is ceftriaxone, a cephalosporin antibiotic.
Enterococcus spp.
facultative anaerobe

group D streptococci ..2527


nucleic acid
Enterococcus

genus


Enterococcus faecalis Enterococcus faecium
Enterococcus spp.
Staphylococcus

spp.

(hospital acquired
community acquired

and

infection)

(infective
endocarditis)


Enterococcus spp.

cephalosporins


Enterococcus spp.


penicillin
ampicillin, aminoglycosides glycopeptides
vancomycin
Enterococcus faecalis
(endocarditis) (fatigue),
(weakness), , ,
(anorexia), (myalgias),
(murmur)

E. faecalis endocarditis
(biliary infection)
(urinary tract) genitourinary tract
manipulation ( )

E. faecalis 6.5%, ph 9.6


gamma-hemolysis (-hemolysis).


penicillin

E. faecalis non-motile microorganism


facultatively anaerobic, ferments glucose
gas, negative catalase test
reduction of litmus milk liquefy
gelatin.
39.

A. Epithelium 10 /HPF

WBC 40/HPF

Sputum specimens with > 25 leukocytes and < 10


squamous epithelial cells per low-powerfield (10x objective)
are ideal.
Adequate specimen
- Less than 10 squamous epithelial cells per low-power
field
- Present of 25 or more PMN per low-power field

Inadequate specimen
- More than 10 squamous epithelial cells per low-power
field
40.

E. HPV
human papillomavirus (HPV) dsDNA

envelope

41.

C. Neisseria meningitides
lab WBC 1200

cell/cu.mm.[] N 90% [] MN 10% Protein 150 g/dl []


Sugar 20 g/dl [] criteria
bact. bact. 2 Neisseria
meningitides S. pneumonia lab
common sense
stiffness of neck
meningitis ( - -)
42.

B.

Proteus mirabilis

Proteus urinary
tract infection ( E. coli) Non-lactose-fermenter
swarming colony
Urease
kidney stones staghorn
renal calculi ( )
43.

A.

Clostridium difficile


Normal flora Clostridium difficile
enterotoxin (toxin A) cytotoxin (toxin B)
mucosa diarrhea psudomembranous
colitis
44.

E. Streptomyces spp.
Keyword

Mycetoma
modified acid fast

modified acid fast

A.

Nocardia asteroids (modified

acid fast) B. Penicillium maneffii () D.


Mycobacterium chelone (modified acid fast)
2 C.

Clostidium

perfringen (.
gas gangrene )
Streptomyces somaliensis
anaerobic actinomycetes

modified acid fast (non-

acid fast bacteria) Nocardia asteroids


modified acid fast
Mycetoma
dermis subcutaneous tissue
granulomatous inflammation
3
1.
2.

3.

(Tumefaction)


(multiple sinus tract)
(grain)

Mycetoma chronic subcutaneous infection


actinomycetes
granulomatous inflammatory response dermis
subcutaneous tissue Mycetoma
grains
multiple sinuses
Mycetoma microaerophilic actinomycetes
actinomycetoma mycetoma true
fungi eumycetoma
45.

D. Bacillus cereus
Keyword

Bacillus cereus
46.

100 CFU/ml
105 CFU/ml in a midstream urine specimen is reliably

associated with infection.

In women with acute dysuria, 100 CFU/ml correlates


with infection (Stamm WE. et al., 1982 N Engl J Med).
Any bacteria found in a specimen from supra-pubic
aspirate may be significant.
47.

A. E. coli
(Nitrite) positive

(
E. coli )
E. coli



(cystitis),
ascending infection
E. coli



48.

A. Bacteriodes fragilis ( 55+ )


Peritonitis 3

1.

Spontaneous bacterial peritonitis E. coli

2.

Secondary bacterial peritonitis E. coli


Bacteriodes fragilis ( anaerobes)

3.

Peritonitis
Staphylococcus sp. S. aureus

49.

C. Neiseria meningitides

50. A. Erysipelothrix fhusiopathiae


Erysipelothrix fhusiopathiae
(pleomorphic)



facultative anaerobe co2 5-10%
blood agar

(-hemolysis) viridians
streptococci





neuraminidase

Part Parasitology 2005

1. D. Gnatostoma spinigerum keyword


2. A.
Taeniasis Trichinosis
(
keyword )
A
Taeniasis solium Taenia solium

cysticercus cellulosae

Trichinosis Trichinella spiralis


( )

sparganosis
Genus spirometra

gnathostomiasis Gnathostoma
spinigerum

Cysticercosis



cysticercus cellulosae
fasciolopsiasis

(metacercaria)

3. D. Entamoeba histolitica
Keyword
RBC



Part Parasitology 2007
4.

A.



) Necator americanus
Ancylostoma duodenale

hypochromic microcytic anemia


filariform

5.

larva

B. albendazol

Trichuris trichiura (?
) albendazole mebendazole,
albendazole
Ascariasis, Trichuriasis, Hookworm

Pinworm ( Praziquantel ,
Metronidazole Amoebiasis,
Trichomoniasis,

Ivermectin

strongyloidiasis onchocerciasis,

amphotericin B

Primary amoebic
meningoencephalitis (PAM) Naegleria fowleri
6.

A. Opisthorchis viverrini

Opisthorchis viverrini

(Metacercaria)


suckers (desquamation)

(hyperplasia)


(cholecystitis)
(cholelithiasis)
(Hepatocarcinoma)
(Cholangiocarcinoma)


-

Enterobius

vermicularis scotch tape


technique


(Immunology)

Part Immunology 2005


1.

3
vaccine antibody response chiken pox

measle

A. neutrophil
B. macrophage
C. T-lymphocyte
D. B- lymphocyte
E. eosinophil
2.

immunoglobulin immuno eletrophoresis

agar
A. adsorption
B. precipitation
C. opsonisation
D. agglutination
E. hemeagglution

3.

25

Ig
A. IgG
B. IgA
C. IgM
D. IgD
E. IgE
4.

25 2

2 Rh
2
A. Rh -

Rh

B. Rh -

Rh +

C. Rh +

Rh

D. Rh -

Rh- Rh+

E. Rh +

Rh- Rh+

5.

30 autoimmune thrombocytopenic purpura

immune complex macrophage Ab


complex
A. Fc
B. Fab
C. CH1domain
D. CH2domain
E. Ringe region
6.

T-cell organ
A. liver
B. spleen
C. lymph node
D. bone marrow
E. thymus gland

7.

renal transplant

graft rejection
A. HLA-A
B. HLA-B
C. HLA-C
D. HLA-DR
E. factor B
8.

(fetus) graft
A. isograft
B. allograft
C. autograft
D. xenograft
E. heterograft

9.

30 2

rheumatoid factor positive rheumatoid


factor antibody
A. complement
B. nuclear antigen
C. IgM
D. cartilage
E. synovial membrane
10.

10 vaccine diphtheria

tetanus- pertussis 5
tetanus toxoid

A. antigen
B. antibody
C. antibody
D.

antibody

E.

11.

antibody

vaccine
A. tetanus toxoid
B. rubella vaccine
C. hepatitis B vaccine
D. rabies vaccine
E. pneumococcal vaccine

12.

30 HIV

3
3 infiltration

Giemsa intracystic body

A. eosinophil
B. neutrophil
C. antibody
D. complement

E. T-lymphocyte
13.

19 booster

6
24 hr.


A. histamine
B. complement
C. Langerhans cell
D. mononuclear cell
E. immunoglobulin M
14.

19 tetanus antitoxoid
booster 6 .
mediator
A. histamine
B. complement

C. IgM
D. mononuclear cell
E. Langerhans cell
15.

3 measle Ab

Ab
A. matrix
B. neucleocapsid
C. hemoflutinin
D. non structural
E. polymerase
16.

17
12
A. tetanus anti-toxin
B. tetanus Ig
C. 2

D. tetaus anti-toxin and adjuvant therapy


E.
17.

20 -thalassemia-HbE disease
1 culture Streptococcus pneumoniae

A. neutrophilicfunction
B. T cell activation
C. complement
D. opsonization

18.

cocci in chain latex brad

A. IgG and fibrinogen


B. factor VII and prothrombin
C. C3b and fibrinogen
19.

A Rh+ fresh frozen plasma

Part Immunology 2006


1.

DPT tetanus toxoid

A.
Ag
B.
Ab
C. Ab
D. Ab
2. 30 1

chest film Lobar pneumonia
right lower lobe culture Strep pneumo



A. Opsonization
B. Complement
C. T cell activation
D. PMN function
E. Antibody production
3.

bone marrow transplantation

cyclosporine

A. Macrophage

B. B-lymphocyte
C. T-lymphocyte
D. neutrophil

4.

A. Immunogenicity
B. Viral associated antigen
C. Unique oncofetal antigen
D. Mutate oncogene product
E. Upregulation MHC class II molecule
5.

20
penicillin
BP 90/60
Wheezing
penicillin

A. hapten
B. antigen
C. adjuvant
D. Immunogen
E. Immunogen regulator

6.

antiserum

type

A. hypersensitivity type I
B. hypersensitivity type II
C. hypersensitivity type III
D. hypersensitivity type IV
7.

30 1

hypersensitivity type
A. I
B. II
C. III
D. IV

8.

15

1 hr

immune
A. IgE Mast cell
B. antigen-antibody complex
C. cytokine macrophage
D. immunoglobulin B- lymphocyte
E. antibody protein dermo-epidermal junction
9.

20

1





A. Cytolysis
B. Spongiosis
C. Acantholysis
D. Ballooning degeneration
E. Liquefaction degeneration
10.

1 O ,

Rh+ A , Rh+


A. IgG
B. T-cell
C. NK-cell
D. Macrophage
E. Complement
11.

X

protein

bacteria


enz. Transpeptidyl ribosome
A. 30s
B. 40s
C. 60s
D. 70s
E. 80s

12. Spherocyte Coomb test negative


Part Immunology 2007
1.


antinuclear ab titer 1:512
diagnosis
A. anti ds DNA ab.
B. SER
C. CD4 T-cell
D. CBC
E. ANA

2.

30 silicone 1

A. Basophil
B. Eosinophil
C. Histiocyte

D. Neutrophil
E. Plasma cell
3.

30 erythrematous plaque,

cell mediator
A. Basophil
B. Eosinophil
C. Macrophage
D. Neutrophil
E. Lymphocyte
4.

4 tuberculin test

A. antibiotic
B. helper T cell
C. B lymphocyte
D. Innate immunity

E. Complement activation
5.

5 nephrotic syndrome
prednisolone 1
vaccine
A. tetanus toxoid
B. varizella vaccine
C. influenza vaccine
D. hepatitis B vaccine
E. Japanese encephalitis vaccine

6.

30 SLE corticosteroid
bacteria infection

A. ingestion
B. attachment
C. chemotaxis

D. degranulation
E. lysosome production

2008
. 50 MCV 120


A. Folate deficiency
B.

B12 deficiency

C. Intrinsic factor deficiency


. WBC IgM IgG IgA

A. B-Cell
B. T-Cell actiuation
C.Opsonization

. 10 Acute Viral Myocarditis

A. Mast cell
B. Neutrophil
C. Eosinophil
D.Lymphocyte
. latex 4
hypersensitivity type
A. I
B. II
C. III
D.IV
. 40
promyeloblast, band 12%, plt 600,000

. 35 renal transplantation

graft rejection

A. Plasma cell
B. T cell
C. B cell
D. Antibody
E. Mast cell
. HBV 1 HBsAg
positive, anti-HBcIgM negative, anti-HBs negative

A. Phagocytosis
B. Mast cell degranulation
C. Cytotoxic T cell
D.
E. Antibody-cytotoxic T cell mediate

.

A. Monocyte
B. Neutrophil
C. Eosinophil
D. Lymphocyte
. granulomatous nonseptate hyphae

A.

Rhizopus spp.
B.Histoplasma capsulatum

. enteric fever
. macrocyte anemia

A. trancobalamin
B. ferrous
C. folic acid
D. intrinsic factor

. gram positive cocci cluster


small gram negative bacilli
. infiltative diffuse

. abscess

A. Bacteroides fragilis
B. Klebsiella pneumoniae
C. Pseudomonas aeruginosa
D.Streptococcus pneumoniae
. intracellulat fission
. giemsa
Cryptospora

stain

.
. aseptum eosiphilic infiltative

. PCR 4 lane autosomal recessive


lane carrier
. (
) (

. CSF lym glucose protein (


)
A. viral infection
B. TB infection
C. parasite infection
.
.
AB
. HLA B 27

Lumbar sacrum

. RS cell

. hydrop fetalis

( )
1. alpha chain
2. beta chain
. WBC 14,000 plt 1,000,000

1. megakaryocyte
2. myeloid cell
. basophilic stripping
. Howell

. plt
. Hb Hct WBC 60,000( N 73%
metamyeloblast 3%)

A. Metamyeloperoxidase
B. Bone marroe examination

Immunology ,

1. A
chicken pox measles
adaptive immunity (plasma cell , memory B-cell
, CTL ) vaccine antibody response
T-cell B-cell
T-cell


neutrophil
Staphylococcus
Streptococcus PMN defective
lysosomal function Chediak Higashi syndrome
pyogenic bacteria


65 (2-5)
polymorphonuclear cell PMN
azurophillic acid hydrolases,
lysozyme, myeloperoxidase, cathepsin, defensin

lysosome
secondary specific granule lactoferrin,
lysozyme, alkaline phosphatase cytochrome b558



chemoattractant

(extracellular pathogen) (intracellular pathogen)
phagosome




(receptor) IgG
IgG
2. B




hydrogen bond, hydrophobic
bond, van der Waals bond ionic bond
monovalent
affinity


precipitation
(
) (zone of equivalent)
Ouchterlony immunodiffusion, immunoelectrophoresis,
radial immudiffusion
particulate insoluble antigen
agglutination blood group typing, Widal
test, hemagglutination, hemagglutination inhibition
3. E
Systemic anaphylaxis type Ianaphylaxis hypersensitivity allergen

crosslink IgE mast cella basophils


wheal and flare


(urticaria)

preformed ( histamine,
tryptase, chymase) newly synthesized mediators (IL-4, IL13, chemokines, leukotrienes)
4. B
Hemolytic disease of the newborn- Rh Incompatibility
Rh ABO
Landsteiner
Rhesus

Rh-positive
C, c, D, d, E, e

C E

D D
Anti-D
Rh-negative
Rh-positive
Rhnegative Rh-positive

hemolytic disease of the newborn

erythroblastosis fetalis
ABO
5. A
Fab fragment of
antigen binding Fc-fragment crystallizable
Fc receptor macrophage Fc R Fc

IgG

ADCC
6 E
alpha-beta T lymphocyte thymus
maturation positive negative
selection ( central tolerance)


lymphoid follicle secondary lymphoid organ

7.
HLA-B HLA-DR polymorphism
class class I (HLA-A, B, C)
typing lymphocytes
class II (HLA-DP, DQ, DR) professional
antigen presenting cells B cells
8. B

exogenous immunosuppressive agents fetus

semi-allograft (allograft = )
isograft = syngeneic graft identical twin, autograft =
xerograft = graft
species
9. C
antiglobulin(anti-immunoglobulin) = IgM , IgG , IgA
IgM (~70%) Fc portion IgG
10.
Secondary immune response
latent
period



cross reaction

11. B rubella vaccine ()


live attenuated vaccine

12. E

(opportunistic infection)


Candida Aspergillus Cryptococcus
neoformans
phagocytes




reactive oxygen species lysozyme



TH1
TH2


Candida
albicans
13. B

Type III hypersensitivity Arthus reaction immune


complex deposit vaccine
booster ( 5 hrs) Ag Ab
vaccine
complement
14. B
Type III hypersensitivity Arthus reaction immune
complex deposit vaccine
booster ( 5 hrs) Ag Ab
vaccine
complement
15. C


HIR CMIR (CMIR infection protection
) mucosal immunity


nucleoprotein
a fourfold rise in antibody titer between acute-phase
and convalescent-phase sera

16. C
tetanus anti-toxin( tetanus Ig) tetanus toxoid
memory
cell
17. D
Streptococcus pneumoniae capsule (anti-phagocytosis)
Ab opsonization Ab
synthesis blood-borneAg
18.
19.
Part Immunology 2006
1.
Secondary immune response
latent
period

cross reaction

2. A

Streptococcus pneumoniae capsule (anti-phagocytosis)


Ab opsonization Ab
synthesis blood-borneAg
3. C

cyclosporine cyclophilins complex block

differentiation activation T cells


calcineurin IL-2 recepter
4. A
Immunogenicity

Tumor antigen Immunogenicity

Tumor antigen

Down regulationMHC I

immuno-suppressive protein

5. A

Hapten


conjugate
6. C






Serum sickness


Arthus
Reaction



7. D

immediate type (type I, II and III hypersensitivity)



sensitized

delayed type (type IV
hypersensitivity)



CD4 MHC
class II
CTL

Jones-Mote Hypersensitivity
24


Contact dermatitis
(chromium, nickel, mercury, atopic

drugs) 2-3

Tuberculin hypersensitivity 2-3


TB, Leishmania, Pneumocystis jiroveci

Granulomatous hypersensitivity
(2-4 )


epithelioid cells giant cells
allograft rejection
GVH reaction

8. A

Systemic anaphylaxis type I-

anaphylaxis hypersensitivity allergen


crosslink IgE mast cella basophils

wheal and flare


(urticaria)

preformed ( histamine,
tryptase, chymase) newly synthesized mediators (IL-4, IL13, chemokines, leukotrienes)
9.
Spongiosis prickle cells


intercellular bridge
Acantholysis keratinocyte

intercellular bridge
Ballooning keratinocyte
10. A


hemolytic disease of the fetus and
the newborn ABO-incompatibility

Rh incompatibility
AB group

(IgG
)
O
anti-A anti-B IgG

A B
IgM
11. A

Part Immunology 2007


1.
2. C
fibrosis Histiocyte = Macrophage tissue
3. A
Systemic anaphylaxis type Ianaphylaxis hypersensitivity allergen
crosslink IgE mast cella basophils

wheal and flare


(urticaria)

preformed ( histamine,
tryptase, chymase) newly synthesized mediators (IL-4, IL13, chemokines, leukotrienes)
4. B
tuberculin test CMI TB tuberculin
test positive X-ray
TB CMI T helper cell

immune response Ab

5. B
varizella vaccine live virus vaccine
6. C

Corticosteroid inflammatory cytokine production, T


cell proliferation, Chemotaxis ( adhesion
molecule), Cytokine secretion Macrophage, Phagocytosis
mononuclear phagocyte
2008
1. C
2. B IgM WBC
hyper IgM syndrome isotype
switching IgM class
T cell
express CD40L

3. D reference Eur Heart J. 2008 September; 29(17):


20732082.
To address the shortcomings, additional immunohistological
and quantitative evaluation of the EMB is required.
Immunohistochemical techniques allow quantification,

identification, and differentiation of inflammatory cells, focusing


on T-lymphocytes.

42

Criteria for immunohistological diagnosis

in the endomyocardial biopsy of inflammatory cardiomyopathy


2

is specified quantitatively as 14 infiltrating leukocytes/mm ,


preferably T-lymphocytes or activated T-cells. If foci of Tlymphocytes are present, active myocarditis can be diagnosed
due to the nature of the infiltrate even when the critical
2

number of 14 leukocytes/mm is not reached, as has been


defined by the task force of the World Heart Federation's
Council on Cardiomyopathies.

43

Infectious causes of myocarditis


Viral
Adenovirus, Arbovirus, Arenavirus, Coxsackie virus, Epstein
Barr virus, Cytomegalovirus, Echovirus, Encephalomyocarditis
virus, Hepatitis B, Human Herpes virus 6, Human
immunodeficiency virus-1, Influenza virus B, Mumps virus,
Parvovirus B19, Poliomyelitis virus, Rabies, Respiratory

syncytial virus, Rubella virus, Rubeola virus, Vaccinia virus,


Varicella virus, Variola virus
Bacterial
Brucellosis, Clostridia, Diphtheria, Francisella, Gonococcus,
Haemophilus, Legionella, Meningococcus, Mycobacterium,
Mycoplasma, Pneumococcus, Psittacosis, Salmonella,
Staphylococcus, Streptococcus, Tropheryma whippleii
Fungal
Actinomyces, Aspergillus, Blastomyces, Candida, Coccidioides,
Cryptococcus, Histoplasma, Nocardia, Sporothrix
Rickettsial
Rocky Mountain spotted fever, Q fever, Scrub typhus, Typhus
Spirochetal
Borrelia, Leptospira, Syphilis
Helminthic
Cysticercus, Echinococcus, Schistosoma, Toxocara, Trichinella

Protozoal
Entamoeba, Leishmania, Trypanosoma, Toxoplasmosis
Eur Heart J. 2008 September; 29(17): 20732082.
Published online 2008 July 9. doi: 10.1093/eurheartj/ehn296.
4. D
delayed type (type IV hypersensitivity)


CD4
MHC class II

CTL
5.
6. B
Chronic graft rejection reaction delayed type (type
IV hypersensitivity)

CD4
MHC class II

CTL

7. C
T cells
+

CD8 T cells ( cytotoxic T cells [CTL] TC)


MHC class I



perforin, granzymes Fas Ligand

carrier hepatitis B 1
anti-HBs virus
viral antigen
CTL
8. C
Eosinophils 5











IgE
IgE





()
9.... Rhizopus spp.
Granulomatous Diseases
Neutrophils usually remove agents that incite acute
inflammatory responses by phagocytosis and digestion.
However, if an agent is indigestible, yet provokes an acute
response, this would lead to a vicious cycle that would
deplete the bodys white count, and cause severe damage to
local normal tissues. The body deals with such indigestible
substances and prolonged inflammatory reactions by forming
granulomas.

The principal cells involved in granulomatous inflammation are


macrophages and lymphocytes. Macrophages are much
longer lived than neutrophils, and can phagocytose an
indigestible agent, thus keeping it from provoking an acute
immune response. Upon phagocytosis of an indigestible
substance, macrophages lose their motility and accumulate at
the site of injury, where they undergo a structural change to
become epithelioid cells. These cells are larger and have
more abundant pale cytoplasm, causing them to resemble
epithelial cells. Nodular collections of these epithelioid cells
form the heart of the granuloma. These cells often become
surrounded by collections of lymphocytes, and contain
multinucleated giant cells, formed by the fusion of up to fifty
macrophages. When the nuclei of these cells is arranged in
a horseshoe pattern, the cell is named a "Langhans Giant
cell". When a pathogen, such as a fungal spore or silica
particle, is found within the giant cell, the cell is named a
"foreign body giant cell". Other inflammatory cells may be
associated with the granuloma as well.
Infectious Etiologies Fungal

Histoplasmata capsulatum
Blastomycosis dermatitidis,
Mucor sp., Rhizopus sp., and Absidia sp. (Mucormycosis)
Aspergillus fumigatus
Candida albicans, Rhinosporidium seeberi
Coccidioides immitis
Cryptococcus neoformans
Infectious Etiologies Parasitic
Leishmaniasis (caused by Leishamania sp.)
Myiasis refers to an infection with the larvae (maggots) of the
common screw-worm fly,
Toxoplasma gondii
Infectious Etiologies Bacterial
Mycobacterium tuberculosis

Nontuberculosis mycobacterial infections (M. kansaii, M.


scrofulaceum, M.avium- intracellularis, M. gordonii, and M.
fortuitum)
Mycobacterium leprae is the organism that causes leprosy
(Hansen's disease)
Cat-Scratch Disease is caused by one of two intracellular,
gram negative bacilli, Rochalimae henselae, or Afipia felis,
which require special staining (Warthin- Starry) for detection.
Actinomycosis is an indolent suppurative infection caused by
an anaerobic or microaerophilic organism.
Syphilis, caused by the spirochete Treponema pallidum
Klebsiella rhinoscleromatis causes rhinoscleroma, which has
three distinct phases: (1) catarrhal stageprolonged purulent
rhinorrhea (honeycombed color), (2) granulomatous stage-characterized by small, nodular masses in the upper airway
which later coalesce, and (3) sclerotic stage--dense fibrosis
that causes stenosis of the nose, larynx, and tracheobronchial
tree.
Neoplastic Etiologies

Eosinophilic granuloma
Hand-Schuller-Christian disease is considered to be a chronic,
disseminated form of histiocytosis X.
Letterer-Siwe is a disease of infants that consists of
hepatosplenomegaly, lymphadenopathy, bleeding diathesis,
anemia, cutaneous lesions, and generalized hyperplasia of
macrophages in a variety of organs.
Benign fibrous histiocytoma most often develops as a
painless mass in sun-exposed and orbital tissues.
Necrotizing sialometaplasia may be found wherever salivary
tissue is present, but is most commonly found in the oral
cavity at the junction of the hard and soft palate.
Polymorphic reticulosis, lethal midline granuloma, and
lymphomatoid granulomatosis, angiocentric lymphoma, are
synonymous terms used previously to describe a condition
that is now believed to be a type of peripheral T-cell
lymphoma (angiocentric lymphoma).
Autoimmune and Vasculitic Disease
Wegener's Granulomatosis is a systemic disease, thought to be autoimmune, characterized by vasculitis
and predominantly epithelioid necrotizing granulomas in the involved tissue.

Systemic lupus erythematosus is a disease in which tissues are damaged by deposition of autoantibodies
and immune complexes.
Sjogrens syndrome afflicts mainly middle aged women (9:1 female to male predilection) and
characterized as an immunologic disorder which produces progressive destruction of the exocrine glands.
Churg-Strauss Syndrome has been described as an allergic angiitis and multi-system granulomatous
vasculitis.
Foreign Body Etiologies
The urate crystals in gout incite granulomatous lesions as they are deposited in the tissues.
Cholesterol granulomas are usually found in the pneumatized area of the temporal bone or the paranasal
sinuses.

10. CD28-dependent activation of CD4 TCR- T


+

cells.
In animals and humans exposed to live Salmonella, cellular
responses are of the Th1 type, as indicated by delayed-type
hypersensitivity (DTH) and by the predominant production of
IL-2 and IFN- upon in vitro restimulation of immune T cells
(Harrison et al., 1997; Sztein et al., 1994).
11....

12.
Gram positive cocci in cluster Staph. aureus
Staph. Coag negative

small gram negative bacilli

Hemophilus Legionella Yersinia gram


negative bacilli Klebsiella

Pseudomonas aeruginosa gram negative bacilli


Nocardia
key word partial acid fast
stain
Anaerobes, either alone or in combination with aerobes,
have been recovered from approximately 80% of lung
abscesses. The anaerobes most frequently isolated are
pigmented Prevotella & Porphyromonas (Gram negative
bacilli), Fusobacterium nucleatum (Gram negative bacilli),
Peptostreptococcus (Gram positive cocci), and
Bacteroides fragilis (Gram negative bacilli).
The major aerobic pathogens that are usually isolated mixed
with anaerobic bacteria are Staphylococcus aureus, Klebsiella
pneumoniae, and Pseudomonas aeruginosa.
gram positive cocci cluster (Staphylococcus aureus)
small gram negative bacilli (Bacteroides fragilis)
13. anti mycoplasma Ab
atypical pneumonia

atypical pneumonia virus


Influenza A, B, RSV, Adenovirus, Parainfluenza
SARS bacteria atypical pneumonia
Legionella, Francisella tularensis
Mycoplasma pneumoniae anti
mycoplasma Ab

14.... A
aspiration pneumonia anaerobes
key word anaerobes
pulmonary abscess
A anaerobes
3
15....

Penicillium marneffei

16....
17.... Candida albicans, Trichomonas vaginalis
18....

Mucormycosis

1.


(
) Woods light
short, fragmented hyphae

round to oval budding

yeast cells (Pityriasis versicolor: Malassezia furfur:


2.5% selenium sulfide shampoo, 1 - 2% zinc
pyrithione shampoo, 2% ketoconazole shampoo, 20%
sodium thiosulfate
2.

40-50% propylene glycol)


(dermatiaceous septate hyphae) ascus
(Blach piedra: Piedra hortae)

3.


yeast cell, chlamydospore branching
septate hyphae (Tinea nigra: Exophiala werneckii)

4.

(Tinea
capitis: Microsporum gypseum, Trichophyton rubrum &
Trichophyton mentagrophytes)

5.


10%KOH branching,
septate hyphae arthroconidia (Dermatophytosis
(Tinea corporis): Microsporum gypseum, Trichophyton
rubrum & Trichophyton mentagrophytes, Epidermophyton
floccosum)

6.



(Dermatophytosis (Tinea pedis): Microsporum gypseum,
Trichophyton rubrum & Trichophyton mentagrophytes,
Epidermophyton floccosum)

7.


(Dermatophytosis (Tinea unguium): Microsporum
gypseum, Trichophyton rubrum & Trichophyton
mentagrophytes, Epidermophyton floccosum)

8.
a.

:
,

, 2

b.

:
,
glans penis, prepuse
(balanitis)

c.

(satellite lesion)


d.



LAB: 10% KOH budding yeast pseudohyphae
(Candida albicans & Candida spp.: Yeast like fungi)
9.

2 cytoplasm macrophage
gian cell (Chromoblastomycosis: Fonsecaea
pedrosoi, F. compacta, Cladosporium carrionii,
Phialophora verrucosa, Rhinocladiella aquaspersa:
Dematiaceous fungi)
10.

(cyst)





(Phaeohyphomycosis: Exophiala jeanselmei, E.
dermatitidis, Cladophialophora bantiana, Alternaria sp.,
Bipolaris sp.: Dematiaceous fungi)
11.



(hyaline branching septate hyphae)
(Hyalohyphomycosis: Scedosporium apiospermum
(Pseudallescheria boydii), Fusarium sp., Penicillium sp.,
Acremonium sp.)

12.

granuloma




septum (aseptate hyphae) inflammatory cells
(Basidioboromycosis: Basidiobolus ranarum)
13.






H&E asteroid body (Lymphocutaneous
sporotrichosis:

Sporothrix schenckii (Dimorphic fungi:

mold form hyaline septate hyphae conidiophore


conidia
))
14.

()

(sinus tract) grain

grain
grain mount 10%KOH mycelium
septate hyphae 2 (Eumycotic

mycetoma: Pseudallescheria boydii, Exophiala jeanselmei,


Madurella mycetomatis, Curvularia geniculata,
Fusarium moniliforme)
Fungi that cause eumycetoma include the following:
Those with a white-to-yellow granule
a.

Acremonium species

b.

Aspergillus nidulans

c.

Aspergillus flavus

d.

Cylindrocarpon cyanescens

e.

Cylindrocarpon destructans

f.

Fusarium species

g.

Neotestudina rosatii

h.

Polycytella hominis

i.

Pseudallescheria boydii

Those with a black granule


j.

Corynespora cassicola

k.

Curvularia species

l.

Exophiala jeanselmei

m.

Leptosphaeria senegalensis

n.

Leptosphaeria tompkinsii

o.

Madurella grisea

p.

Madurella mycetomatis

q.

Phialophora verrucosa

r.

Plenodomus auramii

s.

Pyrenochaeta mackinnonii

t.

Pyrenochaeta romeroi]

15.

grain mount 10%KOH

hyphae 1
(Actinomycotic
mycetoma: Actinomyces Israelii (anaerobe), Actinomadura
madurae (grains ( < 5 .), Actinomadura
pelletieri

(grains ( < 1 .), Nocardia asteroids,

Nocardia brasiliensis

(Partially acid-fast: Grains

( ), Streptomyces somaliensis Grains


( < 2 .) )

16.

(calcification)
(chest X-ray granuloma ) H&E
intracellular yeast

( ) macrophage tissue
histiocyte (Histoplasmosis (Darlings disease):
Histoplasma capsulatum var. capsulatum (Dimorphic
fungi))
17.

(atypical

pneumonia)
(bone marrow, skin, blood and sputum
samples)

Giemsa Wright intracellular fission

yeast histiocytes
pleomorphic yeast with binary fission
(Penicillosis marneffei: Penicillium marneffei (Dimorphic
fungi))
18.

chest X-

ray granuloma


Mucicarmine (Pulmonary
cryptococcosis & cutaneous and mucocutaneous
cryptococcosis: Cryptococcus neoformans)
19.

CSF India ink preparation encapsulated


budding yeast (CNS cryptococcosis: Cryptococcus
neoformans)
20.

neutropenia) (chest X-ray granuloma )


sputum, bronchial washing H&E
(Broad and aseptate hyphae)
(Pulmonary zygomycosis Pulmonary mucormycosis:
Mucor, Rhizopus, Absidia, Cunningghamella)
21.

paranasal sinuses

( neutropenia)
thrombosis, ischemia, necrosis

(Broad and aseptate hyphae) (Rhinocerebral zygomycosis


Rhinocerebral mucormycosis: Mucor, Rhizopus,
Absidia, Cunningghamella)
22.


eosinophils (
fungal ball
, histoplasmosis)

sputum 45
(dichotomously branching septate hyphae) (Allergic
aspergillosis Pulmonary aspergillosis: Aspergillus
fumigatus)



serum LDH (a liver

23.

enzyme) ( 460 U) chest X-ray


bilateral interstitial infiltrates
Giemsa trophozoit Intracystic
bodies (cyst sporozoite 8
) cyst
methenamine silver stain cyst walls
(Pneumocystis jiroveci pneumonia (PCP): Pneumocystis
jirovecii)
Treatment Trimethoprim + sulfamethoxazole

(TMP-SMX)

()
modified acid fast (Pulmonary nocardiosis:
Nocardia asteroids)

19....
20. latex agglutination

21.
A
Pathogen

Virus

Bacteria

Open pressure

Normal

Normal, increased

Appearance

Clear

Turbid

WBC

+ (+)

+++

Lymphocyte

PMN

Protein

++

CSF sugar/ Serum sugar

Normal

Decreased

Others

Special lab

Gram stain, culture, CIE, latex


agglutination

Pathogens

Parasite

Protozoa (Naegleria)

Systemic infection

Open pressure

Normal, increased

Normal, increased

Normal

Appearance

Angio: coconut juice

Turbid

Clear

++

+++

eosinophil

PMN

Lymphocyte

++

++

Gnatho: xanthochrome
WBC

Protein

CSF sugar/ Serum sugar

Normal

Decreased

Normal

Others

Fresh smear, culture -

Pathogen

Tuberculosis

Cryptococcosis

Open pressure

Normal, increased

Normal, increased

Appearance

Straw color

Clear

Xanthochrome
WBC

++

++

Lymphocyte

Lymphocyte

Protein

+++(+)

++

CSF sugar/ Serum sugar

Decreased

Decreased

Others

AFB, culture, cobweb, PCR

India ink, culture, Crypto Ag

22....
23....
hypersensitivity type II
antibody AB
24.... Ankylosing spondylitis (AS) is a chronic, progressive,
connective tissue disorder that is characterized by inflammation of the

joints of the spine (vertebral joints), hipbones, and sacrum (sacroiliac


joints). The condition is one of a group of spondyloarthropathies, including
other inflammatory connective tissue disorders such as rheumatoid arthritis,
Reiter's syndrome, psoriatic arthritis, and the arthritis associated with
inflammatory bowel diseases such as Crohns disease and ulcerative
colitis.
Symptoms of ankylosing spondylitis are similar to those of the more
common rheumatoid arthritis, including pain, swelling, and stiffness in the
affected joints. The difference is that spondylitis primarily affects the
spine, forming bony outgrowths (syndesmophytes) between the vertebrae,
which may fuse vertebrae and lead to total spinal immobility (ankylosis).
Fusion of the vertebrae also can stiffen the rib cage, reducing lung
capacity and function. Constitutional symptoms may develop as the
condition progresses, including anemia, fatigue, loss of appetite, weight
loss, bowel inflammation, and eye inflammation (iritis). The presence and
severity of symptoms varies among individuals.
Diagnosis of ankylosing spondylitis is based on clinical features such as
presence of low back pain and stiffness for more than 3 months, limited
motion in the lumbar spine, and limited chest expansion. AS is termed
primary or idiopathic (of unknown origin) if the disease occurs with

radiographic evidence of inflammation of the sacrum (sacroiliitis) and


without an associated spondyloarthropathy. It is termed secondary if it
occurs in conjunction with psoriasis or chronic inflammatory bowel
disease.
The onset of ankylosing spondylitis usually occurs between ages 16 and
40. Although a single cause for the condition has not been identified,
genetic factors are known to be involved. Human leukocyte antigen B27
(HLA-B27) has been shown to be present in the tissue of a majority of
individuals with ankylosing spondylitis, but may be an association and not
part of the cause of the disease.
Risk: Individuals who test positive for HLA-B27 and are younger
than age 40 are at increased risk for developing ankylosing
spondylitis (Van der Linden). From 90% to 95% of individuals with
the disorder have the HLA-B27 gene (Schaffert). A family history of
the condition also increases risk. Ankylosing spondylitis is 10 to 20
times more common in individuals whose siblings or parents have the
disease ("Ankylosing Spondylitis").
25.... Hodgkin and Reed-Sternberg (H-RS) cells represent
the malignant cell population in classic Hodgkin's disease (HD). By

micromanipulation of single H-RS cells and subsequent amplification


of their rearranged immunoglobulin (Ig) genes, it was demonstrated
that H-RS cells clonally derive from germinal-center B cells in most
cases.
26....
27....
28. Hb aggligation
29....
30....
31....

(Hematology)

Hematology

Part Hematology 2005


1. hydrop fetalis cord blood
Hb Bart genotype
A. thal 0 / thal 0
B. thal 1 / thal 1
C. thal 1 / thal 1
D. thal 1 / thal 2
E. thal 2 / thal 2
2.

HbH disease -thal 1 trait

thalassemia
A. 0
B.

25

C.

50

D. 75
E. 100

3.

6 2

basophilic stippling basophilic stippling


A. thalassemia
B. lead poisoning
C. post splenectomy
D. iron deficiency
E. Henoch Schoenlein purpura
4.

20 2

purpura
Hb 10.5 g/dl, Hct 31 %
WBC 4,500 /cu.mm (N68%,L32% ) platelet count 20,000
/cu.mm
A. female purpura
B. aplastic anemia
C. acute leukemia

D. paroxysmal nocturnal hemolytic anemia


E. autoimmune hemolytic anemia
5.

33


platelet 200,000/cu.mm bleeding time 15 min (2-6) ,APTT

55 sec (22-35 ) ,PT 12 sec (10-13)

A. cryoprecipitate
B. platelet concentration
C. fresh frozen plasma
D. factor VII
E. factor IX
6.

20 2 jaundice

Hb 8 g/dl, Hct 29%, WBC 10,000


3

cells/mm ,PMN 80% ,L 20% ,Platelet 200,000/mm


Peripheral blood smear Heinz body RBC
Heinz body

A. iron containing gene


B. precipitating hemoglobin
C. artifactual aggregating ribosome
D. pathological precipitating ribosome
E. nuclear fragmented containing chromosome
7.

35 2


2 cm lymph node Hb 8
3

g/dl,Hc 24%, reticulocyte 10%,platelet 250,000/mm ,MCV 100


fL blood smear polychromasia,microspherocyte

A. autoimmune
B. vitamin B12
C. hemoglobin
D. RBC
E. lead poisoning

8.

50 2

Hb 7g/dl,Hct 20% MCV 7 recticulocyte 1%


3

WBC 5,000 cells/mm .L30%.PMN 70%

A. renal failure
B.

liver cirrhosis

C.

gastric carcinoma

D. chronic lung disease


9.

6 39.5 C WBC 4,000cells/mm

(PMN, L.E.M. ) atypical lymphocyte 15% ,


3

platelet 20,000/ mm Hb 16 g/dl, Hct 48%


A. acute leukemia
B. dengue hemorrhagic fever
C. immune thrombocytopenic purpura
10.

Howell-Jolly bodies blood smear


A. liver cirrhosis

B. lead poisoning
C. G-6-PD deficiency
Hematology 2006
1.

20
3

2.

30 smear normochromic

normocytic, target cell, howel jolly bodies

A. G6PD def
B. thalassemia
C. post-spleenectomy
3.

WBC, Hb, Hct

LAB ( ecochymosis )
A. coagulogram
B. aPTT

C. BT
D. VCT
4.

50 Hct 23, WBC 3,200 Plt.

70,000 ( )
A. hypersplenism
B. aplastic anemia
C. IDA
D. megaloblastic anemia
E. ATP
5.

60 1

Hb 8 Hct 24%
WBC 6,000 N 65% L 35% Plt 200,000 MCV 90
Reticulocyte1% blood smear rouleaux formation
cell
A. monocyte

B. neutrophil
C. mycloblast
D. plasma cell
E. lymphocyte
6.

20

CBC Hb 10, Hct 30, WBC 6000 , N 65% , L 35%


,platelet 200.000 , MCV 100 , reticulocyte 10%

A. Haptoglobin
B. Indirect Bilirubin
C. Bile
D. Urobilinogen
E. Hemoglobin
7.

30 1

Hemoglobin

A. O2 O2
B. O2
C. CO2
D. O2 tissue
E. CO2 tissue
8.

5 3

coagulogram

A. Heparin
B. sodium citrate
C. EDTA
9.

Hb 12

Hct 36% MCV 60 WBC Plt Hb typing; Hb A2 45%

A. alpha
B. Beta

C. gamma
10.

40

CBC: Hb 7, WBC 3,500(N 30 L 70 ) , platelet


30,000, MCV 92
A. hypersplenism
B. aplastic anemia
C. iron deficiency
D. IPT
11.

25 3
CBC; Hb 10 WBC 7,000 Plt 300,000 Reticulocyte 7%
MCV 92
A. iron def
B. B12 def
C. Hemolytic anemia
D. Hemoglobinopathy

E. Post hemorrahagic anemia


12.

Packed red cell 2 unit


unit 2 200 cc.
Urticaria Urticarial
rash
A. RBC
B. WBC
C. PLT
D. plasma protein
E.

13.

Hct.
, Hb , reticulocyte 12%, Polychromasia,
microspherocyte
A. B12
B. Hb

C. Enz. RBC
D. RBC
E. Ab RBC
14.

30 30 ml
BP
A. ABO Incompatibility
B. Rh Incompatibility
C. Citrate intoxication
D. Bacteria contaminate
E. Fabrile-non-hemolytic-trans

15.

20

2 cm right costal margin CBC: Hb 10 ,


WBC 5,000 Plt 250,000 polychromasia microspherocyte
coomb test negative

A. antibody RBC

B. globin chain
C. RBC
D. enzyme RBC
16.

18

coagulogram PTT 55 PT 12
factor
A. factorI
B. factor II
C. factor VII
D. factor VIII
E. factor X
17.

clotted blood VCT= 10 min


factor
A. II
B. XII

C. X
D. VII
E. V
F. Fibrinogen
18.

15 B thal / HbE

Ferritinl Hepatocyte Brown


Granular pigment complication

A. Hursutism
B. DM
C. Hypergonadal
D. Hypothyroid
E. Hyperpigment
19.

PTT 40 PT 40
A. Vit. K

B. Vit. C
20.

clotted blood serum

electrolyte 06.00 .
9.00. electrolyte (
)
A. Sodium
B. Calcium
C. Chloride
D. Potasium
E. Bicarbonate
21. 6 superficial ecchymosis CBC: Hb
2, WBC 5,000 , N 63%, L 20%, E 12%, M 5% Giant Plt
pale Plt 250,000 lab
a. Bleeding time
b.VCT

c.Clot retraction
d.Plt
Hematology 2007
1. 40 Hct 60% WBC 6,000
platelet 200,000
A.
B.
C.
D.
E.
2.


VDRL 2
. Hb typing
A. alpha 4
B. beta 4

C. gamma 4
D. delta 4
E. zeta 4
3.

50

2 Hb 8
g/dl Hct 24% WBC 6000 N75% L25% platelet 200000 MCV
80 fL reticulocyte 1% creatinine 5

A. Folic acid
B. Erythropoietin
C. Thyroid hormone
D. Cyanocobalamine
E. Colony stimulating factor
4.

Hb 8 HCT

24% platelet 200,000 WBC 6,000 creatinin 5 mg/dl

A. Folic acid
B. Cyanocobalamin
C. Erythropoietin
D. Thyroid hormone
E. Colony stimulating factor
5.

70 7

3 Hct 23 WBC 3200 ( N 30% L 70% ) Plt


70000
A.
B. Autoantibody
C.
D.
E. RBC

6.

B-thalassemia \ HbE disease

HbE homozygote
thalassemia

A. 1
B. 3/4
C. 1/2
D. 1/4
E. 0
7.

60

1 .

A. stercobillin
B. serum albumin
C. urine urobillinogen
D. plasma prothrombin
E. serum direct bilirubin

8.


Hb H

Genotype
A. /-B. -/C. /
D. ot
E. oo
9.

30 30 cc
BP 80/60
A. anaphylaxis
B. citrate intoxication
C. bacterial endotoxic
D. immune complex reaction

E. antibody dependent cytotoxicity


MCQ MD.316 Hematology
1.

50 CBC: Hb 8g/dl , Hct 20 %

a. Iron dextran (50%)


ferrous sulfate (50%)
2.

b. Folic acid

c. EPO

d.

e. cyanocomodulin

CBC Hct MCV RBC


a. Cytoplasm maturation defect

b. Nuclear maturation

defect
c.

. Proliferation defect

d. . Differentiation

defect
3.

10 39.4 C Pharyngitis

cytokine cell
a. Macrophage
cell

e. B- cell

b.

TH 1

c. TH 2

d. cytotoxic T

4.

Biopsy Starry Sky

5.

Aniso poikilo cytosis +3 Inclusion body

positive
a.
6.

Not found Heinz Body


Hb H disease

b. thal / Hb E

Hb typing HB A2 HB F

Genotype
7. MI Aspirin
8.

2 Neonatal jaundice

9.

BAND 3 Defect

10. frushing

a. ABO non match

b.

Rh Abnormal

c. Plasma

Bacteria
11. CBC Prolong PT,PTT,TT
clotting time

and Venous

Factor

12.

LAB


a. Hb
ring sideroblast
13.

b.

Hct

c. MCV

d. Serum ferritin

e.

( sure )

Chemotherapy Neutropenia

14. T lymphocyte
15.

CD 4 Cytoplasmic chain cell

(..B-cell CD4 - -?)


a. Blast (lymphoblast)
d. Immature B cell

b. Pro B cell

c.

Pre B cell

e. Mature B cell

16.

M PHASE

17.

Marker Hodgkin Lymphoma

18.

Sirolimus

19.

Marker Pre T Cell

20.

Hemostasis

21.

Stem Cell

22.

Enhancer Globin chain

23.

IgM

24.

Cell Active

25.

HIV

26.

T cell Bacteria

27.

Ag

Virus

B cell

Cell Cell Cell


28.

Megaloblastic Anemia

RBC

MORPHOLOGY
29.

( )

30.

Which area of lymph node is most likely to defect

Macrophage presenting antigen to T lymphocyte

a. Subcapsular
Medullary
31.

b.

Cortex

c. Paracortex

d.

e. Follicle

B-cell response is most likely to be initiated in which

area of the spleen


b. Follicle

a. PALS
Spleenic cord

e.

c. Marginal zone

d.

Red pulp

32.

33.

Bone marrow Transplantation

34.

Ring sideroblast BM

35.

Chelate

36.

Transplantation

37.

HLA

ABO 5

Solid Organ
38.

Sign

2-3 DPG

39.

Bone Marrow Transplantation

Skin Rash

GI bleeding

40.

41.

Heparin

42.

gene 2 copy

43.
44.

genotype

45.

G6PD

Heinz Body

Ghost Cell

RBC

46.

Congenital Vit K Def

47.

Follicular Hyperplasia Lymph node


a. Toxoplasmosis

Vaccination
48.

b. Dilantin

c. Viral infection

d.

e. Breast Carcinoma

TB

Macrophage

Cytokine Cell

49.

50.

APC

51.

PLT

52.

case Hereditary

Virus Present MHC I

Spherocytosis
53.

Screen test Anemia

54.

Type Thal

55.

Hemolysis

56.

Antigen Back Bone ABO Group

57.

warfarin

Cimitidine
warfarin
a.
d.
58.

Dose

b.

Dose

c. warfarin

CRF

59.

Effect Side Effect

( )
60.

Thalassemic Hemoglobinopathy

61.

factor VIII

62. bone lesion


63.
(Petichiae) small ecchymosis
64.

translocation gene

(9,22)

Philadelphia ]

Hematology

Part Hematology 2005

1. B

hydrop fetalis globin


(--/--)
thal 1 / thal 1
-thal 1(0) globin
(--)
-thal 2(+) globin
(-)
-thal 2
thal 0 (0)
globin (--)
thal 1 (+) globin
( -)
2. C
HbH (--/-)
-thal 1 trait (--/)

genotype 4
(--/--) Hb Barts hydrops fetalis
(--/) -thal 1 trait
(--/-) HbH
(/-) -thal 2 heterozygous
3. B
Thalassemia Inclusion bodies
Lead poisoning Basophilic stippling colicky
pain
Post splenectomy Howell-Jolly Bodies
Iron deficiency Basophilic stippling Lead
poisoning ?
Henoch Schoenlein purpura ?
4. B


Aplastic anemia


stem cell stem cell

5. C
coagulation
factor
lab
Platelet
Bleeding time coagulation factor
aPTT factor XII, XI, IX, VIII
PT factor VII
factor 4
fresh frozen plasma factor
6. B

Heinz body degenerated hemoglobin G6PD


deficiency
7. A
/ RBC

Hb, Hct
Reticulocyte RBC

Platelet
Polychromasia ribosome RBC
mature
Microspherocyte

Hereditary spherocyte, Autoimmune

autoimmune RBC
B12 def. megaloblastic anemia
Hb thalassemia?
RBC spherosis, elliptocytosis

lead poisoning basophilic stippling


8. colon CA

Hb, Hct
MCV RBC
Reticulocyte
WBC
iron def. chronic blood loss
colon CA
Gastric CA associated Pernicious anemia
9. B
Hb Hct plasma
Hct atypical
lymphocyte dengue
hemorrhagic fever

10. choice
?
Howell-Jolly bodies
Megaloblastic anemia, Thalassemia, Splenectomy, Asplenia
liver cirrhosis Heinz bodies
lead poisoning Basophilic stripping
G-6-PD deficiency Heinz bodies
Hematology 2006
1. : hemoglobinuria intravascular hemolysis
free plasma hemoglobin
hemoglobinuria

- Acute glomerulonephritis
- Burns
- Renal cancer
- Malaria
- Paroxysmal nocturnal hemoglobinuria

- Haemolytic uraemic syndrome (HUS)


- Pyelonephritis
- Sickle cell anemia
- Transfusion reactions
- Thrombotic thrombocytopaenic purpura (TTP)
- Tuberculosis of the urinary tract
-G-6 PD def.
2. : C

howel jolly bodies chromosome spindle

(Karyorrhexis)

howel jolly bodies blood smear


asplenia , splenectomy
3. : C
Ecchymosis primary
hemostasis

(platelet ) platelet

endothelium RBC
subcutaneous tissue
Bleeding time assess

platelet function.

partial thromboplastin time (PTT) or activated partial


thromboplastin time (aPTT or APTT) is
efficacy of both

measuring the

intrinsic (factors XII ,XI ,IX,VIII ) and

common coagulation pathway ( factors X , V , II , I )


Prothrombin time (PT) measures of the extrinsic pathway
of coagulation (factors II, V, VII, X and fibrinogen)
4. : B
>> Aplastic anemia (aplastic anemia)

(hypocellularity)

(pancytopenia)

>> Hypersplenism : (rapidly


and prematurelydestroy blood cells.)
enlarged spleen, anemia, leukopenia, or abnormally low
white blood cell counts, or thrombocytopenia, a deficiency of
circulating platelets in the blood
>> IDA Iron Deficiency Anemia
>> Autoimmune Thrombocytopenia Purpura (ATP)
Symptom

- Acute or child form symptoms usually follow a viral


infection

- No early symptoms - especially in the chronic form

- Thrombocytopenia

- Purpura

- Petechiae

5. : D
rouleaux formation
fibrinogen
sialic acid RBC

globulin Multiple
Myeloma, Chronic infection, parasitic infestation Fasciolar
hepatica
Multiple myeloma plasma cells
oncogenes cytokines
cytokine receptor genes tumor suppressor
genes myeloma cell interleukin-6 (IL-6)

2

monoclonal
immunoglobulin (M-protein) light-chain
Bence Jones protein3 Multiple
myeloma


6. B. Indirect Bilirubin
Hb Hct WBC, platelet
reticulocyte
hemolytic anemia
Jaundice extravascular hemolysis
intravascular hemolysis

hemoglobin

(Hemoglobinuria)
A.Haptoglobin alpha-beta dimer

(intravasculas hemolysis) Haptoglobin


B. Indirect Bilirubin

Direct bilirubin

C. Bile ()
D. Urobilinogen
E. Hemoglobin intravascular
hemolysis

Extravascular hemolysis
Intravascular hemolysis
7. D. O2 tissue
tissue O2 Hemoglobin
O2

tissue

The strength with which oxygen binds to hemoglobin is


affected by several factors. In effect these factors shift or
reshape the oxyhemoglobin curve ("the standard curve"). The
standard curve is shifted to the right by an increase in
temperature, 2,3-DPG, or pCO2, or a decrease in pH.
A rightward shift indicates that the hemoglobin under
study has a decreased affinity for oxygen. This makes it
more difficult for hemoglobin to bind to oxygen (requiring a
higher partial pressure of oxygen to achieve the same

oxygen saturation), but it makes it easier for the hemoglobin


to release oxygen bound to it. The effect of this rightward
shift of the curve increases the partial pressure of oxygen in
the tissues when it is most needed, such as during exercise,
or hemorrhagic shock.
In contrast, the curve is shifted to the left by the
opposite of these conditions. This leftward shift indicates that
the hemoglobin under study has an increased affinity for
oxygen so that hemoglobin binds oxygen more easily, but
unloads it more reluctantly.
8. B. sodium citrate
A. heparin
B. sodium citrate coagulogram
PT, PTT, TT, Anti thrombin III, fibrinogen, Protein C,
Protein S
coagulogram
3.8% sodium citrate


2-8
C. EDTA
CBC, . Hemoglobin typing

9. B. Beta
Adult hemoglobin HbA (22) 95-98%
HbA2 (22) 1.5-3.1% HbF (22)0-0.2%
Hb A2 -globin
-globin
10. B. aplastic anemia()
(
) (petechaie
)
lab

Hb

WBC neutrophil relative


lymphocyte increase
platelet
MCV , ,
aplastic anemia

A. hypersplenism
3
1

1.
2. (cytopenia)
1

3.
hematopoietic cells


4.

B. Aplastic anemia

Pathophysiology

Stem cell defect

BM stromal cell defect hematopoietic


growth factor

Causes

radiation, benzene, chloramphenicol, alkylating

agents, viral agents (parvovirus B19, EBV, HIV) , Fanconis


anemia etc.
Diagnosis

hypocellular BM (cellularity < 30%)

Severity
o Severe: 2 of 3 value

Absolute neutrophil count < 500/mm3


(0.5x109/L)
Plt < 20 x109/L
Reticulocyte count < 1% or absolute
reticulocyte < 40x109/L
o Very severe:
As above
Absolute neutrophil count < 200/mm3

1.

(0.2x109/L)

Hematopoietic stem cell transplantation (HPSCT)


stem cell

2.

antithymocyte and
antilymphocyte globulin (ATG/ALG)

3.

: androgen testosterone
derivative, G-CSF GM-CSF

4.

supportive care
(PRC & Platelet concentration)

hematopoietic growth factor


C.

Iron deficiency


1.

peptic ulcer, hiatal hernia, gastritis


NSAID
2.

3.

4.


(Pica)
(Koilonychia Spoon nail) (Angular
Stomatitis)
**
MCV ( Mean
corpuscular volume:MCV 79-93.3 fL) MCV
WBC, platelet

D. ITP (Idiopathic thrombocytopenia purpura)

autoantibody

Hb, WBC
MCV

1. isolated thrombocytopenia
2.

megakaryocyte

3.
4.

SLE
autoimmune

5.

antibody

11. A .Iron deficiency anemia


A .Iron deficiency anemia



( Microcytichypochromic red cells ) Anisocytosis poikilocytosis



Dimorphic picture 2 Microcytichypochromic red cells

Normocytic normochromic red cells

( nucleated red cell )


granulocytic series Megakaryocytic series
B. B 12 deficiency (pernicious anemia)
B 12
DNA

( nuclear cytoplasmic asynchronism or dissociation )


(NRC)
( MCV > 100)

( macro ovalocyte )

5 lobe

( hypersegmented neutrophil

macrocyte )
C. Hemolytic anemia

120 Jaundice

( serum

bilirubin ) reticulocyte

1.

Hereditary

- G6PD deficiency

Hereditary spherocytosis :

spherocytosis polychromasia
coombstest

- Thalassemia
2.

Acquire
-

Autoimmune hemolytic anemia ( AIHA ) :

antibody

Direct coombstest antibody


2

Warm type
37
spherocytosis reticulocytosis NRC

( agglutination )

Cold type

( 37 )


37

D. Hemoglobinopathy

Hemoglobinopathy:

Sickle cell : Hb S carrier


disease form
sickle cell anemia sickle cell
crisis

Target cell

: Hb E ,C target cell

Homozygote 100 %
E. Post hemorrhagic anemia

Post hemorrhagic anemia is a normocytic-normochromic


anemia caused by sudden blood loss in an individual with
normal iron stores. Hemorrhage may be obvious, that is, after
surgery or before a trauma, as in the cases of malignancies
or gastrointestinal disorders. It should be noted that minor
prolonged hemorrhage does not result in classic post
hemorrhagic anemia, but rather in iron deficiency anemia.
Within twenty-four hours of blood loss, there is a
reduction in the number of circulating erythrocytes, which
affects blood volume and its consistency. To compensate,
water and electrolytes from tissues and interstitial spaces are
mobilized to expand plasma volume and accelerate the

formation and development of blood cells in the bone marrow


(hematopoiesis). While this action maintains adequate blood
volume, it decreases the viscosity (thickness) of the blood.
Diluted blood flows faster and more turbulently than normal
blood and can cause heart problems (ventricular dysfunction,
cardiac dilation, and heart valve insufficiency).
12. : D. plasma protein
Pack red cell

oxygen-carrying capacity
Volume expansion
13. : D RBC
A. Vitamin B12 deficiency (pernicious anemia)
( 12 )
( macrocyte )

B. Hb


target cell

C . Enz. RBC
G6PD deficiency

G6PD (Glucose-6Phosphate Dehydrogenase)

Pentose Phosphate Pathway


(PPP.) NADP NADPH
Glutathione reductase
Glutathione peroxidase
(Oxidants) H2O2
G6PD

(Oxidants)
(Hemolysis)
G6PD X-linked
recessive

Acute hemolytic anemia (


)


(Acute
renal failure)
Electrolytes ()
Hyperkalemia ()

CBC

Heinz body


Oxidation



Heinz body

Heinz body
Bite cell Defected spherocyte
CBC


1.


(Fava beans

Broad beans) Vicine, Devicine, Convicine


Isouramil (Oxidants)

2.

(Oxidants)

3.


(Diabetic

ketoacidosis)

4.


Heinze

body

D RBC
!!!!!!!!!
D. RBC





Bilirubin
CBC Hct. , Hb

reticulocyte

E. Ab RBC

( AIHA)

Autoimmune hemolytic anemia

combs test


14. : A. ABO

Incompatibility

Hemolytic transfusion reaction

Immunologic mechanism

Non immunologic mechanism

hemoglobinemia

, hemoglobinuria

ABO Incompatibility DIC


, renal failure
allergic reaction 1 %
A
B. Rh incompatibility

C. Citrate intoxication massive blood transfusion


2,500 ml. 5,000 ml.
24 hr.

D. Bacteria contaminate

DIC shock
15. : C. RBC
13


( )

( Extra-erythropoiesis )
right costal margin

13 coombs test
negative autoimmune RBC

16. D. factor VIII


Extrinsic pathway

Prothrombin time (PT)


coagulation factor
VII , X , V , prothrombin fibrinogen
Intrinsic pathway
Activated partial thromboplastin time
(APTT) ( factor Vlll)
Whole blood clotting time (WBCT) (Venous
clotting time (VCT) )
coagulation factor
XII , XI , IX , VIII , X , V, prothrombin fibrinogen
case PTT 55 ( 30-40 )
PTT

PT 12 ( 10-14 )

PT

case PTT = Intrinsic


pathway = coagulation factor XII , XI ,
IX , VIII , X , V, prothrombin fibrinogen
17. B. XII
case VCT 10 ( 5-15 )

VCT VCT Intrinsic


pathway

coagulation factor factor XII

18. B. DM
thalassemia
(Iron overload)
cytoplasm ferritin (ferritin
hemosiderin
brown
granular pigment) reticuloendothelial
system (hemachromatosis)
-

collagen

formation (hepatic fibrosis)


-

(arrhythmia)

(heart failure)
thalassemia major
-

insulin

hypogonadism

(infertility)
-

pituitary

19. A. Vit. K
case PTT 40 ( 30-40 )
PTT

PT 40 ( 10-14 )

PT

case PT = Entrinsic
pathway = coagulation factor VII , X ,
V , prothrombin fibrinogen
case Vit.K Vit.K
coagulation factor 4 factor
II , VII , IX X
20. D.Potassium


Potassium, Magnesium
Potassium (ICF)
electrolyte serum (
) Potassium

clotted blood


(serology), Biochemistry


( Clotted )






( anticoagulant )




21. Bleeding time

Bleeding time

Ivy method


(vasoconstriction)
(Platelet aggregation)
(Platelet plug)


Venous clotting blood

(Screening test) Coagulation factor


intrinsic pathway


Surface activity

intrinsic
factor
Clot retraction

ATP

ADP
Thrombasthenin




plt 140,000
400,000 d. Bleeding time, VCT, Clot
retraction
Giant plt
plt




ecchymosis


bleeding time

Hematology 2007
1. C.

hematocrit (normal =
40%-54%) Hematocrit
Hct
-

Hct
-
-
-
-
-
-

Carbon monoxide (CO)


tissue Nicotine

(peripheral vasoconstriction)
2. C. gamma 4
Alpha globin gene located on the short arm
16 1globin gene 2globin gene
gene thalassemia1( -thal1) gene
-thalassemia2( -thal2) globin gene
Hb Bart Globin
gene Hb H disease
Thalassemia Death in utero
Hb Barts hydrops fatalis ( -thal1/ -thal1)Hb typing
gamma 4 gene express alpha
globin gene gamma globin gene

globin gene
globin
gamma 4 :

O2 tissue hypoxia

globin gene located on 11


0

globin -Thalassemia
+

globin gene -thalassemia 0

Thal/ -Thal

Preeclampsia
VDRL syphilis ( false negative)

3. B. Erythropoietin
Pathophysiologic classification of anemia
1.

1.1. Hypoproliferation
Iron deficient erythropoiesis
Iron deficiency , Anemia of chronic disorders
Erythropoietin deficiency
Renal disease, Endocrine deficiency
Hypoplastic anemia
Aplastic anemia, Pure red cell aplasia
Infiltration
Leukemia ,Metastatic carcinoma, Myelofibrosis
1.2. Ineffective

Megaloblastic anemia
Vitamin B12 deficiency , Folate deficiency
Microcytic
Sideroblastic anemia
2.
2.1 Inherited hemolytic anemia
2.1.1 Defects in erythrocyte membrane
Hereditary spherocytosis , Hereditary
elliptocytosis
2.1.2 Deficiency of erythrocyte enzymes
Pyruvate kinase defiency ,G6PD deficiency
2.1.3 Defects in globin structure and synthesis
Thalassemia, Hemoglobinopathy
2.2 Acquired hemolytic anemia

2.2.1 Immune hemolytic anemia


Autoimmue hemolytic anemia
Transfusion of incompatible blood
Hemolytic disease of the newborn
2.2.2 Microangiopathic hemolytic anemia
Thrombotic thrombocytopenia purpura
Hemolytic uremic syndrome
Disseminated intravascular coagulation
Prostatic valves and other cardiac abnormality
2.2.3 Infectious agents
Malaria , Clostridial infection
2.2.4 Chemicals, drugs and venoms
2.2.5 Paroxysmal nocturnal hemoglobinuria
2.2.6 Others

Hypersplenism
Hypophosphatemia
Spur cell anemia in liver disease
3.
3.1 External bleeding
3.2 Internal bleeding
Erythropoiesis, erythropoietin (EPO)
regulatory hormone. EPO
committed erythroid progenitor cells
programmed cell death
(apoptosis). EPO
Chronic renal failure (CRF) normocytic
anemia EPO
creatinine ( BUN
anemia CRF) anemia

Folic acid megaloblastic amemia (MCV = 80-95 =


normal)
monocyte

CSF granulocyte
thyroid hormone

4. C. Erythropoietin
C. normal Hb

13 , Hct 33 % Anemia

chronic renal failure

erythropoietin


anemia
A.& B. anemia Folic acid & Cyanocobalamin
Macrocyte (RBC) DNA
E. CSF marrow suppression or failure
3
5. A.
A.

Pancytopenia melena
(chronic blood loss)

C.

Pancytopenia marrow
suppressor e.g. benzene, chroramphenicol

6. C 1/2
Genotype

thal/ Hb E
E

Hb E homozygote = /
E

= /

( / ) >>>> Thalassemia
E

( / ) >>>>Hemoglobinopathy
7. E

serum direct bilirubin


Jaundice bilirubin
() posthepatic
lesion Jaundice direct bilirubin
(conjugated bilirubin)
function
8. A

/--

A. - thalassemia 1 trait (/--):

MCV

( < 80 fl )

Hb type A2 Hb A2

Hb H inclusion body (1:10000)

( microcyte )

B. homozygote - thalassemia 2 (-/-)


- thalassemia 1 trait
9. D. immune complex reaction
A.

anaphylaxis Ag IgE

mast cell mast cell granule


B.

citrate intoxication
(>2500
ml. 5000 ml. 24 hr.

C.

bacterial endotoxic ,
,
hypotension/shock, DIC,

renal failure, ,

,
D.

immune complex reaction

Ab recipient

Ag donor cytokine(IL-1,TNF)

hypothalamus PGE2

Plt.
antibody-dependent cell-mediated cytotoxicity (ADCC)

MCQ MD.316 Hematology


1. a. Iron dextran (50%)


Iron dextran
(ferrous sulfate )
2. a. Cytoplasm maturation defect
3. a. Macrophage
4. ANS

Burkitt Lymphoma

5. b. thal / Hb E
0

6. / Hb A 2
0

/ /


7. Protect PLT Agregation

8. Hemoglobin ( Hemoglobinuria)
9. Anion Exchanger
10. a. ABO non match
11. Factor VIII ( Hemophilia A
prolong & Family History)
12. e. ring sideroblast ( sure )
13. G-CSF (increase production of neutrophils)
14. Ran. ( T T )
15. d. Immature B cell
16. Vincistrine

( 3 )

17. CD 15,3
18. Hyperlipidemia
19. CD2, TCR (Marker Pre T Cell TdT, CD2,
CD3, pre-T receptor)
20. Blood Coagulation

21. Self Renewal


22. HS-40

Chromosome 16

23. D
24. Phagocyte ( Phagocyte
ADCC)ifferentiation (Isotype
switching)
25. Antibody
26. Polysaccharide Bacteria
B cell

27. Cytotoxic T Cell


28. Normochromic

Macrocytic

29. BT ( Bleeding Time ) [


Plt]
30. c.

Paracortex

31. b.

Follicle

32. PLT ABO typing group


33. Monocyte Appearance
34. Porphyrin

Synthesis

35. Deferiprone

( Caution

Deferoxamine )
36. Tissue Typing

37. Blood group


HLA
2
38. Oxygen Tissue
39. GVHD

( APC

Recipient )
40.
41. Anti Thrombin III

42. -thal 2 / -thal 2


43. PLT

sure

44. Bart Hydrop Fetalis genotype

_ _ / _ _

match
/_ _ / _ _
45. Enzyme Defect
46. Phytonadione
47. Toxoplasmosis
48. TH1

(M. tuberculosis Intracellular bacteria

delayed type hypersens.)


49. IgM
50. Protein/Peptide
51. VWF disease
52. Survival

53. CBC ( 80% )


54. Hemato Electrophoresis
55. Serum Hct
56. H antigen
57. b.

Dose

(cimetidine enz.inhibitor

warfarin)
58. EPO
59. ..sole
60. Hb

(+o+)

Cs

61. Cryoprecipitate

( Fresh Frozen Plasma

Cryo )
62. Plasma cell Myeloma
63. Bleeding time (Petichiae Primary
hemostasis
capillary&Plt)
64. Neutrophil

(Pathology)

Part Pathology 2005

1.

56 adenocarcinoma colon

poor

prognosis
A. polypoid growth
B. surface ulceration
C. circumferential growth
D. extention serosa
E. extention muscularis mucosae
2.

50 hypertension catecholamine
CT suprarenal mass
A. benign adrenal cortex mass
B. benign adrenal medullar mass
C. malignant adrenal cortex mass
D. malignant adrenal medullar mass
E. nodular adrenal cortex mass

3.

4 lower quadrant
right lower

quadrant mass. Hyperactive

bowel sounds
A. appendicitis
B. intussusception
C. strangulated hernia
D. Meckels diverticulum
E. Necrotizing enterocolitis
4.

75 stool occult
blood positive biopsy

A. vulvulus
B. diverticular disease
C. adenocarcinoma
D. vilous adenoma

E. polypoid adenoma
5.

25 HCG 8

HCG
A. adrenal adenoma
B. choriocarcinoma
C. pituitary insufficiency
D. ectopic pregnancy
E. second invasive mole

6.

24 upper outer
quadrant right breast


A. fibro adenoma
B. fibrocystic change

C. traumatic fat necrosis


D. breast abscess with scarring
E. infiltrative mammary carcinoma
7.

lymph

node
A. follicular carcinoma
B. papillary carcinoma
C. Hustle
D. medullary carcinoma
E. anaplastic carcinoma
8.

organ tetany
A. pancreas
B. thyroid gland
C. pituitary gland

D. adrenal gland
E. parathyroid gland
9.

40 thyroid gland T4, T3 TSH


antimicrosomal
antibody positive antibody TSH

A. Riedel syndrome
B. Graves disease
C. multinodular goiter
D. subacute thyroiditis
E. Hashimoto thyroiditis
10.

74 colicky

WBC

A. diverticulitis
B. ulcerative colitis

C. CA of sigmoid colon
D. familial adenomatous polyposis
E. villous adenoma of upper rectum
11.

60 40 10

crackle adventitious
sound, bronchial breath sound, dullness on percussion,
increased tactile fremitus
A. pneumothorax
B. pleural effusion
C. acute bronchitis
D. lobar pneumonia
E. pulmonary emphysema
12.

26

tubular, worm-like, freely move,



A. hydrocoele

B. vericocoele
C. spermatocoele
D. indirect hermia
E. ectasia of spermatic duct
13.

53

femeral fracture 4
autopsy petechiae white matter
petechiae
A. septicemia
B. Contracoup injury
C. fat embolization
D. deep venous thrombosis
E.
14.

subdural hematoma

malignancy
A. high mitotic activity

B. distant metastasis
C. nuclear plenomorphism
D. incomplete vacuolization
E. high NC ratio
15. 50
WBC 50,000/cu.mm
PMN 75%
A. gout

B. pseudogout

C. osteoarthritis

D. septic arthritis

E. rheumatoid arthritis
16.

irreversible cellular injury


A. karyorhesis
B. mitochondrial swelling
C. cytoplasmic vacuolization

D. detachment of ribosome
E. cytoplasmic hyaline droplets

17.

20

2 mild erythema cervix mucopurulent


discharge gram intracellular gram negative diplococi

A. endometriosis
B. ectopic pregnancy
C. cervical CA
D. endometrial hyperplasia
E. dysfunction uterine bleeding
18.

40

A. sebaceous gland

B. sweat gland
C. Focal invasive
D. suspensory ligament
E. scar contracture subcutaneous tissue
19.

9 7

2 body
temperature 37.3 C , BP 150/110 mmHg puffy eyelids,
pitting edema urine analysis specific
gravity 1.020, albumin 1+ ,RBC numerous, RBC cast 13

2/LPF, WBC 3-5 cells/mm

A. focal proliferative glomerulonephritis


B. diffuse proliferative glomerulonephritis
C. focal and segmental glomerulonephritis
D. diffuse thickening of glomerula capillary well
E. fusion of foot process of visceral epithelial cell\

20.

40 adult polycystic kidney


A. hepatic cyst
B. hepatic fibrosis

C. esophageal varices
D. renal cell carcinoma
E. cerebral artery aneurysm
21.

50


2 cm right upper lobe

A. lung abscess
B. bronchiectasis
C. pulmonary infarct
D. pulmonary tuberculosis

E. bronchogenic carcinoma

22.

chronic granulomatous disease

A. neutrophil
B. macrophage
C. NK-cell
D. T-cell
E. B-cell
23.

20


A. acanthosis
B. spongiosis
C. ballooning degeneration
D. cytosis

E. liquative degeneration
24.

50

1
markedly icteric sclera

AST 120 U/L ALT


150 U/L alkaline phosphatase 700

U/L total bilirubin

20 mg/dl, direct bilirubin

15 mg/dl
A. bile salt
B. vitamin K
C. globulin
D. cholesterol
E. gamma glutamyltransferase

25.

40 1 T4 2.3

(4.5-10.9) nmol/L, TSH 30

(0.5-4.7) mU/L antithyroglobulin antibody


antimicrosomal antibody titer

A. Graves disease
B. Hashimotos thyroiditis
C. subacute thyroiditis
D. thyroid carcinoma
E. submyxedema
26.

10 2

Kernig sign positive,


stiff neck test positive
cell 300/cu.mm. lymphocyte
100%,protein 240 ,sugar 10 mg/dl (blood sugar 90
mg/dl)
A. viral meningitis

B. bacterial meningitis
C. parasite meningitis
D. tuberculous meningitis
E. carcinomatous meningitis
27.

60

osteophyte spine knee joint 2


A. neoplasm
B. autoimmune
C. degenerative disorder
D. infection
E. crystal deposit
28.

50

sugar negative, protein trace, RBC


numerous, WBC numerous,

bacteria numerous, no cast


A. acute pyelonephritis
B. nephritic syndrome
C. acute cystitis
D. chronic glomerulonephritis
E. acute glomerulonephritis

29.

3
Staphylococcus Scalded skin syndrome

A. exfoliative
B. leukociditis

C. hemolysis
D. hyaluronidase
E. endotoxin-like syndrome
30.

30 fracture of femur

thrombophlebitis

A. splenic infarction
B. renal infarction
C. hepatic infarction
D. cerebral infarction
E. pulmonary infarction
31.

50 mitral valve stenosis

atrial fibrillation 10

600 g ( 250300g ) fibrinoid endocarditis ventricles


A. amyloidosis
B. infective endocarditis
C. rheumatic fever
D. ischemic cardiomyopathy
E. SLE
32.

50

jaundice
increased total bilirubin,
direct bilirubin, AST & ALT
100 U/L, alkaline phosphatase 600 U/L,
urine bilinogen positive
, urine urobilinogen negative

A. alcoholic hepatitis
B. alcoholic cirrhosis

C. hepatocellular carcinoma
D. obstructive jaundice
E. hemolysis
33.

40
2

smear sliding hiatal hernia


A. incarcerated

hernia
B. strangulated hernia
C. esophageal obstruction
D. esophageal perforation
E. gastroesophageal reflux
34.

25 1 UA

protein 3+ , WBC 10-15


3

cells/mm ,RBC 30-40 cells/ mm anti-dsDNA


positive


A. serum sickness
B. rheumatoid arthritis
C. Henoch-Scholein purpura
D. SLE
E. post-streptococcal glomrulonephritis
35.

28

hyaline membrane
disease

A. alveolar infiltration
B. alveolar collapse
C. pulmonary congestion
D. pulmonary artery anomaly
E. airway obstruction

36.

40 pleural effusion

cell count

50

cells/ mm ,L 100% , protein 1.5

U/L (serum protein 7 g/l, serum


LDH 350 U/L ) pleural effusion

A. metastatic cancer
B. rheumatoid arthritis
C. tuberculosis
D. congestive heart failure
E. SLE
37.

50 20

specific.gravity 1.012,sugar 3+
,protein 3+

A. acute pyelonephritis
B. cortical necrosis
C. necrotizing papillitis
D. nodular glomerulosclerosis
E. diffuse proliferative glomerulonephritis
38.

50 2 chest x-ray

apex of left lung biopsy


acidophilic amorphous necrotic
material
lymphocyte, histiocyte, multinucleated giant cell


A. tuberculosis
B. lung abscess

C. large cell carcinoma of lung


D. pneumocystic carinii pneumonia

E. cytomegalic inclusion disease


39.

osteogenic sarcoma long bone

joint

A.
B.
C.
D.

E. microenvironment joint
40.

59

respiratory failure

A. symmetrical atrophy of frontal cortex

B. decreased pigmentation at locus celureus and


substantia nigra
C. granulovascular degeneration and neuritic plaque
with neruofibrillary tangles
D. demyelation of posterior column of spinal cord
E.

degeneration of motor neuron at motor

cortex,anterior horn cell


41.

x-ray markedly dilated loop

of small and large intestine

A. aganglionosis in the rectum


B. atrophy of the colonic mucosa
C. hypertrophic pyloric stenosis
D. Meckels diverticulum
E. multiple polyps in the colon

42.

A. tetralogy of Fallot
B. ventricular septal defect
C. atrial septal defect
D. patent ductus arteriosus
E. coarctation of aorta
43.

60 1 biopsy

metastatic tumor
AFP primary tumor

A. prostatic cancer
B. renal cell carcinoma
C. colorectal adenocarcioma
D. hepatocellar carcinoma
E. aquamous cell carcinoma of lung

44.

35
2 free T4 TSH

A. colloid goiter
B. multinodular goiter
C. follicular adenoma
D. diffuse hyperplasia
E. subacute granulomatous thyroiditis

45.

60 1 biopsy
metastatic tumor -fetoprotein
primary tumor
A. prostatic carcinoma
B. renal cell carcinoma
C. adenocarcinoma of colon
D. hepatocellular carcinoma

E. squamous cell carcinoma of lung


46.

50 colonic polyps

A. retention polyps
B. lymphoid polyps
C. hyperplastic polyps
D. Pertz Jehger syndrome
E. familial adenomatous polyposis
47.

50 sliding
hernia
A. strangulated hernia
B. incarcerated hernia
C. esophageal obstruction
D. carcinoma of stomach

E. esophageal reflux
48.

50 parathyroid gland 8
parathyroid gland 2 .
parotid gland

microscopic examination neoplastic

lesion epithelium and myoepithelium


acini lobule,duct
complication
A. ipsilateral submaxillary salivary gland carcinoma
B. contralateral immune mediated parotitis
C. regional LN metastasis
D. hematogenous spreading to bone or lung
E. local recurrence
49.

50 parotid gland 8
parotid gland single mass
2cm parotid gland microscopy
neoplastic lesion epithelial & myoepithelial cell

acute tubular & duct ,myxoid &


chondroid stroma parotid lesion

A. local recurrence
B. regional lymph node metastasis
C. hematogenous metastasis to bone and lung
D. contralateral immune-mediated parotitis
E. ipsilateral submaxillary salivary gland neoplasm
50.

50 1

investigate
A.
B.
C.
D.

51.

50 3
lobar pneumonia right lower lobe

A. Streptococcus.pneumoniae
B. Klebsiella pneumoniae
C. Mycobacterium pneumoniae
D. Staphylococcus aureus
52.

30
cumulative risk

A.
B.

mammogram

C. PAP smear
D. occult blood
53.

diverticulitis

A. malabsorption
B. carcinoma of colon
54.

acute hepatitis HBV


A. immune

55.

VDRL positive 1:128

Part Pathology 2006


1.


A. lymphatic dilate
B. vascular permeability

C. increase blood flow


D. increase hydrostatic pressure

2.

Irreversible cellular injury


A. karyorrhexis
B. mitochondrial swelling

C. cytoplasmic vacuolization
D. detachment of ribosome
E. cytoplasmic hyaline droplets
3.

liquefactive necrosis

A. brain
B. liver
C. pancreas
D. kidney
E. lung

4.

25 . CT scan
parenchyma cyst

A. hydatid encephalitis
B. Neurocysticerocosis
C. Toxoplasmotic encephalitis
D. Amoebic meningoencephalitis
E. Eosinophilic meningoencephalitis

5.



Na+ 145, Cl - 120

specific gravity 1.001

A. Hypothalamus + par distalis


B. Hypothalamus +par nervosa
C. Par distalis + thalamus
6.

A. Ankylosing
B. Hip translocation

7.

25
bacterial meningitis

A. 4
B. 37

C. lab
D. lab
E. preservative

8.

10

A. Myocarditis
B. Contractive pericarditis
C. sepsis Arthritis
D. Endocarditis fibroelastosis

9.

30 stiff neck
CSF WBC 1,000, L 90% , N 10% blood sugar 90,
protein 60, lymphocyte 60%, neutrophil 20%

A. viral meningitis

B. fungal meningitis
C. bacterial meningitis
D. TB meningitis
E. cranio tumor
10.

Cell count 10,000 N 90 % L10%

30 ( 80) 80
A. N.menigitis
B. H.infiuenza
C. Listeria monocytogenes
D. Streptococcus agalicatiae
E. Streptococcus pneumoniae
11.

12. COPD 2 .
O2

A. Pons
B. Medulla
C. Limbic system
D. Hypothalamus
E. carotid body
13.


A. neural tube
B. anterior neuropore
C. posterior neuropore
D. vertebra + spine
E. Back wall

14.

60 X-ray osteoblastic lesion

femur ,humerus metastatic tumor tumor


15.

De Quervains tenosynovitis
A.
B.
c.
C.
D.

16.

30 fine tremor,

moist skin,enlarge thyroid gland


A. colloid goitor
B. multinodular goitor
C. follicular adenoma

D. diffuse hyperplasia
17.

25 transaction

pituitary ( pituitary stalk) 1 wk


A. hyperaldosteronism
B. hypervassopressin
C. hypergonadotrophin
D. hyperthyroidism
E. hyperprolactin
18.

60 80 kg. 165 cm.


hyperuricemia CT scan Internal
capsule infarct infarct
A. Obesity
B. DM
C. Hyperuricemia
D.

E. Postmenopause
19.


marker
A. CEA
B. alpha-fetoprotein
C. AFP
D. ALT
E. CA-125

20.


A. Thyroglossal duct cyst
B. Lingual thyroid

21.

A. Urachus
B. Merkels diverticulum
22.

loop small
intestine dilate colon

A. rectal aganglion
B. Meckel diverticulum
23.

50 20 (portal
hypertension) Cell

A. regeneration and fibrosis

24.

5 hematuria urine sp.gr.= 1.035


WBC 10-20 cell/HF RBC: numerous with RBC cast sugar:
negative protein 2+
A. endothelial proliferation
B. increase mesangial matrix
C. glomerular sclerosis

D. mesangial interposition
25.

10 , BP
140/90, WBC 20-30 /HPF, protein +2, RBC numerous, red
cell cast 1-2 /HPF, , 3

26.



Lab

A.
B. 4 c
C. 37 c
D. -21 c
E. transportation

27.

50
2 BP 160 /100 BUN 80 creatinin

8 ultrasound Urine

A. RBC cast
B. WBC cast
C. Board cast
D. Uric sediment
E. Oval fat body
F. Waxy cast
28.

40 diarrhea Moderate
dehydration BP 80/60 fluid
BP 110/70 200 mL/24. BUN
50 Cr 5
A. acute tubular necrosis
B. acute interstitial nephritis

29.

35
U/A protein3+, glucose-ve,RBC

A. Lupus nephritis
B. Nodular glomerulosclerosis
C. Membranous proliferative glomerulonephritis
D. Minimal change glomerulonephritis
E. Endocapillary glomerulonephritis
30.

protein 4+, RBC 2-3


cell / HF, WBC 2-4 cell / HF
A. Focal sclerosis
B. Membranous.

31.

3 edema 2 BP 90/160,
WBC 2-4 /HPF, RBC 2-3 /HPF, fine granular
cast 0-2 /LPF, oval fat body 2-3 /HPF protein +4

32.

50 adult polycystic kidney disease

A. Hepatic fibrosis
B. Hepatic hemagioma
C. esophageal varices
D. enal cell carcinoma
E. cerebral artery aneurysm
33.

3
intravenous pyelography 3 cm

A. Wilms tumor
B. Neuroblastoma
C. Polycystic
D. Renal cell carcinoma
D. Transitional cell carcinoma

34.

50 pap
smear malignant squamous cell

A. External os
B. Endocervix
C. Ectoendocervical junction
D. Lower Ectoendocervical junction
E. Posterior part of vagina
35.

45


20
A. cervix
B. ovary
C. vagina
D. vulva
E. endometrium

36.


10 cm

A. teratoma
B. choristoma
C. chondroma
37.

2 2 cm
alfa-fetoprotein ultrasound
solid mass 2 cm neoplasm
A. teratoma
B. seminoma
C. leydig cell tumor
D. yolk salk tumor
E. gonadoblastoma

38.

60 Old
myocardial infarction slide fibrous scar
myocardium
A. Atrophy
B. Hypertrophy
C. Hyperplasia
D. Coagulation necrosis
E. Cloudy swelling

39.

40.

benign malignant
A. metastasis
B. active mitotic
C. pleomorphic
D. pyknotic nucleus
E. encapsulate

41.

osteoblastic reaction X-ray


A. colonic carcinoma
B. Prostatic carcinoma
C. multiple myeloma
D. Blongcogenic carcinoma

Part Pathology 2007


1.

50 10

A. liver
B. stomach
C. pancreases
D. duodenum
E. gall bladder

2.

17
pubic hair blind
vaginal pouch labia majora
ultrasound uterus
A. Turner syndrome
B. Gonadal dysgenesis
C. Adenogenital syndrome
D. Female pseudohermaphrodite
E. Testicular feminizing syndrome

3.

55
transitional cell carcinoma

A.
B.
C.
D.

E.
4.


( NG tube )
A. midgut volvulus
B. duodenal atresia
C. annular pancrease
D. hypertrophic pyrolic stenosis
E. esophageal atresia

5.

30

stiff neck papilledema muscle


weakness
A. brain abscess
B. hydrocephalus
C. Thrombotic stroke

D. acute bacterial meningitis


E. Ruptured cerebral aneurism
6.

35 1
6 cm hard consistency bloody discharge
nipple lymph node 4 cm

A. 6 cm
B.
C. bloody discharge
D. LN
E. Hard consistency

7. resolution
A. ruptured spleen
B. skin ulcer
C. lobar pneumonia

D. bowel gangrene
E. chronic viral hepatitis
8.

35 diffuse
enlargement of thyroid gland, free T4 lab

A. thyroid scan
B. triiodothyroxine
C. thyroxine binding globulin
D. TSH
E. TRH

9.

BP buffalo
hump serum electrolyte
A. hyponatremia
B. hypokalemia

C. hypocalcemia
D. metabolic alkalosis
E. hyperuricemia
10.

acute

appendicitis inflammation
A. serous inflammation
B. organized inflammation
C. fibropurulent inflammation
D. granulomatous inflammation
11.

abscess Gram +ve cocci in

cluster Lymph node pathology


A. sinus histiocytosis
B. follicular hyperplasia
C. parafollicular hyperplasia
D. granulomatous hyperplasia

E. interfollicular hyperplasia

Pathology
Part Pathology 2005
1.

D. extention serosa, surgical staging of

colonic cancer
2. B. suprarenal /adrenal mass
catecholamine NE+E

hypertension

pheochromocytoma adrenal medulla


benign tumor malignant metastsis

3.

B. intussusception

4.

B. diverticular disease, outpouching of mucosa

5.

E. second invasive mole

see gross patho of intuss

( invasive mole 10% choriocarcinoma


2.5%)

6.

D. breast abscess with scarring


( acute

inflammation acute
mastitis breast abscess nipple fissure
bacteria
reparative process fibrosis abscess
cavity )
7.

B. thyroid carcinoma
lymphatic papillary

cancer follicular cancer



hematogeneous

8.

E. parathyroid gland
tetany calcium

parathyroid calcium
9.

E. Hashimoto thyroiditis

antimicrosomal antibody positive Hashimoto thyroiditis

hypothyroid
10.

inadequate data

11.

D. lobar pneumonia
( signs consolidation crackle adventitious

sound, bronchial breath sound, dullness on percussion,


increased tactile fremitus)
12.

B. Varicocele
(a non-tender, twisted mass along the spermatic cord is

felt (it feels like a bag of worms.) The mass may not be
obvious, especially when lying down, hydrocele sac
contains fluid with transillumination test +, indirect hernia

(congenital) ,
spermatocele ectasia of spermatic duct
small cystic accumulation of semen in dilated efferent ducts
or ducts of rete testis smooth soft well circumscribed)

13.

C. fat embolization
fat emboli complication fracture

fat emboli
fat marrow bone marrow
vascular injury
fat emboli blood circulation


(ischemia) petechiae
necrosis (Reference Kumar
V, Abbas AK, Fausto N. Robbins & Cotran Pathological Basis
th

of Disease. 7 Philadelphia: Elsevier Saunders; 2005)


14.

B. distant metastasis, most important

characteristic of malignancy
15.

A. Gout
(cystral) crystal-induced arthritis

urate crystal pseudogout


calcium pyrophosphate crystal rhomboid

polarized light microscopy urate crystal negative


birefingent calcium pyrophosphate positive
birefringent
Reference Kumar V, Abbas AK, Fausto N. Robbins & Cotran
th

Pathological Basis of Disease. 7

Philadelphia: Elsevier

Saunders; 2005
(synovial fluid)
inflammatory noninflammatory disorders

noninflammotory diseases
(viscous) (ampur-colored) WBC count
2000/mL mononuclear cells
inflammatory disorders
WBC count 2,000-50,000/mL
neutrophils

septic arthritis WBC count

50,000/mL neutrophils (
rheumatologist . )

Reference Harris ED, Budd RC, Firestein GS, Genovese MC,


Sergent JS, Ruddy S, et al.

Kelley's Textbook of

th

Rheumatology. 7 ed. Philadelphia: Elsevier Saunders; 2005


16.

A. karyorhexsis

necrosis
reversible cellular injury

17.

B. Ectopic pregnancy
Gonorrhea (intracellular gram negative diplococi)

ascending infection pelvic


inflammatory disease tuboovarian abscess PID
ectopic pregnancy
A. Endometriosis

1) Regurgitation theory
utrine tube

2)Metaplasic differentiation of coelomic epithelium


(endometrium coelomic epithelium )
3) Invasive/lymphatic dissemination theory
endometrium /
C. Cervical CA HPV
type16, 18(high risk)
D. Endometrium hyperplasia
estrogen
Intrinsic estrogen ( polycystic ovarian disease, functioning
granulose cell tumor of ovary)
Extrinsic estrogen (estrogen replacement therapy)
E.

Dysfunction uterine bleeding


1)Anovulatory bleeding
estrogen

2)Luteal phase defect corpus luteum


progesterone

18.

C. Focal invasive
local

lymphatic drainage lymphedema thickening


skin
19.

B. Diffuse proliferative glomerulonephritis


strep hypertension ,

nephritic syndrome , nephritic acute poststrep GN pathology


diffuse proliferative glomerulonephritis
20.

E. Cerebral artery aneurysm

Adult polycystic kidney cerebral artery aneurysm


hepatic cyst fibrosis
infantile/autosomal recessive polycystic
21.

A. Lung abscess

infection 2

acute process
x-ray cavity 2 cm right upper lobe
22.

B. macrophage

chronic

granulomatous disease
immune response

Chronic granulomatous disease


delayed type

hypersensitivity (hypersensitivity type IV)

aggregate of epithelioid cells


granuloma lymphocytes multinucleated giant cells
epithelioid cells activated macrophages
macrophages T helper1 cytokine
IFN-

Reference Kumar V, Abbas AK, Fausto N. Robbins &


Cotran

Pathological

Basis

of

Disease.

th

Philadelphia:

Elsevier Saunders; 2005


23.

B. spongiosis, underlying pathology of vesicle in

acute eczema
24.

B. Vitamin K

Obstructive jaundice fat malabsorption vit K


def ( K )

25.

B. Hashimotos thyroiditis common cause of

acquired hypothyroidism autoimmune


antibody (show positive antithyroglobulin antibody
antimicrosomal antibody ) thyroid gland thyroid
peroxidase,thyroglobulin
hyperthyroidism follicular cell
thyroid gland hypothyroidism
26.

D. tuberculous meningitis CSF protein sugar

lymphocyte
predominate
27.

C. degenerative disorder
(Symmetric) osteophyte

spine (
) Knee joint
osteoarthritis

Osteoarthritis degenerative joint disease (misnomer


itis
inflammatory disease) progressive degeneration of
articular cartilage primary (idiopathic) secondary
OA post-traumatic underlying systemic diseases
degeneration cartilage
(granular) cartilage subchondral
bone eburnation of articular
surface narrowing of joint space osteophyte
Reference Kumar V, Abbas AK, Fausto N. Robbins & Cotran
Pathological

Basis

of

Disease.

th

Philadelphia:

Elsevier

Saunders; 2005
28.

C. Acute cystitis =acute onset , pain at

bladder ,urine show infection+inflammation


no kidney lesion
29.

A. Exfoliative toxin

no cast indicate

Staphylococcus scalded skin syndrome (SSSS)


Staphylococcuc
aureus Exfoliative toxin Exotoxin

30.

E. Pulmonary infarction
Complication of Fracture

Infection

fracture healing process

Delayed union resulting from inadequate immobilization


or poor alignment

Nonunion

bone

Malunion

bone bone

Fat embolism bone marrow

circulate in blood circulation obstruct vessel


respiratory failure

31.

Nerve damage
C. Rheumatic fever

Mitral valve stenosis rheumatic


heart disease
dyspnea, congestive heart failure, arrhythmia
(especially atrial fibrillation) risk endocarditis

Amyloidosis - deposit of proteinaceous substance


between cell
Infective endocarditis - , fungal,
rickettsiae, Chlamydia subacute
fatique, weight loss,
flulike syndrome acute - rapid develop fever, chills,
weakness, glomerulonephritis, hematuria : invasion of
microbe at heart valve or mural endocardium vegetation
Diag : Duke criteria
SLE - Systemic Lupus Erythematosus Endocarditis
(Libman-Sacks Disease) mitral & tricuspid valvulitis :
small vegetation

32.

D. Obstructive jaundice, typical lab findings


Serum alkaline phosphatase Biliary obstruction,

serum aminotransferase 2-10


Jaundice type

Hyperbilirubinemi

Urine

Urine

bilirubin

urobilinogen

Hepatocellular

Conjugated/Unco

jaundice

njugated

Obstructive

Conjugated

Normal /

jaundice
Hemolytic

unconjugated

absent

jaundice

33.

E. gastroesophageal reflux (common complication

of hiatal hernia)
hiatal hernia - sliding hiatal hernia

- paraesophageal hiatal hernia

strangulated
gastroesophageal reflex sliding hiatal hernia
Hiatal hernia bleeding perforate

34.

D. Systemic lupus nephritis


autoimmune disease antibodies nuclear

antigen
anti-double stain, anti-Sm 4 11
clinical criteria

(1997

revised criteria for classification of SLE)


1. Malar rash
2. Discoid rash
3. Photosensitivity
4. Oral ulcer
5. Arthritis
6. Serositis

7.

Renal disorder proteinuria, cellular cast

8.

Neurologic disorder seizure, psychosis

9.

Hematologic disorder hemolytic anemia, leucopenia,


lymphopenia, thrombocytopenia

10. immunologic

disorder anti-ds DNA, anti-Sm and/or

antiphospholipid
11.

antinuclear antibodies

Reference Kumar V, Abbas AK, Fausto N. Robbins & Cotran


Pathological

Basis

of

Disease.

th

Philadelphia:

Elsevier

Saunders; 2005
B. alveolar collapse

35.

prematurity deficiency of surfactant


pneumocytes
type II alveolar collapse
D. Congestive heart failure

36.

transudate effusion [low cell count <2500/


3

mm , lymphocyte 100%,

Protein pleural to serum ratio < 0.5] exudate


effusion
Transudates: increased in hydrostatic pressure or
decreased plasma oncotic pressure

Congestive

heart failure
Hepatic uremia
cirrhosis
Hypoproteinemia (e.g., nephrotic syndrome)
Exudates: increased capillary permeability or decreased
lymphatic resorption
Infections
Tuberculosis
Bacterial pneumonia
Viral or mycoplasma pneumonia
Neoplasms

Bronchogenic carcinoma
Metastatic carcinoma
Lymphoma
Mesothelioma (increased hyaluronate
content of effusion fluid)
Pulmonary infarct (may be associated with
hemorrhagic effusion)
Noninfectious inflammatory disease involving
pleura
Rheumatoid disease (low pleural fluid glucose
in most cases)
Systemic lupus erythematosus (LE cells
occasionally present)
Fluid From Extrapleural sources
Pancreatitis (elevated amylase activity and low
pH)

Urinothorax (elevated creatinine and low pH)

37.

D. nodular glomerulosclerosis

pathology in DM with renal involvement(diabetic


nephropathy) are nodular glmerulosclerosis( is pathognomonic
lesion )and diffuse
38.

glmerulosclerosis

A. tuberculosis
typical caseous

granulomatous inflammation
mycobacterial infection Mycobacterial tuberculosis

apex of lung high oxygen tension


39.

B.

40.

E. degeneration of motor neuron at motor cortex,

anterior horn cell




ascending paralysis Guillian-Barre
syndrome

acute inflammatory demyelination


demyelination anterior
column of spinal cord degeneration of
motor neuron
D lesion anterior column (motor)
posterior column
(sensory)
Reference Kumar V, Abbas AK, Fausto N. Robbins & Cotran
Pathological

Basis

of

Disease.

th

Philadelphia:

Elsevier

Saunders; 2005
41.

A. aganglionosis in the rectum

most compatible

anatomical site and


clinical than other choices
42.

A. tetralogy of Fallot
congenital cyanotic heart disease right

to left shunt

noncyanotic heart
disease left to
right shunt patent ductus arteriosus, atrial and ventricular
septal defect
shunt coarctation of aorta
late cyanosis

43. D. hepatocellar carcinoma (AFP commonly relates


to HCC and nonseminoma testicular CA)
44.

D. Graves disease is primary hyperthyroid (free

T4 TSH )

+ thyrotoxicosis(

2 ) pathology= diffuse hyperplasia


45.

D. hepatocellular carcinoma (see above)

46.

E. familial adenomatous polyposis (see

pathogenesis of CA colon in any patho textbooks) Nonneoplastic polyps - hyperplastic polyps


- harmatomatous polyps PeutzJeghers syndrome,

Cronkhite-

Canada syndrome, Cowden disease


- inflammatory polyps tuberous
sclerosis
- lymphoid polyps
Adenomatous polyps -true neoplastic lesion
familial adenomatous polyposis
47.

E. esophageal reflux (?, incomplete question,

missing hiatal?)
48..
49..
50.

D.

abnormal uterine bleeding postmenopausal


woman endometrial carcinoma

51.

B. Klebsiella pneumoniae
K. pneumoniae

lobar pneumonia Streptococcus


pneumoniae alcoholism middleaged men
52 ..
53. diverticulitis obstruction perforateion
bleeding, peridiverticulitis,
dissect into the immediately adjacent pericolic fat, extension
g pericolic abscess, pelvic and generalized
peritonitis

54. HBV Proliferative phase


HBV infect hepatocytes hepatocye express HBsAg & HBcAg
+


CD 8 T cell infected hepatocyte
Integrative phase viral DNA
Host genome risk of hepatocellualr CA
immune defect mild liver injury
55.

VDRL positve syphilis

Part Pathology 2006


1.

B. vascular permeability

inflammation
2.

injury to capillary endothelium

reversible cellular injury - generalized swelling

of cell and its organs


- blebing of plasma membrane
- detachment of ribosome
- nuclear chromatin clumping

Irreversible cellular injury - swelling & disruption of


lysosome
- large amorphous densities in swollen
of mitochondria
- disruption of cellular membrane
- pyknosis (nuclear condensation)
- karyorrhexis (nuclear fragmentation)
- karyolysis (dissolution of the nuclear)
3.

()

liver

brain ischemic

pancreas kidney lung cell

bacterial infect

ischemic

coagulative necrosis ( acute pancreatitis fat necrosis)


4.

B. Neurocysticerocosis

T. solium in raw meat ->

cysticercosis present with multiple cysts in cerebral cortex


5.

most likely central DI involve posteria

pituitary(par nervosa?) and hypothalamus


6..
7..

8.

A. Myocarditis

9.

A. viral meningitis mild increase WBC, lymphocyte

predominate, no low sugar


10.

A. N.menigitis more common than other choices

CSF show bacterial infection(marked increase wbc with n


predominate
11
12
13

14.

prostatic carcinoma lung

carcinoma
common metastatic
cancer prostatic carcinoma, breast carcinoma,
lung carcinoma carcinoma of kidney

neuroblastoma, Wilms tumor rhabdosarcoma


osteoblastic lesion prostatic carcinoma, lung
carcinoma breast carcinoma
Reference Kumar V, Abbas AK, Fausto N. Robbins & Cotran
Pathological

Basis

of

Disease.

th

Philadelphia:

Elsevier

Saunders; 2005
15.

B. De Quervains tenosynovitis
abduction policis longus extensor

policis bravis typing, gripping objects


wringing clothes ( anatomy
orthopaedics )
Reference Salter RB. 3

rd

Textbook of Disorders and Injuries

of the Musculoskeletal System; 1999


16.

D. diffuse hyperplasia , hes Graves disease

17.

B. hypervassopressin

pituitary stalk is the neural tract to post pituitary regurate


ADH(vasopressin) secretion trauma can cause both
hypo/hyper vasopressin
journal hyperprolactin
18.
19.

A. CEA (tumor marker of CA colon)

20.

A. Thyroglossal duct cyst more common than

Lingual thyroid
21
22. A. rectal aganglion ()
23..
24.

incomplete

question? History of strep infection

+nephritic+nephritic->diffuse proliferative GN

absece of history

may be minimal change GN with focal glomerulosclerosis

25.

History of strep infection,hypertension BP

140/90 +nephritic

RBC numerous, red cell cast

1-2 /HPF +nephrotic protein +2->diffuse proliferative GN


Poststrep GN
26.
27.

F. Waxy cast
chronic renal failure ( waxy

cast?)
28.

A. acute tubular necrosis

with acute renal failure BUN 50 Cr 5 (oliguric


phase 200 mL/24.)
29.

A. Lupus nephritis
malar rash

polyarthritis,

alopecia , renal involvement(nephro+nephri)


30.

Incomplete question, nephrotic syndrome in

children=minimal change , in adult = membranous GN

31.

nephrotic syndrome

3 /HPF protein +4
32.

, protienuria oval fat body 2-

in children=minimal change

E. cerebral artery aneurysm at circle of Willis


ADPKD autosomal dominant

polycystin vascular smooth muscle cerebral artery


aneurysm
33. A. Wilms tumor
This disease

is the most common renal tumor in

children b. Neuroblastoma suprarenal tumor ,d. Renal cell


carcinoma adult, e. Transitional cell carcinoma
adult
34.

D. Lower Ectoendocervical junction


squamo-columnar junction

squamous cell carcinoma


35.

A. cervix

epidemiologic factors invasive squamous


cell carcinoma (Early sexual activity, multiple sexual partners,
cigarette smoking)
36.

A. teratoma
tumor of germ cell origin

tissue elements 2-3 embryonic layers


ectoderm, mesoderm endoderm
37.

D. yolk sac tumor (endodermal sinus tumor)


alfa-fetoprotein

38.

B. hypertrophy
cellular adaptation work load

39.
40.

A. metastasis ()

41.

B. Prostatic carcinoma

osteoblastic lesion prostatic


carcinoma, lung carcinoma breast carcinoma
Part Pathology 2007
1.

C. pancreases
(chronic pancreatitis

2.

fat malbsorption steatorrhea)

E. Testicular feminizing syndrome


complete androgen insensitivity syndrome

genetic disorder xy fetuses


androgen external genitalia female
phenotype blind pouch vagina
uterus, fallopian tubes ovaries testes
abdomen inguinal canal
3.

A.
risk factor of TCC

4.

E. esophageal atresia

(most compatible with clinical of proximal obstruction)


5.

E. Ruptured cerebral aneurism


acute


ICP

subarachnoid space the worst


headache I ever had
Brain abscess headache, lethargy, fever, nuchal
rigidity blood borne
Metastasis
Hydrocephalus communicate (nonobstructive) hydrocephalus
noncomunicate (obstructive) hydrocephalus
ICP headache, papilledema
Thrombotic stroke atherosclerosis
carotid bifurcation,

origin of middle cerebral artery

acute bacterial menigitis , , , stiff neck

6.

D.

7.

C. lobar

LN

pneumonia
Lobar pneumonia

Consolidation of entire lobe

Organism:

Streptococcus pneumoniae (95%) Type 1, 2, 3, 7 or


Klebsiella pneumoniae, Haemophilus influenzae

Four classic stages


Stage of Congestion (Day 1-2)
Gross:
Heavy, boggy, and red

Microscopic:

Vascular engorgement, intra-alveolar fluid


with few neutrophils and numerous
bacteria

Stage of Red Hepatization (Day 2-4)


Gross:

Distinctly red, firm, and airless, liver-like


consistency
Microscopic:

Massive confluent with exudation (RBC,


neutrophils, and fibrin filling in alveolar
spaces)

Stage of Grey Hepatization (Day 4-8)


Gross:
Grayish brown, dry surface
Microscopic:
Disintegration of red cells

Persistence of a fibrinosuppurative exudate

Stage of Resolution (Day 8)


Gross:Brownish red, spongy cut surface
Microscopic:
Progressive enzymatic digestion to produce
a granular, semisolid debris
8.

D. TSH
This case is Graves disease =Low TSH ->primary( high

TSH -.> secondary )

9.

B. hypokalemia
buffalo hump cushings

syndrome
Glucocorticoid has mineralocorticoid-liked property->NA
retension(hypernatremia) and increase K exceretion->
hypokalemia
BP
ACTH Cortisol & Aldosterone hyponatremia
10. A. serous inflammation
Acute appendicitis supperative
purulent inflammation
Characterized by large pus Neutrophil, necrotic
cells & edema fluid
- Serous inflammation - skin blister out pouring of
thin fluid plasma mesothelial - cell secretion
peritoneal, pleural, pericardial cavities

- Fibrous inflammation - in the lining of body


cavities meninges, pericardium & pleura fibrinous
exudates vascular leakage fibrinogen
vascular barrier
- granulomatous inflammation
focal accumulation of activated
Macrophage epithelioid cells TB
11.

B. follicular hyperplasia
A. sinus histiocytosis
Sinus histiocytosis (reticular hyperplasia)
Distention and prominence of the lymphatic
sinusoids
Causes: nonspecific but may be prominent in
lymph nodes draining of cancers e.g. breast
carcinoma
B. follicular hyperplasia
Several different morphologic alterations depending on
the underlying stimuli

Follicular hyperplasia
B-cell response
Germinal center & Marginal zone hyperplasia

Causes: rheumatoid arthritis, toxoplasmosis and


early stage of HIV infection

C. parafollicular hyperplasia
Paracortical lymphoid hyperplasia
T-cell response
Paracortical expansion
Causes: drug-induced reaction (especially
Dilantin), acute viral infection (e.g. infectious
mononucleosis), vaccination
D. granulomatous hyperplasia



- lung abscess

Acute lung abscesses are not well separated from


the surrounding pulmonary parenchyma. They exhibit
abundant polymorphonuclear leukocytes and, depending on

the age of the lesion, variable numbers of macrophages.


Debris from necrotic tissue may be evident. The abscess
surrounded by hemorrhage, fibrin, and inflammatory cells. As
the abscess ages, a fibrous wall forms around the margin.
Lung abscess differ from those elsewhere in their capacity for
spontaneous drainage. The cavity thus formed air, necrotic
debris, and inflammatory exudate, creating a fluid level that is
easily seen radiographically. The lining becomes covered with
regenrating squamous epithelium. Walls of old abscesses may
be lined by ciliated respiratory epithelium, making distinction
from bronchiectasis difficult.
Rubins pathology, 5th edition, p.503.
- Aschoff body
Rheumatic heart disease

- MG thymus

Thymic epithelial cells nicotinic acetylcholine receptor


anti-acetylcholine antibodies
thymus MG
thymic hyperplasia thymoma

Reference Rubin E, editor. 5 Rubins Pathology; 2008


th

- MG Thymus
myasthenia gravis associated with thymoma

(Epidermiology and Medical


biostatistics)

Part Epidemiology and Medical biostatistics 2005


1.

60-70
linear relation
null hypothesis
A.
B.

C. inverse relation
D.

E.
< 0.05
2.

serum potassium
25 2
serum potassium

A. rapid T-test
B. chi-squae test
C. logistic regression
D. pearson correlation
E. variant analysis

3.

200

positive

negative

60

40

20

80

positive

A. 0.3
B. 0.33
C. 0.6
D. 0.67
E. 0.75
4.


cut point -

5.

cerebrovascular disease
ODD ratio

95%Cl

1.8

0.99-2.20

0.4

0.35-0.45

0.9

0.75-2.10

50

2.3

1.55

0,96-3.20
0.44-2.85

protective factor
A.
B.
C.
D.

50

E.
6.

clinical trial

( validity )
A.
B.

C.
D. ( randomization )

E.

100%

Part Epidemiology and Medical biostatistics 2006


1.




cutoff

2.

infarction



A. chi-square test
B. paired t test
3.

1,000

Specificity

A. 60
B. 86
C. 71
D. 92
E. 83
4.

Ratio
A.
B.
C.
D.
E.

5.

50

(
)
type II error
A.

B.
C. p< 0.05 p< 0.01
D.
p < 0.05 p< 0.1
E.
6.

Hct% 60
normal distribution

A. median S.D.
B. mean S.D.
C. median range
D. mean range
E. mode range

7.

.
1,000 100
10
A. cluster sampling
B. simple random sampling

C. systematic random sampling


8.

60 1,000
DM 200

DM
50 incidence
A. 5%
B. 6.25%
C. 20%
D. 25%
E.

9.

50%

case-control aspirin
Odds ratio = 0.7 confident
interval 95% 0.3-1.3
2
confident interval 95%
A. 0.3-1.3
B. 0.3-0.7
C. 0.5-0.9
D. 0.1-1.1
E. 1.3-2.9

10.

4/1000 P-value =

0.9 power = 93%


A. Type I error = 0.07


B. Type II error = 0.07
C. Type I error = 0.1
D. Type II error = 0.1
E. Type I error = 0.9
11.

( )


A. chi-square test
B. unpaired t-test
C. paired t-test
D. correlative regression

Part Epidemiology and Medical biostatistics 2007


1.

70 0.1/
0.08/
0.02/ life expectation 70

A. 1

B. 2
C. 5
D. 8
E. 10
2.

CA breast stage 1,2,3,4


20,40,60,80 ( 4 )
A. Ratio
B. Interval
C. Nominal
D. Ordinal

3.


4 20,35,35,10

A. Interval
B. Ordinal

C. Ratio
D. Nominal
4.

leptospirosis 2548
= 0.4 = 0.02
case fatality rate
A. 0.0002
B. 0.004
C. 0.4
D. 0.02
E. 0.4

(Epidemiology and Medical Biostatistics)


Part Epidemiology and Medical Biostatistics 2005
1.

B.

1. ratio scale

Interval scale
Quatitative data

simple regressive y = mx+c


linear relation
2. Null hypothesis
(y) (x)
2
2.

B. chi-squae test
Serum potassium ratio scale outcome
input
A. rapid T-test
B. chi-squae test +
C. logistic regression

OR(odds ratio)

D. pearson correlation ratio scale

E. variant analysis ANOVA


3.

C. 0.6
Positive predictive value

(60/100) x 100 =60%


= TP/(TP+FP)
Negative predictive value= TN/(TN+FN)
4.

C.

cut-point

2
cut-point
2
(Type 1 Type 2 Error )
5.

B.
95%CI 1
<1
odds ratio

>1 risk factor

=1
<1 protect factor
95% CI 1 95
100 1

odds ratio

=1
6.

D. (randomization)


random error
internal validity
Part Epidemiology and Medical Biostatistics 2006
1.

A.

2.

choice

A. chi-square
B. Paired t test

Attribute risk
3.

B. 86
specificity() = TN/
(TN+FP)=600/700=0.857 0.86

sensitivity()= TP/(TP+FN)=
250/300=0.83
4.

A.

5.

A.

6.

B. mean S.D.

7.

A. cluster sampling

8.

B. 6.25%
Incidence () =

/
= - DM = 1000-200
= 800
Incidence = (50/800) x 100 = 6.25
9.

C. 0.5-0.9

95%CI

point estimate odds ratio 0.7


B
-

95%CI
sinificant 1 D,E

10.

C. Type I error = 0.1

11.

A. chi-square test
ordinal scale
nominal scale
A. chi-square test
B. unpaired t-test
C. paired t-test

ratio scale +

D. correlative regression
Part Epidemiology and Medical Biostatistics 2007

1.

E. 10
life expectation

= 1/mortality = 1/0.1=1/(0.02+0.08)
=10

2.

C. Nominal

3.

C. Ratio

4.


case fatality rate

total case death / total of that case

(Professional skill and Bioethics)

Part Professional skill and Bioethics 2005


1.

10

A. paternalism
B. virtue theory
C. rights theory
D. utilitarianism
E. libertarianism

2.

A. autonomy
B. beneficence
C. confidentiality
D. non-maleficence
E. conflict of interest

3.


A. attitude
B. norm
C. belief
D. faith
E. value

4.

(Mental

health problem)
A. familial institute
B. political institute
C. religious institute
D. economic institute
E. educational institute
5.

16
20 30




A.
B.
C.
D.

E.
6.

A.
B. HIV

C.

D.

E.

Part Professional skill and Bioethics 2006


1.




A. non-maleticence
B. beniticence
C. autonomy
D. justice

2.

15


20
30
A.
B.
C.

D.

E.

3.

DM insulin
. DKA .
A.

B.
C.
D.

4. 50



Cushing
A.

B.
C.
D.

E.
5.

40
20


A.
B.
C.
D.


E.

Part Professional skill and Bioethics 2007


1.

15 3

A. familial institute
B. political institute
C. religious institute

D. Economic institute
E. Education institute
2.

30 5



analysis

A.
B.
C.
D.
E.

3.

A. faith
B. value

C. norms
D. belief
E. attitude

(Professional skill and Bioethics)


Part Ethics 2005
1.

D. utilitarianism
10


Utilitarianism
Paternalism

(') n. ,
, S. . paternalist n.,adj. paternalistic adj.

Beneficence


- teleology


paternalism

Deontology [deon = duty



(
) ()




Teleology [teleo = completion]
utilitarianism





Virtue :


[5]
virtue ethics

(Kollar, Nathan R.)

[1]


3 (3,4)
1. (virtue)
2. (right) (duty)
3. (Utilitarianism)
1.
(virtue)
(3,4)


(Aristotle)
384-322
(Plato)
(Socrates)

practical wisdom

(pleasure)
(Hedonism)
Hedonist

(happiness)

(virtue)

(want)
(need)


(basic need)









temperance (
, ,), courage
()
(foolhardiness)

justice ()

(virtue)
compassion (), honesty
(), integrity (),
fortitude (), humility ()

2.
(right and duty)
(3,4)

(Immanuel Kant .. 1724-1804)
Kant
Enlightenment Kant
,



Kant




Kant
Critique of Pure Reason (3)

Kant

? Kant


(right)

(duty)

Kant
2 good will
good will


good will
good will

Kant
good will



free will


free will

Kant

Kant

Kant

( .to treat human beings as an end, no as a means..) (4)







Kant
(absolue),(unconditional),

(imperaive) categorical
imperative Kant
(happiness) good will

Kant
Titanic


Kant

3. ( Utilitarianism)
Hedonist
(pleasure)

(pain, suffering)
pleasure
Hedonism

pleasure ? pleasure ?




(Jeremy Bentham ..
1748-1832) (utility)

(happiness) (pleasure)

(pain)
(Utilitarianism Bentham


Utilitarianism (John Stuart Mill
..1806-1873) Mill



Mill

Utilitaianism
(consequence) Kant

(motive)
Utilitarianism

Bentham Mill

Utilitaianism


( Environmental ethics)
(5)
2.

A. autonomy



beneficence nonmaleficence
autonomy

Autonomy





informed consent

:Phylos

Axios = Valve

Value in Human conduct

Aestetic Value

in arts

- /,/
-
Normative ethics

Non normative

1. : Ethical

1.Descriptive ethics :

Theories

- What is good life?:

2 ,

- moral judgement :

1.

: Respect for person


- Autonomy ( Auto = self , Nomos = Governance ) =>

- Veracity : =>

- Informed

consent : ( )

- Confidentiality :

Medical
ethics

Traditional

meaning

(utilitarianism)

Mill

-

-

(Deontological
Theory)

(Duty-Based)

-()-

Kant

-
(Right-Based)

Moral problem

:

,

- 2 :

dilemma

-
Dilemma

-------------

-----------------

-------------

-------------

-----------------

-----------------

1.

Dilemma

2.

No

vote

key person

3.

( Health

Care service)

1.

-

2. ( Paradigm

-

(Engineering

Model)

(contractual
(Paternalitic Model)
-

Model)
-

-=

-


(Deliberative

-


Model)


-
-

()

)
3.

( Autonomy )

-
-

4.

- Respect for person

- Benificence
- Justice

5.
< humanized Medicine >

6.

()

-
-

-



(well-educated)


1.
2.
3.

1.

2.

1.

2.

+++

()

()

3.

3.

4. :

5. :

6.

Relation

care


Trust =

Empathy
=

Rapport =

Accountability

ReEvaluation

Professional

Healt
Empovement h
=

Belief
Insight(

Evidence

Treatment =

()

Collaboration =

Culture

Action

1. Health care service

2.

1.

3.

()

compassion

4.

2. =

5.

competence

6.
7.

( morals,mores) =

(Ethical vules)

1)

moral

()

2)

3)

(Humanity)

code ofcethics

Learning of Medicine May Be Likened Of Growth Of Plants


1.

2.

Our natural ability is the soil


The words of our teacher


3.

Learning from childhood

The seed

The prepared ground

The nutriment

4.

The place of instruction

5.

Deligence is the working of the soil

6.

Time strengthens all these things.



2
-


Hedonism
- (The sophists)
: -

Non- Hedonism

-(Epicureanism)
:

-(The Platonism)
-(The cynicism)
-(The stoicism)

(Humanism)
:

-
-
-
-
-
Ethics

:
2. :
3. :
4. :
()
1.

o
o
o

-

1.
2.
3.

Science
Fact

Ethics

What, How, Must?

/?
Should?


Pattern

Pattern


()
Anatomy of an ethical dilemma (
)
Dicision
Theory
Databas
e

Dimention
Ethical
Dilemma
Situation
Argument

Ethical theories


VS
Old Paradism
Beneficence
Paternalism
Traditional problem solving
process
Solution

New Paradism
Beneficence
Patien-autonomy
Ethical dilemma

Must

Hostitic view
Integrated

Should

1.
2.
3.
4.
5.
6.
7.
8.
9.

10.

: (
? )
:
: Should/Shouldnt
?
Ans.



(
TU 110/120 )
John Locke :
social contract()

2. Jean Jacque-Russo:

3. Thomas Hobbes:


1.

-
-

-
-

-
-
-

informed consent
Respect for person

Autonomy
3.

B. norm
Norm (

)
E.value ( ) norm

Norm

1.( 6) .
2.( 7 +8

.
4.

A. familial institute
9

5.

B.
2005 51

2006

18

(
vital sign )
10





10.

informed consent



...

informed consent



1. Implied & Explicit Consent
implied consent




explicit consent
2. Non-Documented & Documented Consent
consent
doctor-patient communication
consent form
informed consent



informed consent autonomy


...
6.

D.

criteria 4

A C
B

1.

2.




3.



4.


5.

6.

7.

8.

9.



10.

Part Ethics 2006

1.

C. autonomy
79

2.

E.
191. 2005 51.

3.

2005

2006

D.



D.
4.

D.

5.

A.
D.

C.
Transtheoretical model of health behavior

change Prochaska JO.

Robert E. Rakel. Textbook of Family


th

Practice.6 ed.2002

chapter 16 pp.253-261.

Part Ethics 2007


1.

A. familial institute



key
B D E

2.

E.

B. D.
3.

C. norms
B

Part Ethic Exercise for National Test Step I



1.


personality trait
A. Histrionic
B. Dependent
C. Narcissistic
D. Borderline
E. Schizotypal
2.





A. Stigma
B. Mistrusted
C. Unfamilial

D. Experience
E. Lack of privacy
3.

Kubler Ross drief stages
A. Denial
B. Despair
C. Anger
D. Depression
E. Bargain
4. 70 4
6


A.
B.

C.
D.
E.

1. A Histrionic personality disorder


Symptom: A pervasive pattern of excessive emotionality and
attention seeking, beginning by early adulthood and present
in a variety of contexts, as indicated by five (or more) of the
following:
1. in uncomfortable in situations in which he or she is
not the center of attention
2. interaction with others is often characterized by
inappropriate sexually seductive or provocative behavior
3. displays rapidly shifting and shallow expression of
emotion

4. consistently uses physical appearance to draw


attention to self
5. has a style of speech that is excessively
impressionistic and lacking in detail
6. shows self-dramatization, theatricality, and exaggerated
of emotion
7. is suggestible, i.e., easily influenced by others or
circumstances
8. considers relationships to be more intimate than they
actually are


Personality disorders

Early adulthood

Cluster
A


Paranoid, Schizoid, Schizotypal

(Mad)
B

Antisocial, Borderline, Histrionic,



Narcissistic
(Bad)

Avoidant, Dependent, Obsessive (Sad)


compulsive
Paranoid personality disorder





projection

Trust Mistrust

Schizoid personality disorder





,


(Schizoid fantasy)

Schizotypal personality disorder









Antisocial personality disorder
( )



Borderline personality disorder




(All good)
(All bad)


identity








Histrionic personality disorder


(dramatize)




(Reassurance)
Narcissistic personality disorder











Avoidant personality disorder







Dependent personality disorder








Obsessive compulsive personality disorder

(perfectionist)< Narcissistic
>

(inflexibility)









2. B. Experience


3. A. Denial
Grief reaction
Denial



Despair
Anger
Depression


The Kbler-Ross grief cycle
Background
For many years, people with terminal illnesses were an
embarrassment for doctors. Someone who could not be
cured was evidence of the doctors' fallibility, and as a result
the doctors regularly shunned the dying with the excuse that
there was nothing more that could be done (and that there
was plenty of other demand on the doctors' time).

Elizabeth Kbler-Ross was a doctor in Switzerland who railed


against this unkindness and spent a lot of time with dying
people, both comforting and studying them. She wrote a
book, called 'On Death and Dying' which included a cycle of
emotional states that is often referred to (but not exclusively
called) the Grief Cycle.
In the ensuing years, it was noticed that this emotional cycle
was not exclusive just to the terminally ill, but also other
people who were affected by bad news, such as losing their
jobs or otherwise being negatively affected by change. The
important factor is not that the change is good or bad, but
that they perceive it as a significantly negative event.
The Grief Cycle
The Grief Cycle can be shown as in the chart below,
indicating the roller-coaster ride of activity and passivity as
the person wriggles and turns in their desperate efforts to
avoid the change.

The initial state before the cycle is received is stable, at


least in terms of the subsequent reaction on hearing the bad
news. Compared with the ups and downs to come, even if
there is some variation, this is indeed a stable state.
And then, into the calm of this relative paradise, a bombshell
bursts...

Shock stage: Initial paralysis at hearing the bad

news.
Symptoms
The first reaction on hearing the bad news is one of classic
shock. This initially may appear as if there is no reaction at
all to the news. The person may nod and accept the news
without appearing to be troubled by it. Inside, they have

frozen out the news that has not really taken hold yet. To
get the news through, they may need to be told several
times.
This is followed by a more external shock, where there may
be physical reactions such as paling of the skin, shortness
of breath and physical freezing.
Treatment
When shock occurs, they may need to be sat down (to stop
them falling) and given a drink of some sort (mostly as
something to hold onto). Show them sympathy and
acceptance. If the shock is not a short-lived one, help them
get to a place where they can sit safely and let the news
sink in.
You can pre-empt shock symptoms to some extent by taking
the person to a place where they can safely hear the news.
Especially if they are likely to move swiftly into more emotive
stages, they will need to be in a private place, away from

the embarrassment of public tears, and in the company of


trusted friends or family.

Denial stage: Trying to avoid the inevitable.

Symptoms
After the initial shock has worn off, the next stage is usually
one of classic denial, where they pretend that the news has
not been given. They effectively close their eyes to any
evidence and pretend that nothing has happened.
Typically, they will continue their life as if nothing has
happened. In the workplace, they will carry on doing their
job even if that job is no longer required.
A classic behavior here is a 'flight into health', where
previously-perceived problems are suddenly seen as having
miraculously fixed themselves.
Treatment
You can move a person out of denial by deliberately
provoking them to anger. Hold up the future (sympathetically)

so they cannot avoid or deny it. Tell them that it is not fair.
Show anger yourself (thus legitimizing that they get angry).
This, to some extent, is done on daytime TV shows where
people in precarious situations are prodded into emotional
explosions that make good TV and (where sympathetically
done) may even be good for them.

Anger stage: Frustrated outpouring of bottled-up

emotion.
Symptoms
The next step after denial is a sudden swing into anger,
which often occurs in an explosion of emotion, where the
bottled-up feelings of the previous stages are expulsed in a
huge outpouring of grief. Whoever is in the way is likely to
be blamed. In a company this includes the managers, peers,
shareholders customers and suppliers. The phrase 'Why me?'
may be repeated in an endless loop in their heads. A part
of this anger thus is 'Why not you?', which fuels their anger

at the those who are not affected, or perhaps not as


seriously so.
Treatment
When they are angry, the best thing you can do is give
them space, allowing them to rail and bellow. The more the
storm blows, the sooner it will blow itself out.
Where anger becomes destructive then it must be addressed
directly. As necessary, you may need to remind people of
appropriate and inappropriate behavior. Reframe their anger
into useful channels, such as problem areas and ways to
move foreword.
Beware, when faced with anger, of it becoming an argument
where you may push them back into denial or cause later
problems. Support their anger. Accept it. Let them be angry
at you.

Bargaining stage: Seeking in vain for a way out.

Symptoms

After the fires of anger have been blow out, the next stage
is a desperate round of bargaining, seeking ways to avoid
having the bad thing happen. Bargaining is thus a vain
expression of hope that the bad news is reversible.
Bargaining in illness includes seeking alternative therapies
and experimental drugs. In organizations, it includes offering
to work for less money (or even none!), offering to do
alternative work or be demoted down the hierarchy. One's
loyalties, debts and dependants may be paraded as evidence
of the essentiality of being saved.
Treatment
When people are bargaining, you should not offer them any
false hope. Although there may be practical things they can
do which you can offer them, never offer them something
that cannot be fulfilled.
Sometimes the best you can do at this stage is point even
more at the inevitable, even though this may well tip them
into depression (which may well be a necessary move).

When they are in a bargaining mood, sometimes there are


things you can offer them, such as support for change or
new opportunities. In these cases you may be able to strike
a win-win deal, where they get an improved deal and you
get collaboration or some other contribution. In a business
setting, this may include finishing off some important work
before they leave and receiving a special bonus for doing
so.

Depression stage: Final realization of the inevitable.

Symptoms
After denial, anger and bargaining, the inevitability of the
news eventually (and not before time) sinks in and the
person reluctantly accepts that it is going to happen. From
the animation of anger and bargaining, they slump into a
slough of despond. In this deep depression, they see only a
horrible end with nothing beyond it. In turning in towards
themselves, they turn away from any solution and any help
that others can give them.

Depression may be seen in a number of passive behaviors.


In the workplace, this includes physical absenteeism, long
lunch breaks and mediocre work performance. It can also
appear in tearful and morose episodes where the person's
main concern is focused on their own world.
Treatment
The first thing you can do with people who are in
depression is to be there with them, accepting them in all
their misery. People who are depressed feel very much
alone and you company, even though it may not seem that
way, is likely to be welcome.
The second thing to do is to keep them moving. It is easy
to get stuck in depression, and the longer they stay there,
the deeper into the mud they are likely to slide. So keep up
a steady stream of support, showing them that there is light
ahead and encouraging them to reach towards it.
In the workplace, provision of professional coaching,
counselling and other support can do a lot to help people

recognize their depression and find a way to clamber out of


the pit.

Testing stage: Seeking realistic solutions.

Symptoms
Even in the pit of depressive despair, reality eventually starts
to bite and the person realizes that they cannot stay in that
deep, dark hole forever. They thus start looking for realistic
things that they can do. These may be taken on as
'experiments' to see if doing these things help the situation
in any way. As this activity starts to work, at least in some
ways, it is found to be preferred to the depression and so
the person crawls out of that dark hole.
This escape is often done with the support of friends, family
and professionals who specialize in helping people in
whatever situation this is. In medicine, hospices help the
terminally ill face their short futures with courage. In
organizations, counselors and outplacement consultants help
individuals move on to other work.

Treatment
When they reach out towards the road to acceptance, they
are at last on their way out of the mire. Help them try
different solutions and to see that they can, after all, have
an effect on their future. Hand as much control to them as
possible, as this gives them a lifeline of stability on which to
pull themselves forward.

Acceptance stage: Finally finding the way forward.

Symptoms
The final stage is back to one of stability, where the person
is ready and actively involved in moving on to the next
phase of their lives, no matter how short. The terminally ill
person will be putting their life in order, sorting out wills and
helping others to accept the inevitability that then now have
countenanced and faced.
In the workplace, people who have lost their jobs will be
actively seeking new work, whilst others who have had their

work changed will be tidying up and getting ready to move


on.
Acceptance is typically visible by people taking ownership
both for themselves and their actions. They start to do things
and take note of the results, and then changing their actions
in response. They will appear increasingly happier and more
content as they find their way forward.
Treatment
Help the person to establish themselves permanently in their
new position. Fix them there, ensuring that there is no way
back to the previous situation. Congratulate them on getting
through the change. Celebrate the completion of their
transition.
Sticking and cycling
Getting stuck
A common problem with the above cycle is that people get
stuck in one phase. Thus a person may become stuck in
denial, never moving on from the position of not accepting

the inevitable future. When it happens, they still keep on


denying it, such as the person who has lost their job still
going into the city only to sit on a park bench all day.
Getting stuck in denial is common in 'cool' cultures (such as
in Britain, particularly Southern England) where expressing
anger is not acceptable. The person may feel that anger, but
may then repress it, bottling it up inside.
Likewise, a person may be stuck in permanent anger (which
is itself a form of flight from reality) or repeated bargaining.
It is more difficult to get stuck in active states than in
passivity, and getting stuck in depression is perhaps a more
common ailment.
Going in cycles
Another trap is that when a person moves on to the next
phase, they have not completed an earlier phase and so
move backwards in cyclic loops that repeat previous emotion
and actions. Thus, for example, a person that finds
bargaining not to be working, may go back into anger or
denial.

Cycling is itself a form of avoidance of the inevitable, and


going backwards in time may seem to be a way of
extending the time before the perceived bad thing happens.
http://changingminds.org/disciplines/change_management/kubler
_ross/
. C


Informed consent

(Psychiatry)

Part Psychiatry 2005


1.

regression

A.
B.

C.

D.

E.

2.

(temper tantrums) ( 4
)
A.
B.
C.
D.
3.

50 breast cancer metastasis


liver

A. denial
B. regression

C. repression
D. sublimitation
E. displacement
4.

30 1
2



A. dysthymic disorder
B. normal rief reaction
C. schizoaffective
D. major depressive symdrome
E. obsessive compulsive disorder

5.

38


A. social phobia
B. generalized anxiety disorder

C. obcessive compulsive disorder


D. agoraphobia with Hx of panic disorder
E. prostatism
6.

A. anxiety
B. insomnia
C. regression
D. repression
E. depression
7.

25 1

A. culture lag
B. culture trait

C. culture shock
D. culture change
E. culture relativity
8.

10

A. separation anxiety disorder


B. stranger anxiety ( )
C. (maltreatment)
D.

E.

Psychiatry
Part Psychiatry 2005
1. E

Regression
...
A.
displacement
C.

isolation of affect
D.
denial
B.


suppression (
)
( )
2. B
15-24
temper
tantrum ( )



temper tantrum
3. B

choice
A. denial

C. repression

D. sublimatation

E. displacement

B. regression

4. D.
Major depressive disorder (MDD)
subset depressive disorder (depressive disorder
subset mood disorder )



psychomotor retardation (
) psychomotor agitation (
)
( 60%)
dysthymic disorder MDD

schizoaffective disorder = schizophrenia + mood disorder


~ schizophrenia (
2 ) mood disorder

obsessive-compulsive disorder =

5. D
obsessive-compulsive disorder
- generalized anxiety disorder



/

/
- phobia 1.Specific phobia
2. Social
phobia

- prostatism ... symptoms caused by an enlarged


prostate gland, including difficulty with urination (
)
- agoraphobia panic disorder subset
anxiety disorder /

panic disorder panic attack


)
/ agoraphobia
panic attack
/ 3
1. panic disorder without agoraphobia
2.

panic disorder with agoraphobia

3. agoraphobia without history of panic disorder

panic disorder
panic attack panic

panic
attack D
6. A


(?)

Anxiety =

1.

2.

3.
4.

7. C

Culture shock refers to the anxiety and feelings (of


surprise, disorientation, uncertainty, confusion, etc.) felt when
people have to operate within a different and unknown
cultural or social environment, such as a foreign country.


1. (Material Culture)






2. (Non - material Culture)



(Culture Lag)

8. B

stranger anxiety 6-9 separation
anxiety 10-18
stranger anxiety

separation anxiety

You might also like